Wills & Trusts Panikowski Outline

Introduction to Estate Planning

Section A. The Power to Transmit Property at Death: Its Justification and Limitations

1.      The Right to Inherit and the Right to Convey

a.   Thomas Jefferson – “The earth belongs in usufruct to the living; the dead have neither powers nor rights over it. The portion occupied by any individual ceases to be his when he himself ceases to be, and reverts to society.”

b.   William Blackstone, Commentaries- The right of inheritance, or descent to the children and relations of the deceased, seems to have been allowed much earlier than the right of devising by testament. The permanent right to property is a civil not a natural right.

                                        i.     Will or Testament – written or oral instructions properly witnessed or authenticated according to the pleasure of the deceased.

c.   John Locke, Two Treatises of Government – Children have a right to inherit their parent’s possessions

d.  Irving Trust Co. v. Day – Rights of succession to property of a deceased, whether by will or by intestacy, are of statutory creation, and the dead hand rules succession only by sufferance. Nothing in the Federal Constitution forbids the legislature of a state to limit, condition, or even abolish the power of testamentary disposition over property within its jurisdiction.

Issue: Did the original version of the “escheat” provision of the Indian Land Consolidation Act of 1983, effect a taking of appellee’s’ decedents’ property without just compensation.

Holding: The Indian Land Consolidation Act of 1983 is unconstitutional due to the lack of just compensation, because it abrogates the constitutional right to descent and devise property to one’s heirs. Compensation must be paid if this right to devise stick is removed from the bundle.

Reasoning:

Arguments for:

This type of fractionation creates economic waste and does not promote productive use of the land. None of the appellees had investment backed interests in the land. The whole benefit derived by the tribe is greater than the fractionated losses.

Arguments Against:

It most likely is unconstitutional to completely abrogate the right to descent and devise property to one’s heirs.

Possible solution: It would be fair to force the owner to designate an heir to stop further fractionation.

                                             i.    Types of inter vivos transfers: joint tenancy, gift of a remainder interest, reserving of a life estate, often in a revocable trust; designating a death beneficiary on a contract, pension plan, or bank account.

                                            ii.    Trusts are not inherently more complex than wills.

                                          iii.    Some societies do not permit wills

                                          iv.    Many European countries do not allow children to be disinherited

                                           v.    In Louisiana, minor and disabled children may not be disinherited

2.   The Policy of Passing Wealth at Death

a.   Talcott Parsons The “Gift of Life” and its reciprocation. – We may regard death as a major contributor to the evolutionary enhancement of life, and thereby it becomes a significant part of the aggregate “gift of life” that all particular lives should end in death.” – death contributes to cultural growth.

b.   John A. Brittain, Inheritance and the Inequality of Material Wealth 13 (1978) – The less the rewards of wealth are associated with one’s own contribution, the better the case for taxing them…Inheritance remains one of the purest forms of ‘getting something for nothing.’”

                                             i.    Arguments in favor of inheritance

1.   In a society based on private property, it may be the least objectionable arrangement for dealing with property on the owner’s death.

2.   Inheritance is as natural and proper as both an expression and a reinforcement of family ties, which in turn are important to a healthy society and a good life.

3.   Serves as an incentive to bring forth creativity, hard work, initiative and ultimately productivity that benefits others, as well as encouraging individual responsibility

4.   Society will not have to support those who are supported via inheritance

5.   Induces saving and hence creates corporate accumulations

6.   Not having inheritance would increase wasteful consumption

7.   Social justice

8.   Should reap the reward of the fruits of your own labor

                                            ii.    Arguments not in favor

1.   Motivation for power and recognition outweigh the need for inheritance as a motivator

2.   Habit is also a good motivator

3.   Grants wealth to donees without requiring performance

4.   The transfer of great fortunes perpetuates wide disparities in the distribution of wealth and denies equality of opportunity to the poor.

5.   Estate taxes may be zero in the year 2010.

c.   Jeremy Bentham, The Theory of Legislation - The interests of the young in inheritance will ensure that they are dutious to their elders

d.   Melvin L. Oliver; Thomas M. Shapiro & Julie E. Press “Them That’s Got Shall Get” – The wealthiest Americans are not passing and are leaving large estates due to higher rates of saving, better wages and lower housing costs in the 40’s and 50’s

e.   De Tocqueville - Inherited wealth creates a danger of enduring privilege

f.    Forbes Magazine – The source of wealth for the 400 richest, only 1/3 inheritance; however, a majority seemed to have inherited some wealth

g.   Mark L. Ascher, Curtailing Inherited Wealth – About $150 billion pass each year at death, wealth transfer tax raised less than $8 billion, transfer taxes be inhibit freedom of testation. May increase opportunity and sources of revenue. Inheritance by healthy adult children would cease immediately. Estates of $250,000 or less would be untouched. There should be no inheritance with six exemptions:

                                             i.    Marital exemption, amount would depend on the length of the marriage

                                            ii.    Dependant lineal descendants

                                          iii.    Disabled lineal descendants

                                          iv.    Lineal ascendants unlimited

                                           v.    Moderate amounts of property

                                          vi.    Fixed fraction to charity, 20%

h.   Irving Kristol, Taxes, Poverty and Equality

i.     Beware of the inherited wealth oligarchy; no individual in a lifetime should inherit more than 1,000,000. It would be tax free. Institutional donations could be of any size. Any person could still be enormously wealthy in his lifetime.

j.    Inheritance in the Erstwhile Soviet Union – abolition of inheritance proved unpopular and it was reestablished. Relieved state of burden for caring for descendants.

k.   Walter J. Blum & Harry Kalven, Jr., The Uneasy Case for Progressive Taxation – Inheritance is both economic and cultural and not just about money, education, healthful diet, medical care

l.     John H. Langerbein, The Twentieth-Century Revolution in Family Wealth Transmission – Middle class transfers wealth in skills, education, transfer wealth by paying for education

3.   An Introduction to the Problem of Dead Hand – the use of wealth to influence behavior after death

a.   Restatement (Third) of Property: Wills and Other Donative Transfers (2003) § 10.1 Donor’s Intention Determines the Meaning of a Donative Document and Is Given Effect to the Maximum Extent Allowed by Law – The controlling consideration in determining the meaning of a donative document is the donor’s intention (by the maximum extent of the law).

                                             i.    (a) Rationale. The organizing principle of the American law of donative transfers is freedom of disposition. Property owners have the nearly unrestricted right to dispose of their property as they please…

                                            ii.    (c) Effective of a donative document – Unless disallowed by law, the donor’s intention not only determines the meaning but also the effect of a donative document.

                                          iii.    The government’s role is to facilitate the transfer of property, but it will intervene if the disposition is not legal.

1.   Rules if law that prohibit or restrict freedom of disposition

a.   Spousal rights

b.   Creditor’s rights

c.   Unreasonable restraints on alienation or marriage

d.   Provisions on promoting separation or divorce

e.   Impermissible racial or other categorical restrictions

f.    Encouragement of illegal activity

g.   Rules against perpetuities and accumulations

2.   Shapira v. Union National Bank (marry Jewish woman within 7 years or to State of Israel)

Holding: The conditions of the will are reasonable restrictions on marriage and are valid.

Reasoning:

Constitutionality

The right to marry is constitutionally protected. However, in the case at hand, the court is not being asked to enforce any restriction upon the heir apparent. The right to receive property by law is a creature of the law and not a natural right.

Public Policy

The condition that Daniel share would be turned over to him if he married a Jewish girl is a partial restraint on marriage. Most US authority  states that gifts conditioned upon the beneficiary’s marrying within a particular religious class or faith are reasonable. Supposedly, this restraint does not restrain the practice of religious freedom as does imposing a restraint to raise a child in a particular faith.

Maddox v. Maddox – There were only 5 men in the area who were a member of the religion. This was considered a restraint on religious practice and it would have been a prohibition on marriage. There was an absence of a gift over in this case. There are many more eligible Jewish women available for Daniel.

a.   A dead hand without a live mind cannot adapt as circumstances change

                                          iv.    Restatement of Property 2nd: Donative Transfers § 6.2 (1983) – a restraint to induce a person to marry within a religious faith is valid if and only if under the circumstances the restraint does not unreasonably limit the transferee’s opportunity to marry. (a) the restraint unreasonably limits the transferee’s opportunity to marry if a marriage permitted by the restrain is not likely to occur.

                                           v.    A will or trust provision is ordinarily invalid if it is intended or tends to encourage disruption of a family relationship. Like those that encourage separation or divorce.

1.   Donner – held back trust til age 65 unless daughter became divorced or husband died – was upheld because the intent was to provide support if daughter became divorced or husband died.

2.   Schmitz – can not use wealth to stop members of family from communicating

                                          vi.    Destruction of property at death

1.   The right to destroy property after death limits waste while the person is alive when they can enjoy the benefit

2.   But isn’t the net waste after death more

                                        vii.    Restatement (Third) of Trusts § 29 invalidates trusts that are contrary to public policy – should prohibit waste, but balance social values – disfavors restraints on marriage or religious freedom, disrupts family relationships, and choice of careers

Section B. Transfer of the Decedent’s Estate

1.      Probate and Nonprobate Property

a.       Probate property – property that passes under the decedent’s will or by intestacy.

b.      Nonprobate property – property passing under an instrument other than a will. Most property is passed this way.

                                                              i.      Joint tenancy property, both real and personal

1.      Decedent’s interest vanishes at death

2.      Survivor has the whole property relieved of the decedent’s participation

3.      No interest passes to the survivor at the decedent’s death

4.      Survivor must file a death certificate

                                                            ii.      Life Insurance

1.      Life insurance proceeds of a policy on the decedent’s life are paid by the insurance company to the beneficiary named in the insurance contract

2.      The company will pay on receipt of the death certificate

                                                          iii.      Contracts with payable-on-death provisions

1.      Pension plans

2.      Tax deferred investment plans

                                                          iv.      Interests in trust

1.      When property is transferred in trust, the trustee holds the property for the benefit of the named beneficiaries, who may have life estates, or remainders or other types of interests.

2.      The property is distributed to the beneficiaries by the trustee in accordance with the terms of the trust instrument.

a.       If the trust was created by the decedent, the trust may be revocable or irrevocable

b.      If the decedent has a testamentary power of appointment over assets in the trust, the decedent’s will must be admitted to probate, but the trust assets are distributed directly by the trustee to the beneficiaries named in the will and do not go through probate

2.      Administration of Probate Estates

a.       History and Terminology

                                                              i.      Personal representative – oversees the winding up of the decedent’s affairs when they die if probate is necessary

1.      Inventory and collect the assets of the decedent

2.      Manage the assets during administration

3.      Receive and pay the claims of creditors and tax collectors

4.      Clear any titles to cars, real estate, or other assets

5.      Distribute the remaining assets to those entitled

6.      They are usually selected by the probate court in this order

a.       Surviving spouse

b.      Children

c.       Parents

d.      Siblings

e.       Creditors

                                                            ii.      Executor – Executes the will and administers the probate estate when a person dies testate

                                                          iii.      Probate court – appoints, controls and evaluates personal representatives

1.      The person appointed as an administrator (personal rep or executor) must give a personal bond

2.      Most wills waive the bond requirement

3.      In some states non resident executors and fiduciary representatives cannot serve as executors

                                                          iv.      Will (testament) – an instrument disposing of both real and personal property

                                                            v.      Devise – to give real property to heirs after death via a will

                                                          vi.      Bequeath – to give personal property via a will

                                                        vii.      Now, devise and bequeath and give may be used

                                                      viii.      Legatees

                                                          ix.      Descends to Heirs – real property is given to heirs

                                                            x.      Distributed to next of kin- personal property is given to next of kin

                                                          xi.      Statute of descent and distribution – governs intestacy

                                                        xii.      Heirs (Next-of-kin) – those persons designated by the applicable statute to take a decedent’s intestate property, both real and personal

                                                      xiii.      Curtesy or dower rights - @CL spouse was not an heir, the right to take some of the decedent’s personal property

b.      A Summary of Probate Procedure

                                                              i.      Opening probate

1.      Functions of probate

a.       Provides evidence of transfer of title to the new owners by a probated will or decree of intestate succession;

b.      It protects creditors by requiring payments of debts

c.       It distributes the decedent’s property to those intended after the creditors are paid

2.      Letters of testamentary (executor) – authorizes person to act on behalf of the estate

3.      Letters of administration (administrator) – authorizes person act on behalf of the estate

4.      Operations in the East

a.       Common form – an ex parte proceeding in which no notice or process was issued to any person

5.      Majority of States – do not permit ex parte proceedings, but require prior notice to interested parties before the appointment of a personal representative of a probate will

a.       The petition of letters must be accompanied by an affidavit state that the statutory notice requirements have been met (personal service, mailing or publishing

b.      If a will is to be probated it must be proved by the testimony or affidavits of the witnesses

6.      The Uniform Probate Code (UPC), originally promulgated in 1969, revised in 1990, and adopted in a number of states, is representative of statutes, regulating probate procedures. It provides for both ex parte and probate and notice probate. (No proceeding can be initiated 3 years after death under the UPC §3-108)

a.       Informal probate, UPC §3-301 – ex parte

                                                                                                                                      i.      Representative without giving notice to anyone petitions for appointment

                                                                                                                                    ii.      If for will, will must accompany petition

                                                                                                                                  iii.      Executor swears that the will was validly executed (no witness necessary)

                                                                                                                                  iv.      UPC § 303 – A will that appears to have the required signatures and that contains an attestation clause showing that requirements have been met is probated by the registrar without further proof.

                                                                                                                                    v.      UPC § 3-705 – Within 30 days after appointment, the personal representative has the duty of mailing notice to every interested person, including heirs apparently disinherited by a will.

b.      Formal proceedings

                                                                                                                                      i.      A judicial determination after notice to interested parties.

                                                                                                                                    ii.      Any interested party can demand formal probate.

                                                                                                                                  iii.      A formal proceeding may be used to probate a will, to block an informal proceeding or to secure a declaratory judgment of intestacy.

                                                                                                                                  iv.      Formal proceedings become final judgments if not appealed.

7.      Time for Contest – The time for contesting probate of a will is dependent upon the statute in the jurisdiction, must be constitutional

8.      Barring creditors of the decedent –

a.       Non claim statutes - Every state has a statute requiring creditors to file claims within a specified time period

b.      Supreme Court has held that the Due Process Clause requires that known or reasonably ascertainable creditors receive actual notice before they are barred by a short-term statute running from the commencement of probate proceedings.

                                                                                                                                      i.      Either bar claims not filed within a relatively short period after probate proceedings are begun, generally two to six months

                                                                                                                                    ii.      UPC § 3-308 - Or whether or not probate proceedings are commenced, they bar claims not filed within a longer period after the decedents death, generally one to five years

                                                            ii.      Supervising the representative’s actions

1.      The UPC authorizes both supervised an § 3-715 unsupervised administration

a.       Collects assets

b.      Clears titles

c.       Sells property

d.      Invests other assets

e.       Pays creditors

f.       Continues business

g.      Distributes the estate

2.      If the interests of a minor are involved, the court will most often supervise

3.      § 3-502 If any party demands supervision, the probate court will supervise

                                                          iii.      Closing the estate

1.      The personal representative of an estate is expected to complete the administration and distribute the assets as promptly as possible.

a.       Creditors must be paid

b.      Titles must be cleared

c.       Taxes must be paid and tax returns audited and accepted by the appropriate authorities

d.      Real estate or a sole proprietorship may have to be sold

e.       Judicial approval of the personal representatives actions, court must grant discharge to relieve personal representative from duty

c.       Is Probate Necessary?

                                                              i.      Administrative costs of probate:

1.      Probate court fees

2.      Commission of the personal representative

3.      The attorney’s fee (federal income tax deductible)

4.      Sometimes appraiser

5.      Sometimes guardian ad litem

                                                            ii.      Estates of 2,000,000 are exempt in 2007-2008

                                                          iii.      Probate can be avoided by transferring all property into a joint tenancy or a revocable or irrevocable trust or in many states executes a contract providing for distribution of contract assets to named beneficiaries on the owner’s death.

                                                          iv.      It is assumed that the possessor has title of furniture and such things

                                                            v.      For personal property that has a title or document specifying ownership, the transferee needs some official recognition of his rights to the property

                                                          vi.      Many states permit close relatives of the decedent to obtain possession of the decedent’s personal property by presenting an affidavit to the holder of the property. The amount of the property TBD by statute.

                                                        vii.      20% of estates in CA go through probate

d.      Universal Succession – the heirs or the residuary devisees succeed to the title of all of the decedent’s property; there is no personal representative appointed by a court. The heirs become tenants in common.

                                                              i.      The UPC authorizes universal succession as an alternative to probate administration. No state has yet adopted these provisions of the UPC.

                                                            ii.      Under CA law, property that passed to the surviving spouse by intestacy or by will is not subject to administration unless the surviving spouse elects to have it administered.

                                                          iii.      If the surviving spouse takes title to the property and assumes personal liability for the decedent’s debts chargeable against the property.

Section C. An Estate Planning Problem

1.     The Client’s Letter and Its Enclosures

2.     Questions

a.       Must consider debts like mortgage

b.      Death taxes

c.       Appointment of another executor upon the death of currently assigned executor

d.      Arrangement if they die before their children

3.     Additional Data on the Browns’ Family Assets

Section D. Professional Responsibility

a.      Duties to Intended Beneficiaries

a.       Simpson v. Calivas (homestead to wife, ambiguous, malpractice lawyer)

Issues: Did the attorney owe a duty to an intended beneficiary? Did the findings of the probate court collaterally estopp the d from bringing action?

Holding: The attorney had a duty to the intended beneficiaries.

Rule: In order to recover for negligence, a p must show that “there exists a duty whose breach by the d causes the injury for which the p seeks to recover.” Must show a relationship and reasonably foreseeable harm.

Reasoning:

Duty to Intended Beneficiaries

Morvay – Duty may result from reliance on the performance listed in the contract without privity.

R. Mallen – duty runs from an attorney to an intended beneficiary of a will.

CA Supreme Court – duty to the beneficiary is greater than to the testator in some ways

Heyer – even though no privity, foreseeability of injury to the beneficiary is great

Estate of Wood, Third party-beneficiaries may make a claim of negligence.

Collateral Estoppel

Collateral estoppel is only applicable if the finding in the first proceeding was essential to the judgment of the court.  A finding of actual intent is not necessary to that judgment. Accordingly even an explicit finding of actual intent by a probate court cannot be the basis for collateral estoppel.

b.     Conflicts of Interest

a.       Hotz v. Minyard

Issue: Did p’s father’s attorney breach his fiduciary duty to her by misrepresenting her father’s will in January 1985.

Holding: Yes, the attorney breached his fiduciary duty to Judy.

Rule: A fiduciary relationship exists when one has a special confidence in another so that the latter, in equity and good conscience, is bound to act in good faith.

Reasoning: The attorney and the firm had acted as done her accounting and legal work for the past 20 years. He had a duty to not actively misrepresent the first will.

b.      It is a good idea if there are conflicts of interest to have a separate discussion with each member of the family and if needed have a written representation agreement including possible waivers of conflicts of interest might be required in some situations.

c.       Lawyers are pressing for less malpractice for wills, they are asking that judges be able to reform mistakes after the death of the testator


Chapter 2 Intestacy: An Estate Plan by Default

 

Section A. The Basic Scheme

 

1.      Introduction

a.       Intestate – without a valid will

b.      Partial intestacy- when a will is so poorly drafter that it disposes of only part of the probate estate

c.       The law of the state where the decedent was domiciled at death governs the disposition of personal property,

d.      The law of the state where the decedent’s real property is located governs the disposition of her real property

e.       Uniform Probate Code (1990)

                                                                                                        i.      § 2-101. Intestate Estate

1.      (a) Any part of a decedent’s estate not effectively disposed of by will passes by intestate succession to the decedent’s heirs as prescribed in this Code, except as modified by the decedent’s will

2.      (b) A decedent by will may expressly exclude or limit the right of an individual or class to succeed to property of the decedent passing by intestate succession. If that individual or a member of that class survives the decedent, the share of the decedent’s intestate estate to which that individual or class would have succeeded passes as if that individual or each member of that class had disclaimed his [or her] intestate share.

                                                                                                      ii.      § 2-102 Share of Spouse - The intestate share of a decedent’s surviving spouse is

1.      the entire estate if:

a.       (i) no descendant or parent of the decedent survives the decedent; or

b.      (ii) all of the decedent’s surviving descendants are also descendants of the surviving spouse and there is no other descendant of the surviving spouse who survives the decedent;

2.      The first $200,000, plus ¾ of any balance of the intestate estate, if no descendant of the decedent survives the decedent, but a parent of the decedent survives the decedent;

3.      The first [150,000], plus one-half of any balance of the intestate estate, if all of the decedent’s surviving descendants are also descendants of the surviving spouse and the surviving spouse has one or more surviving descendants who are not descendants of the decedent;

4.      The first [100,000], plus ½ of any balance of the intestate estate, if one or more of the decedent’s surviving descendants are not descendants of the surviving spouse

                                                                                                    iii.      § 2-103. Share of Heirs Other than Surviving Spouse – Any part of the intestate estate not passing to the decedent’s surviving spouse under Section 2-102 or the entire intestate estate if there is no surviving spouse, passes in the following order to the individuals designated below who survive the decedent:

1.      to the decedent’s descendants by representation

2.      if there is no surviving descendant, to the decedent’s parents equally if both survive, or to the surviving parent;

3.      if there is no surviving descendant or parent, to the descendants of the decedent’s parents or either of them by representation

4.      if there is no surviving descendant, parent, or descendant of a parent, but the decedent is survived by one or more grandparents or descendants of grandparents, half of the estate passes to the decedent’s paternal grandparents equally if both survive, or to the surviving paternal grandparents or either of them if both are deceased, the descendants taking by representation; and the other half passes to the decedent’s maternal relatives in the same manner; but if there is no surviving grandparent or descendant of a grandparent on either the paternal or the maternal side, the entire estate passes to the decedent’s relatives on the other side in the same manner as the half

                                                                                                    iv.      § 2-105. No Taker

1.      If there is no taker under the provisions of this Article, the intestate estate passes to the [state].

                                                                                                      v.      The Meaning of Heirs and the Transfer of an Expectancy – no living person has heirs, they have heirs apparent, the heirs apparent have a mere expectancy

2.      Share of Surviving Spouse

a.       Most estates favor the spouse inheriting everything, especially for those of middle income, those with higher incomes will distribute more among collaterals

b.      Current law usually provides only ½

c.       Family protection – preserving the economic health of the family after a death

d.      Mandatory minimum share [for spouse] – a share to which the law entitles a surviving spouse in spite of a contrary will by the decedent

e.       1996 Congress passed the Defense of Marriage Act, restricting the extension of spousal rights to same-sex couples under federal programs and providing that states could not be forced to recognize same-sex marriages performed in other states. In response, 35 states enacted similar enacted similar statutes and 3 banned same-sex marriages in their state constitutions.

                                                                                                        i.      Hawaii – reciprocal beneficiaries

                                                                                                      ii.      VT – civil unions – open only to same-sex couples, but gives many of the same rights as marriage

                                                                                                    iii.      CA domestic partners – applies to same sex partners and opposite sex couples where one of the partners is 62 older

                                                                                                    iv.      MA allows same-sex marriages, but only for MA residents

                                                                                                      v.      Proposed amendment to the UPC §2-202B—that would have provided an intestate share for “committed partners.” A person sharing a common household with the decedent in a marriage-like relationship. Both same sex and opposite sex.

CPC S 6402.5: PREDECEASED SPOUSE; PORTION OF DECEDENT'S ESTATE ATTRIBUTABLE TO DECEDENT'S PREDECEASED SPOUSE

 

            (a) REAL property attributable to PDS passes as follows if PDS died not more than 15 years       before D, AND no SS or issue of the D.

                        *1. if D survived by issue of PDS, to the surviving issue of PDS equally.

                        2. parent(s) of PDS who survive D equally.

                        *3. issue of parent of PDS (siblings) equally

                        4. next of kin of the decedent.

                        5. if portion attributable to PDS would otherwise escheat, to next of kind of PDS.

 

            (b) PERSONAL property attributable to PDS passes as follows if PDS died not more than five     years before D.

                        *1. if D survived by issue of PDS, to the surviving issue of PDS equally.

                        2. parent(s) of PDS who survive D equally.

                        *3. issue(s) of parent of PDS (siblings) equally.

                        4. next of kinds of decedent.

                        5. if portion attributable to PDS would otherwise escheat, to next of kin of PDS.

            **(c): Claimant heir bears BOP to show the exact personal property to be disposed of to the heir.

            **(d): Notice: if aggregate fair market value is believed in good faith by the petitioning part to be             less than $10K, the petitioning party need NOT give notice to the issue or next of kin of PDS.

            -if the PP is subsequently determined > $10K, notice shall be given to the issue or next of kin of   PDS as provided by law.

            **(e): 'Personal Property' for section (b): personal property in which there is a written record of     title or ownership AND value in aggregate is $10K or more.

            **(f): 'portion of decedent's estate attributable to PDS' means all of following in D's estate:

                        1) 1/2 of CP in existence at time of death of PDS.

                        2) 1/2 of CP in existence at time of death of PDS which was given to D by PDS by way of                      gift, descent, or devise.

                        3) portion of any CP in which PDS had any incident of ownership and which vested in D                                     upon death of PDS by right of survivorship.

                        4) any SP of PDS which came to decedent by gift, descent, or devise of PDS, OR which                           vested in D upon death of PDS by right of survivorship.

                        **(g): quasi-community property shall be treated the same as CP.

                        **(h): for purposes of this section:

                                    1. Relatives of PDS conceived before the decedent's death but born thereafter                                           inherit as if they had been born in the lifetime of D.

                                    2. A person related to PDS through two lines of relationship is entitled to only a                                        single share based on the relationship which would entitle the person tot he larger                          share.

PDS = predeceased spouses

SS = surviving spouse

D = decedent

CP = community property

SP = separate property

BOP = burden of proof

* = if of unequal degree of kinship, those of more remote degree take in the manner provided in Section 240 (I don't think we were given this).

                                                                                                    vi.       

f.       Simultaneous death. – A person succeeds to the property of a decedent only if the person survives the decedent for an instant of time.

                                                                                                        i.      Uniform Simultaneous Death Act (USDA) – there is no sufficient evidence of the order of deaths, the beneficiary is deemed to have predeceased the donor. – avoids double taxation, most well drafter wills require the decedent to to have survived the testator by 30 to 60 days. Clear and convincing evidence of survival is required under the USDA and the UPC.

                                                                                                      ii.      Janus v. Tarasewicz

Issue: Was there sufficient evidence that Theresa Janus survived her husband?

Holding: There was sufficient evidence that Thresa died after her husband.

Rule: Survivorship is a fact which must be proven by a party whose claim depends on survivorship. Uniform Simultaneous Death Act: If the title to property or its devolution depends upon the priority of death and there is no sufficient evidence that the persons have died otherwise than simultaneously and there is no other provision in the will, trust agreement, deed, contract of insurance or other governing instrument for distribution of the property different from the provisions of this Section: (a) The property of each person shall be disposed of as if he had survived…(d) If the insured and he beneficiary of a policy of life or accident insurance have so died, the proceeds of the policy shall be distributed as if the insured had survived the beneficiary.

Reasoning: They only need have sufficient evidence that she survived her husband. There was sufficient evidence based on her vital signs and other circumstances.

3.      Share of Descendants

a.       After the spouse’s share if any is set aside, children and issue of the deceased children take the remainder of the property to the exclusion of everyone else.

b.      Representation - When one of several children has died before the decedent, leaving descendants, all states provide that the child’s descendants shall represent the dead child and divide the child’s share among themselves.

                                                                                                        i.      Sons-in-law and daughters-in-law are excluded as intestate successors in virtually all states. Legislatures have decided it should escheat to the state.

                                                                                                      ii.      English per stirpes – Divide the property into as many shares as there are living children of the designator person and deceased children who have descendants living. The children of each descendant represent their deceased parent and are moved into their parent’s position beginning at the first generation below the designated person.

                                                                                                    iii.      Modern per stirpes or per capita representation. – Where no children survive the decedent, the distribution is identical to that under English per stirpes. However, where no children survive the decedent, then the estate is divided equally (per capita) at the first generation in which there are living takers, which is usually the generation of the decedent’s grandchildren. This system treats each line beginning at the closest living generation equally

                                                                                                    iv.      Per Capita at Each generation – If under Section 2-103(1) a decedent’s intestate estate or a part thereof passes by representation to the decedent’s descendants, the estate or part thereof is divided into as many equal shares as there are (i) surviving descendants in the generation nearest to the decedent which contains one or more surviving descendants and (ii) Deceased descendants in the same generation who left surviving descendants, if any. Each surviving descendant in the nearest generation is allocated one share. The remaining shares, if any are combined and then divided in the same manner among the surviving descendants of the deceased descendants as if the surviving descents who were allocated a share and their surviving descendants had predeceased the descendant.

                                                                                                      v.      UPC §2-106(b) – the initial division of shares is made at the level where one or more descendants are alive (as under modern per stirpes), but the shares of deceased persons on that level are treated as one pot and are dropped down and divided equally among the representatives on the next generational level. Each generational level is treated equally.

                                                                                                    vi.      Negative Disinheritance – an express statement in their wills disinheriting a child,

1.      It is also necessary that the entire estate be devised to other persons. If there is a partial intestacy for some reason, the disinherited person will take an intestate share notwithstanding such a scheme without giving the property to others

2.      § 2-101(b)(1990), authorizes a negative will. The barred heir is treated as if he disclaimed his intestate share, which means he is treated as having predeceased the intestate.

4.      Shares of Ancestors and Collaterals

a.       When the intestate decedent is survived by a descendant, the decedent’s ancestors and collaterals do not take.

b.      When there is no descendant, after deducting the spouse’s share, in nearly half the states the rest of the intestate’s property is usually distributed to the decedent’s parents, as under the UPC.

c.       Collateral kindred – all persons who are related by blood to the decedent but who are not descendants or ancestors.

d.      First-line collaterals – descendants of the decedent’s parents, other than the decedent and the decedent’s issue.

e.       Second-line collaterals – descendants of the decedent’s grandparents, other than decedent’s parents and their issue

f.       Parentelic system – the intestate estate passes to grandparents and their descendants and if no to great-grandparents and their descendants and if no to great-great-grandparents and their descendants

g.      Degree of relationship system – intestate estate passes to the closest kin, counting degrees of kinship. To count the steps (counting one for each generation) up from the decedent to the nearest common ancestor of the decedent and the claimant, and then you count the steps down to the claimant from the common ancestor.

h.      Massachusetts General Laws (2004) – If the descendent leaves no issue, and no father, mother, brother or sister, and no issue of any deceased brother or sister, the to his next of kin in equal degree, but if there are two or more collateral kindred in equal degree claiming through different ancestors, those claiming through the nearest ancestor shall be preferred to those claiming through an ancestor more remote.

i.        In CA, step-children, parents-in-law and brothers and sisters in law are allowed to inherit

j.        Half-bloods – England puts a great weight on whole blood relations. Half-blooded relatives cannot inherit land through intestate succession.

                                                                                                        i.      §2-107 – Treats half-bloods are treated the same as blood relatives.

                                                                                                      ii.      Various states treat half-bloods differently, for instance giving half-bloods a half share.

Section B. Transfers to Children

  1. Meaning of Children
    1. Adopted Children

b.      Hall v. Vallandingham

Issue: Do children that have been adopted have the right to inherit from the natural parent by way of representation.

Holding: They do not have the right to inherit from their natural parent by way of representation.

Reasoning: The primary purpose for adoption was, and still is, inheritance rights.

Rule: Maryland Code, Family Law Article Ann. § 5-308.

(b) After a decree of adoption is entered

(1) the individual adopted:

(i)          is the child of the petitioner for all intents and purposes; and

(ii)          is entitled to all the rights and privileges of and is subject to all the obligations of a child born to the petitioner in wedlock;

(2) Each living natural parent of the individual is adopted is

(i)  relived of all parental duties and obligations to the individual adopted and

(ii) divested of all parental rights as to the individual adopted; and

(3) all rights of inheritance between the individual adopted and the natural

relations shall be governed by the Estates and Trusts Article.  Md. Estates and Trusts Code Ann. § 1-207(a): An adopted child shall be treated as a natural child of his adopted parent or parents. On adoption, a child no longer shall be considered a child of either natural parent, except that upon adoption by the spouse of a natural parent, the child shall be considered the child of that natural parent.

When children were adopted, the code stated adopted children retained the right to inherit from their natural parent.

Reasoning: Limiting dual inheritance…what is so bad about this???

c.       § UPC 2-113. Individuals Related to Decedent Through Two Lines – An individual who is related to the decedent through two lines of relationship is entitled to only a single share based on the relationship that would entitle the individual to the larger share.

d.      Minority rule § UPC 2-114. Parent and Child Relationship – (a) Except as provided in subsections (b) and (c), for purposes of intestate succession by, through, or from a person, an individual is the child of his [or her] natural parents, regardless of their marital status. The parent and child relationship may be established under [the Uniform Parentage Act][applicable state law][insert appropriate statutory reference]. (b) An adopted individual is the child of his [or her] adopting parent or parents and not of his [or her] natural parents, but adoption of a child by the spouse of either natural parent has no effect on (i) the relationship between the child and that natural parent or (ii) the right of the child or a descendant of the child to inherit from or through the other natural parent. (c) Inheritance from or through a child by either natural parent or his [or her] kindred is precluded unless that natural parent has openly treated the child as his [or hers], and has not refused to support the child.

e.       States vary their rules on these issues, some adoptive parent inheritance only, some natural parent only, some both, some both and relatives of both

f.       Adult adoption – adults can adopt adults and these adoptions are treated the same as children for the most part for the purposes of inheritance. However, NY will not allow lovers to adopt their lovers.

g.      Stranger to the adoption rule – The adopted child is presumptively barred, whatever generic word is used, except when the donor is the adoptive parent – in most states today adopted children are included as issue, children, descendants and heirs.

h.      Minary v. Citizens Fidelity Bank & Trust Co.

Issue: Should an adopted wife be able to inherit as an heir.

Holding: An adopted wife cannot inherit if it thwarts the intent of the testator.

Rule: Where no language shows a contrary intent…an adopted daughter clearly falls within the class designated as heirs at law. KRS 199.520 from and after the date of the judgment the child shall be deemed the child of petitioners and shall be considered for purposes of inheritance and succession and for all other legal considerations, the natural, legitimate child of the parents adopting it the same as if born of their bodies.

Reasoning: A heir should not be able to use adult adoption to go around the intent of the testator.

i.        Special power of appointment – the use of the adoption procedure for the purpose of creating a child to come within a class gift is in effect using adoption as a special power of appointment

j.        O’Neal v. Wilkes

Issue: Did the court correctly determine that Page was without authority to contract O’Neals adoption.

Procedural History: Executor refused to recognize O’Neal as an heir. O’Neal filed a petition for a virtual adoption.

Rule: The first essential of a contract for adoption is that it be made between persons competent to contract for the disposition of the child. P must prove some showing of an agreement between the natural and adoptive parents, performance by the natural parents of the child in giving up custody, performance by the child by living in the home of the adoptive parents, partial performance by the foster parents in taking the child into the home and treat it as their child, …. The intestacy of the foster parent.

The Georgia Code defines legal custodian as a person to whom legal custody has been given by court order and who has the right to physical custody of the child and to determine the antue of the care and treatment of the child and the duty to provide for the care, protection, training and education and the physical, mental and moral welfare of the child and does not have the right to consent to adoption.

Reasoning: Page was without authority to contract for the O’Neals in their daughters adoption.

Dissent: Sears J. – Equity considers that which ought to have been done. Foster parent has acted in loco parentis. Child should obtain estate. Full performance of a contract negates its unenforceability. Child was not of the age to ensure that the contract was consented to when it was made. The equitable reasoning does not focus on the “fiction of the contract”, but rather on the relationship of the parent and the child.

Notes: Lankford, the effect of equitable adoption should be limited to the parent and not extended to brothers and sisters.

Estate of Ford: Equitable adoption must be proved by clear and convincing evidence in CA.

b.      Posthumous Children – child who is conceived before but born after one of their parent’s death usually the father

                                i.      Child is treat as in being from the time of conception because it is to their advantage (if born alive)

                              ii.      There is a rebuttable presumption that a child born more than 280 days after the decedent’s death is not the d’s child; however, the child may prove otherwise

                            iii.      Uniform Parentage Act § 204 – allows for 300 rather than 280 days

c.       Nonmarital Children

                                i.      All jurisdictions permit nonmarital children to inherit out of wedlock

                              ii.      Trimble v. Gordon – held unconstitutional, as a denial of protection, an Illinois statute denying a nonmarital child inheritance rights from the father. The court held that state discrimination against nonmarital children, although not a suspect classification subject to the strict scrutiny test, must have substantial justification as serving an important state interest. There must be reliable proof of paternity

                            iii.      Paternity can now be established by the subsequent marriage of the parents, acknowledgement by the father, an adjudication during the life of the father, or clear and convincing proof after f’s death.

1.      UPA (Uniform Parentage Act) – marital status, living with parent and held out as child when less than age 2

2.      Acknowledges paternity in writing

3.      Courts in NY are now allowing for more posthumous DNA testing (Brancato)

4.      Hoffoauir – allow for DNA testing of surviving relatives

d.      Reproductive Technology and New Forms of Parentage

                                i.      Hecht – woman may be given sperm via will

                              ii.      Woodward v. Commissioner of Social Security

Issue: If a married man and woman arrange for sperm to be withdrawn from the husband for the purpose of artificially impregnating the wife, and the woman is impregnated with that sperm after the man, her husband, has died, will children resulting from such pregnancy enjoy the inheritance rights of natural children under MA law of intestate succession?

Holding: In certain limited circumstances, a child resulting from posthumous reproduction may enjoy the inheritance rights of “issue under the MA intestacy statute.

Rule: There must be evidence that the deceased intestate parent affirmatively consented (1) to the posthumous reproduction and (2) to support any resulting child

Reasoning: Children no matter how conceived should have the same rights. The legislature holds that it supports reproductive technology. It is inconsistent to not give children produced via this technology the same rights as those produced via natural birth. A posthumously conceived child will lower the share of the estate available to other children. Must uphold reproductive rights while also looking at the statute of limitations. Other types of use of frozen gametes must be taken into consideration.

                            iii.      SSA has not been amended to account for posthumous children yet

                            iv.      Kolacy and Barnhart – also cases that approve posthumous children as intestate heirs

                              v.      UPA §707 says that deceased parent must have consented on record to being the child’s parent

                            vi.      CA statute allows posthumous children as heirs, but is more restrictive than the UPA – Must be in writing, clear and convincing evidence, notice must be give to estate admin within 4 mos of death and child must be in utero within two years

                          vii.      What about the rule of perpetuities

e.       Surrogate Motherhood and Married Couples

                                i.      Johnson v. Calvert – parentage is TBD by the parties involved in the surrogate contract

                              ii.      J.R. v. Utah – child cannot be adopted without the consent of both biological parents (one being the surrogate)

                            iii.      Marriage of Buzzanca – if parent agrees via k to be parent, they must support child

                            iv.      Jane Doe v. John Doe – genetic mother not mother for financial purposes, but if in the best interests of the child, custody may be turned over to her

                              v.      In England, legal adoption must take place for genetic mother or father to take legal custody of the child

f.       Assisted Reproduction and Same-Sex Couples

                                i.      Adoption of Tammy – child may be adopted by same sex partner of natural mother

                              ii.      King – when two women involved in a domestic relationship agree to bear and raise a child together by artificial insemination of one of the partners with donor semen, both women are the legal parents of the resulting child

4.     Advancements

a.       Advancements – If any child wishes to share in the intestate distribution of a deceased parents estate, the child must permit the admin to include in the determination of the distributive shares the value of any property that the decedent while living gave the child by way of an advancement.

b.      To not be included they must prove that the property was an absolute gift

c.       Advancements are deducted from a child’s descendants share when the child predeceases the parent

d.      Hotchpot – The amount of the advancement is added to the total amount of the estate and then the estate is divided according to the law among the other heirs, if the amount is more than their share, they do not have to give any back because the state recognizes that the donor wanted them to have at least that much

e.       UPC

                                i.      § 2-109 Advancements – (a) if an individual dies intestate as to all or a portion of his [or her] estate, property the decedent gave during the decedent’s lifetime to an individual who, at the decedent’s death is an heir is treated as an advancement against the heir’s intestate share only if (i) the decedent declared in a contemporaneous writing or the heir acknowledged in writing that the gift is an advancement or (ii) the decedent’s contemporaneous writing or the heir’s written acknowledgement otherwise indicates that the gift is to be taken into account in computing the division and distribution of the decedent’s intestate estate. (b) For purposes of subsection (a), property advanced is valued as of the time the heir came into possession or enjoyment of the property or as of the time of the decedent’s death whichever first occurs. (c) if the recipient of the property fails to survive the decedent, the property is not taken into account in computing the division and distribution of the decedent’s intestate estate, unless the decedent’s contemporaneous writing provides otherwise

                              ii.      This legislation calling for advancements to be in writing avoids costly litigation over lifetime events

3. Guardianship and Conservatorship of Minors

a.    Guardian of the Person – a guardian of the person has responsibility for the minor child’s custody and care.

                          i.      If both parents die without specifying a guardian, the court will appoint one for them. Nearest relative.

                        ii.      Guardianship of the person terminates when the child reaches the age of majority, dies or is adopted.

b. Property Management Options – A guardian of the person has the right to deal with the child’s property

                      iii.       Guardianship of the property

1.      Does not have title to the ward’s property

2.      Cannot change investments without a court order

3.      Duty of preserving the specific property left the minor and delivering it to the ward at age 18, unless the court approves a sale, lease or mortgage

4.      Can only use the income from the property to support the ward

5.      Expensive to go to court

                      iv.      Conservatorship

1.      UPC Article V 1998

2.      Uniform Guardianship and Protective Proceeding Act 1997

a.       Guardian now called conservator

b.      Appointment and supervision by court still required

c.       More flexible powers

d.      One trip to courthouse for accounting

e.       Terminates when reaches age of majority or dies UPC § 5-431

f.       Court does not become involved unless the minor contests

3.      Custodianship

a.       UTMA – custodian is given property to hold for the benefit of a minor under either the Uniform Transfers to Minors Act (UTMA)

b.      Facility of payment clause – assets to be distributed outright to a minor may be paid instead to a custodian or even to the parent or guardian of the minor

c.       § 6 Payments to custodians over $10,000 require court approval

d.      Must transfer property to minor when he reaches age of majority

e.       UTMA §12 Has the right to transfer property, but is subject to the standard of care that would be observed by a prudent person dealing with the property of another

4.      Trusts

a.       Better for large amounts of money

b.      A trust can postpone possession until the donor things the child is competent to manage the property

5.      Variations on transfers to minors

a.       Representative payee – can accept SSA benefits on behalf of a minor or an incompetent

b.      Facility of payment clause – if a testator wants to make a cash bequest to a beneficiary who is now a minor, and the testator does not want to create a custodianship or trust, the will can provide that any cash bequest to a minor beneficiary may be distributed to the beneficiaries parents or those having custody of the child

c.       § UPC 5-104 – small sums may be paid without court approval $5000, UTMA $10,000

Section C. Bars to Succession

1. Homicide

a.       In re Estate of Mahoney

Issue: Whether a widow convicted of manslaughter in connection with the death of her husband may inherit from his estate.

Holding: Decision must be referred to chancery court which can make decisions concerning equity.

Rule: There are no statutes in the state of VT determining what should happen to an estate to be inherited by the murderer of the deceased. Other states use constructive trusts to keep the law of decent and inheritance in place. Other states do not allow the felonious to profit from their crimes.

Reasoning: Probate court does not have the right to create a constructive trust or to deny inheritance to the murderous wife. The chancery court can make decisions concerning equity.

Dicta: This court believes that a line should be drawn between voluntary and involuntary manslaughter. Those who commit voluntary manslaughter should not inherit.

b.      UPC § 2-803 – Treats the killer as having disclaimed the property

c.       § 2-1106 – the killer is treated as having predeceased the decedent

d.      Riggs v. Palmer: Slayer rule, murderer cannot take estate of victim

e.       Primerica Life Ins. Co.; Estate of Mueller: killer treated as predeceased

f.       Heinzman: killer’s heirs cannot inherit

g.      Majority view: § 2-803: a criminal conviction of a felonious and intentional killing is conclusive. Acquittal however is not dispositive of the acquitted individual’s status as a slayer. The court must determine whether a preponderance of evidence standard the individual would be found criminally accountable for the killing. Killer who commits suicide may be barred under this section.

h.      In a few states spouses who abandon their spouse are barred from inheriting and also in some states parents who do not support their child

i.        The Chinese system. Rewards for good behavior and support of the decedent. Requires much administration. Penalizes unworthy heirs. Includes blended and extended family emebers.

j.        CA Probate Code 2004 - § 259 Abuse of Elder or Dependent Adult Decedent

§  (a) Any person shall be deemed to have predeceased a decedent…where all of the following apply

ú  (1) It has been proven by clear and convincing evidence that the person is liable for physical abuse, neglect, or fiduciary abuse of the decedent, who was an elder or dependent adult

ú  (2) The person is found to have acted in bad faith

ú  (3) The person has been found to have been reckless, oppressive, fraudulent, or malicious in the commission of any of these acts upon the decedent.

ú  (4) The decedent, at the time those acts occurred and thereafter until the time of his or her death, has been found to have been substantially unable to manage his or her financial resources or to resist fraud or undue influence

2. Disclaimer – when an heir refuses to take property, the property is treated as if title passed to the heir and then from the heir to the next intestate successor because it can reduce taxes or be taken by creditors

a.       If a person dies testate – the devisee can refuse to accept the devise; thereby preventing title from passing to the devisee. All gifts require acceptance.

·         Tax consequences

o   No tax consequences if testamentary gift is not accepted under Brown

§  If the heir has a lower tax rate than the “predeceased heir”, the inheritance will be taxed at a lower rate

§  Most states require a disclaimer within 9 months as a reaction to IRC § 2518 which requires a disclaimer to waive gift tax, point of contention for accepting the UPC, see below

§  UPC § 2-1101 through § 2-1107 does not have a time limit

o   Disclaimant is treated as having predeceased the decedent §UPC 2-1105 and § 2-1106

o   Lawyer can be charged with malpractice for not telling clients about disclaimer because much money can be saved.

o   Passes only to the descendants of the disclaimant who survive the time of distribution

·         Avoiding Creditors

o   Disclaimer relates back to the date of the decedents death

o   Drye- Insolvent man disclaims property then puts inheritance in a trust for daughter of which he and his wife and the beneficiaries and which he decides distribution. This power to channel the estate’s assets warrants the conclusion that Drye held the property or a right to the property subject to the government’s liens.

·         Troy v. Hart (estranged sister persuades brother to disclaim)

o   Issue: Did the court err in holding that the Medicaid recipient could disclaim his inheritance?

o   Holding: No, the Medicaid recipient cannot disclaim his inheritance.

o   Rule: Medicaid requires that recipient’s notify them of inheritance within 10 days.

o   Reasoning: Medicaid is a means-tested program. It is against public policy that someone who is financially self-sufficient may voluntarily relinquish a windfall. To permit disclaimed property to pass to transferees free and clear of any obligation would be a violation of public policy. They suggested that the sisters had to pay that portion to Medicaid.

·         Spending down of assets can disqualify a person from Medicaid

·         It is a crime to conduct Medicaid Planning, congress repealed the sections of the law targeting the elderly


Chapter 3 Wills: Capacity and Contests

Section A. Mental Capacity

1. The Test of Mental Capacity Restatement of Property: Wills and Other Donative Transfers § 8.1 – Capacity to make a will is governed by a different legal test and requires less mental ability than to manage one’s investments, make a contract or to make a gift, but less capacity than is required for marriage.

                                         i.    Age 18 or older

                                        ii.    Capable of knowing and understanding in a general way

1.   The nature and extent of his or her property

2.   The natural objects of his or her bounty

3.   The disposition that he or she is making of his or her property

4.   Capable of relating these elements together in an orderly desire regarding the disposition of property

b.   In re Estate of Wright

Procedural History: Deceased’s will was denied on the grounds of testamentary incapacity.

Holding: The evidence is insufficient to sustain the judgment order.

Rule: The legal presumption is always in favor of sanity. Werstler

Reasoning: The witnesses of the will gave their approval to the will. Witness has a duty before they give their stamp to ensure that the testator is of sound mind. Seems that man was cared for by others and not his family.

c.   For a lawyer to draft a will for an incompetent is a breach of ethics

d.   § 8.1: To make an irrevocable lifetime gift, not only must one meet all the elements for making a will, but one “must also be capable of understanding the effect that the gift may have on the future financial security of the donor and of anyone who may be dependent on the donor.

2. Why Require Mental Capacity?

a.   Must represent the testator’s true desires

b.   Mental incompetent person is not defined as a person

c.   Protect the descendant’s family, provides support and care for the aged

d.   Legitimacy cannot exist unless decisions are reasoned

e.   Assures a sane person that if they later become insane their will will stand against new “insane” will.

f.    Protects society at large from irrational acts.

g.   Protects senile or incompetent from exploitation by cunning persons.

3. Insane Delusion – a false concept of reality to which the testator adheres against all evidence and reason to the contrary; if there is any factual basis for the delusion, the testator is not insane

a.   Only the part of the will caused by the insane delusion fails, can fail in part, whole or not at all depending on the affect of the delusions

b.   Estate of Raney, father penned a will disinheriting his children while in jail for a drunken driving charge and it was upheld.

c.   In re Strittmater

d.   Honigman

Issue: Did the testator have the mental capacity required to make a will?

Holding: Insane delusion

Procedural History: Jury decided he did not have the mental capacity to make a will. Appellate division reversed.

Reasoning: Court said that testator suffered from an unwarranted and insane delusion that his wife was cheating on him after illness and surgery. His obsession was established by a preponderance of the evidence. He told many people unreasonable stories about his wife’s purported cheating. Although his reasoning was not sound, his unsound reasoning is not enough to invalidate his will.

e.   An insane delusion is a belief not susceptible to correction by presenting the testator with evidence indicating the falsity of the belief. A mistake is susceptible to correction if the testator is told the truth.

f.    If the T is mistaken not under insane delusion, the will is entitled to probate. (i.e. believes son is dead, but he is alive, leaves all to daughter)

g.   Living Probate Allowed by Some States, Langbein

a.   Begins with the problem of the will contest alleging testamentary incapacity.

                                                                 i.    In civil law countries, children as well as the spouse have a force share entitlement in the estate of a parent.

                                                                ii.    No jury in civil law countries

                                                              iii.    P does not have to pay attorney fees in the US

                                                              iv.    Civil law systems provide for an authenticated will officiated by a quasi-official called the notary

·          

·         Estate of Fritschi (SC of CA, 1963) (LACK OF CAPACITY AND UNDUE INF.)

·         F: Dr. Fritschi planned to marry his receptionist Mary Tweed once his divorce was final. In the meantime, he was admitted to the hospital. Tweed began to exert control over his personal affairs,  and while in the hospital, Dr. Fritschi executed the contested will which would benefit Mary Tweed to the detriment of Dr. Fritschi's children. Will opponents also assert that Dr. Fritschi's use of certain tranquilizers deemed him to lack testamentary capacity.

·         Issue 1: is the will invalid because the testator lacked testamentary capacity at the time of execution?  because undue influence was exerted upon him by Marie Tweed?

·         H/R:

·         The evidence provided by the plaintiffs only equated to minor irrational displays, physical weakness of a man afflicted with a fatal illness, and certain personality quirks. No testimony provided reached the requisite showing of an insane delusion operating directly on the testamentary act. Plaintiffs also offered no testimony to prove that the drugs (mere tranquilizers... not morphine!) would or could cause lasting incompetence as opposed to a temporary reaction. Additionally, Dr. Fritschi's tolerance to such drugs argues against a finding of lack of capacity. Altogether, the plaintiff's evidence of incapacity amounts to mere speculation.

·         *A testator is presumed sane and competent and the burden rests on the contestant to overcome this presumption.

·         Issue 2: Undue influence. Does the evidence sufficiently demonstrate that Marie tweed actively participated in the procuring of the will?

·         Rules:

·         Such influence must "destroy T's free agency and substitute for his own another person's will." (Estate of Arnold)

·         "Evidence must be produced that pressure was brought to bear directly upon the testamentary act... the influence must amount to coercion destroying free agency on the part of the testator." (Estate of welch).

·         "Circumstances must be inconsistent with voluntary action on the part of the testator." (Ibid)

·         -Opportunity coupled with a motive/interest is not in itself sufficient.

·         -mere general influence is not enough (contestants must show that the influence was brought directly to bear upon the testamentary act).

·         H/R: No. Contestants provided no evidence that Marie Tweed participated in the procurement of the will, or suggested any of its provisions. Contestants only showed mere incidental activity in the execution (her pen used to sign it).

·         Note: This appellate court over turned the trial court/jury. This case seems to be a good example of a jury being much more likely to find that the testator lacked capacity so the money ends up with whom they think it should. Although the law of testamentary capacity is extremely soft/fact sensitive, appellate judges must find evidence that contestants met their burden of proof (beyond mere speculation!).

·         Estate of Risch (2006, pg 9) (LACK OF CAPCITY)

·         F: decdent's cousin seeks to admit to probate one of four wills the decedent executed in 2002 which name him as sole beneficiary. Decedent has a previous 1987 will that purports to divide Risch's estate equally among his 3 stepchildren. Risch was subject to a conservatorship from 2001 until his death in 2003, but as conservatee would still have the right to make a new will as long as he remained "mentally competent" to do so.

·         I: Did Risch lack the requisite mental competence to make a valid will on the dates of execution?

·         Rules:

·         Contestant must prove lack of testamentary capacity by a preponderance of the evidence.

·         -once it is shown that testamentary incompetency exists and that it is caused by a mental disorder of a general and continuous nature, the inference is reasonable that the incompetency continues to exist. (especially strong if condition becomes progressively worse). (Estate of Clegg).

·         -mental competency may hinge on the testimony of only ONE expert witness.

·         H/R: The evidence is clear and convincing that Risch was not competent to make the 2002 wills, therefore they are invalid to revoke the 1987 will. There was expert testimony that Risch was suffering from 'moderately severe dementia" by mid-january 2002. A finding that a testator lacked capacity is more likely with conditions such as dementia that inevitably become progressively worse. The 1987 will should be admitted to probate.

·         Estate of Callahan (1967, pg 13)

·         F: decedent executed an unusual holographic will dated july 7, 1950 consisting of 3 pieces of paper fastened together. A handwriting expert confirmed all 3 sheets were in decdent's handwriting.

·         I: was the trial court's issuing of the proponents non-suit proper?

·         H/R:  No... the evidence of incapacity was sufficient to enter to a jury. The writing and integration occurred after decedent had started to suffer from a condition of senile dementia to the extent that she did not understand the nature of her property or her relationship to persons with claims on her bounty.

·         **this case doesn't seem to bring any new rules/concerns to the table.

·         Estate of Horowitz

·         Court of Appeal of California

·         Facts: Decedent was married to the Sr. Horowitz. She had no children. Her husband died in 1959 leaving 2/3 of his estate to her and 1/3 to his son. She left some personal property to David, his wife and daughter. She left her real property to a nephew and left the remaining estate to her five nephews. Decedent left a handwritten document in her bedside drawer that stated it was her last will and testament. The document was written in several colors of ink, contained interlineatings, corrections and writings in the margin. The document was not dated and decedent’s signature did not appear at the end of the document. The document left certain items to appellant stepson, but left the remainder of real property to respondent relatives.

·         Procedural History: The trial court admitted the holographic document to probate, noting that there was no question the document was in decedent’s handwriting and that I contained decedent’s signature. The court noted that the document made a complete disposition of property and that decedent would not have chosen to die intestate. Appellant stepson challenged an order from the Superior Court of San Diego County, which admitted a holographic instrument into probate upon request by petitioner relatives after the trial court found the holographic document to be decedent’s will.

·         Issue: Did the T lack capacity and if not does that lack of a date on the holographic make it invalid?

·         Holding: No and no. The document was decedent’s valid will.

·         Rules:

·         Wills § 3 – Before an instrument may be admitted to probate as a will, it must appear that it was executed with testamentary intent. The basic test is not the testator’s realization that the testator was making a will, but whether the testator intended by the particular instrument offered for probate to created a revocable disposition of the testator’s property to take effect only on the testator’s death. No particular words are necessary to show testamentary intent, but it must satisfactorily appear from the proffered instrument that the testator intended by the very paper itself to make a disposition of the testator’s property after death.

·         Wills: § 85 – Regardless of the language of an instrument, extrinsic evidence may be introduced to show it was not  intended by the testator to be effective as a will. A reviewing court is bound by the trial court’s interpretation of a written instrument where it turns on the credibility of conflicting evidence. However, this is not the case where no extrinsic evidence was presented in the trial court or where the evidence acted on there was  no in conflict.

·         Wills § 21- A signature need not be located at the end of an instrument, but may appear in another part of it, to qualify as a properly executed holographic will, providing the testator wrote his or her name there with the intention of authenticating or executing the instrument as his or her will. The required intention must appear in the face of the instrument, and parol evidence is not admissible to show that a signature found elsewhere than at the end is a signature of execution. Completeness of the testamentary declaration can be sufficient evidence of the instrument’s signing.

·         Wills §30 – The rule that it is a judicial function to interpret a written instrument unless the interpretation turns on the credibility of extrinsic evidence is applicable to the interpretation of wills.

·         Wills § 85 – Where the issue turns merely on interpretation, the appellate court must independently interpret whether the will was signed.

·         Wills § 21 – Statutes dispensing with formalities previously required in the execution of wills may be applied retrospectively where the effect is to validate previously executed wills of testators who were alive when the statutes were adopted. This practice is not based on arcane rules relating to the ambulatory nature of wills, but rather on the strong policy of the law to uphold, when possible, the validity of wills.

·         Reasoning: Although not in keeping with the T’s usual care with personal matters. The document was signed, albeit at the beginning, it stated that it was her will and it was a complete device for disposition of property.

Section B. Undue Influence

a. Undue influence must involve coercion. Lord Hannen.

b. Undue influence may occur where there is a confidential relationship between the parties or where there is no such relationship.

c. Proof may be wholly inferential and circumstantial.

d. May be H or third party.

e. Elements of undue influence

1.   The testator was susceptible to undue influence

2.   The influencer has disposition or motive to exercise undue influence

3.   The influencer had opportunity to exercise undue influence

4.   The disposition is the result of the influence.

f. Estate of Lakatosh (burden shifting presumtion) – the burden of proof may be shifted as to require the proponent to disprove undue influence, the contestant must prove by clear and convincing evidence that

1.   There was a confidential relationship

2.   That the person enjoying such relationship received the bulk of the estate

3.   The decedent’s intellect was weakened

g. Restatement Third of Property: Wills and Other Donative Transfers (2003) §8.3 Undue Influence, Duress or Fraud

    (a) A donative transfer is invalid to the extent that it was procured by undue influence, duress or fraud.

    (b) A donative transfer is procured by undue influence if the wrongdoer exerted such influence over the donor that it overcame the donor’s freewill and caused the donor to make a donative transfer that the donor would not have otherwise made.

h. Comment H to §8.3 Suspicious Circumstances – The existence of a confidential relationship is not sufficient to raise a presumption of undue influence. There must also be suspicious circumstances surrounding the preparation, execution, or formulation of the donative transfers. Suspicious circumstances raise an inference of an abuse of the confidential relationship between the alleged wrongdoer and the donor. All relevant factors to be considered.

1.   The extent to which the donor was in a weakened condition, physically, mentally or both and therefore susceptible to undue influence;

2.   The extent to which the alleged wrongdoer participated in the preparations or procurement of the will or will substitute;

3.   Whether the donor received independent advice from an attorney or from other competent and disinterested advisors in preparing the will or will substitute;

4.   Whether the will or will substitute was prepared in secrecy or in haste;

5.   Whether the donor’s attitude toward others had changed by reason of his or her relationship with the alleged wrongdoer;

6.   Whether there is a decided discrepancy between a new and previous wills or will substitutes of the donor

7.   Whether there was a continuity of purpose running through former wills or wills substitutes indicating a settled intent in the disposition of his or her property; and

8.   Whether the disposition of the property is such that a reasonable person would regard it as unnatural, unjust or unfair, for example whether the disposition abruptly and without apparent reason disinherited a faithful and deserving family member.

i. The proponent of a will has the burden of persuasion concerning the wills validity (easy).

j. The person contesting the will then has the burden of proof considering undue influence ( in some jurisdictions they must prove that the influencer procured the will.)

k. Then the influencer must prove there was no undue influence by clear, satisfactory, and convincing evidence that the grantee acted in good faith throughout the transaction and the grantor acted freely, intelligently and voluntarily.

l. Parts of wills can be struck and others left in place if they were not effected by the undue influence.

m. Lipper v. Weslow

Procedural History: The TC entered judgment on the verdict setting aside the will.

Issue:  Is there insufficient evident to support the finding that the will was procured by undue influence?

Holding: No probative force to support eh verdict of the jury.

Rule: Was there control exercised over the mind of the testator as to overcome her free agency and free will and to substitute the will of another so as to cause the testatrix to do what she would not otherwise have done but for such control.

Reasoning: There was no substitution of a plan of testamentary disposition by another as the will of the testatrix. She talked to disinterested witnesses about her will and was physically active.

n. No-Contest Clauses – provide that a beneficiary who contests the will shall take nothing or a token amount, in lieu of the provisions made for the beneficiary in the will.

1.      The majority of courts enforce a no-contest clause unless there is probable cause for the contest.

2.      RS Prop § 8.5 and UPC §§ 2-517 and 3-905 – the majority of courts enforce a no contest clause unless there is probable cause for a contest

3.      Minority rule – enforced unless the contestant alleged forgery or subsequent revocation by a later will or codicil or the beneficiary is contesting a provision benefiting the drafter of the will or any witness.

4.      CA – procedure for declaratory judgment that a particular suit will thwart the intention of the testator and trigger a no contest clause which will be strictly enforced.

o   Nairne v. Humblet (2002, pg 5)

o   F: Son/Beneficiary argues that the settlors (his mother and her husband) orally agreed to give certain property to him, therefore such property should not be included in the trust. The settlors will contains a broadly written No-contest clause which includes forfeiture in the event of a contest.

o   I: May a beneficiary's claim to enforce an oral contract be considered separate from the terms of the will, and hence not violate the No Contest Clause? 

o   H/R: Overturning the trial court, the proposed complaint would violate the No-Contest Clause in the trust. The beneficiary's claim directly attacked a provision of the trust, because the subject property was specified in the trust. If successful, his claim would frustrate settlor's intent.

o   Policy concerns

o   -NCCs are favored in CA for discouraging litigation and giving effect to purposes expressed by the testator.

o    

o   What a court must look at:

o   A court must narrowly construe a NCC, and may not extend it beyond what was plainly the testator's intent (because resulting forfeiture is so harsh!).

o   FACTORS

o   -the particular language of the NCC

o   -what was the purpose of the instrument.

o   -whether the challenge, if successful, would result in thwarting the testator's intent.

o   -whether the testamentary instrument specifically characterizes the property (i.e. as separate versus community property)

o   Example of discrepancy regarding specificity of instrument:

o   *In Estate of Pittman and Burch v. George the beneficiary's attempt to re-characterize property as community or separate involved property that was expressly included, and characterized in the trust instrument as community property or separate property.

o   *In contrast, if the testamentary instrument broadly refers to "my property" or "property which I have a right to dispose," a challenge as to whether the property should be characterized as community property or separate property would NOT violate a no contest clause.

o   Estate of Strader (2003, pg 10)

o   F: unforeseeable litigation involving decedent's conservatorship occurs. Upon completion, the court decides to distribute income from the litigation only amongst those beneficiaries who supported the action, thus altering the residue. Challenge is brought by a beneficiary who voluntarily elected to not take part in the litigation.

o   I: did the final accounting of the will violate the no contest clause?

o   H/R: No. The will (and for that matter the testator) did not contemplate the existence of such proceeds from litigation, so there is no unequivocally expressed intent that anyone be disinherited due to altered distribution resulting from unforeseeable litigation proceeds that were not supported by all beneficiaries. The court analogizes the situation as similar to an attempt to characterize property not expressly mentioned in the will. (The NCC is only violated when the instrument already sets forth the manner in which the property is the be characterized).

o   Estate of Pittman (1998)

o   F: Children filed an action attempting to re-characterize trust property as community property. The re-characterization would eventually be advantageous to their interest. The trust contained an extremely broad no-contest clause. Provisions of the trust declared that all property listed on the particular schedules was of a particular type, with each item meticulously listed and characterized.

o   H/R: the children's petition to re-characterize trust property as community property violated the no-contest clause. By challenging the characterization of the property, the children contested the provisions of the trust. The trust as a whole evinced a clear and decided intent that each piece of property in the trust estate pass in precisely the manner established in the trust.

o   *The NCC in this case is not merely denouncing a challenge of the validity of the will in its entirely. The Pittman trust stretches beyond and prohibits anyone from seeking to merely set aside any of the provisions of the trust.

o   *clear Pittman's PURPOSE was to expansively prohibit any attempt to set aside any provision of the trust.

o   Estate of Coplan (2005):

o   F: Will provides "no personal representatives shall receive compensation for service as executor." The personal representative/beneficiary petitioned the court solely as a representative, requesting that the court relieve her from the effect of the non-compensation provision in light of her extensive duties (she defended the estate in litigation that lasted several years).

o   H/R: the administrator/beneficiary's petition for relief (which was eventually withdrawn) from the non-compensation clause did not violate the NCC. Although withdrawn, her petition arguably qualified under an exception to the NCC which applied to "proceedings relating to instructions or accountings."

o   Key Facts, and key statements made be the court:

o   -the fact that upholding a petition would reduce the residue does not necessarily mean it is a will contest.

o   -It is not apparent that the testator intended to disinherit a personal representative who requested compensation. The personal representative/beneficiary was actually appointed by the court. The testator's first two choices were not beneficiaries.

o   -the decedent did not anticipate the extent of services necessary to probate the will.

o   -the executor/beneficiary was never compensated, and defended the estate by eventually withdrawing her request.

·         Bequests to Attorneys

·         A presumption of undue influence arises when an attorney drafter received a legacy except when the attorney is related to the testator.

·         Can be rebutted only by clear and convincing evidence provided by the attorney.

·         In NY the surrogate must investigate any bequest to the attorney who drafted the will. Must submit an affidavit concerning the facts and circumstances of the gift.

·         Model Rules of professional conduct – A lawyer shall not prepare an instrument giving the lawyer or a person related to the lawyer as parent, child, sibling or spouse any substantial gift from a client, including a testamentary gift, except where the client is related to the donee.

·         A lawyer may accept a gift if the transaction meets general standards of fairness.

·         In Re Will of Moses

Procedural History: Sister claimed undue influence. The chancellor found undue influence and denied probate. B appealed.

Issue: Was T under undue influence when she drafted her will.

Holding: There was undue influence over T when she drafted the will.

Rule:

Reasoning: B was in charge of her affairs as attorney, but an independent attorney drafted the will. Such a relationship gave rise to a presumption of undue influence which could be overcome by evidence that, in making the 1964 will, Mrs. Moses had acted upon the independent advice of counsel of one entirely devoted to her interest. Holland’s sexual relationship with T speaks to his influence over her. The independent attorney did not give her advice or counsel and just wrote down her will. This did not meet the burden nor overcome the presumption.

Dissent: She acted independently. The court ruled based on her non traditional lifestyle.

5.      RE Kaufmann’s Will: same-sex testator, consider’s the undue influence of the same sex beneficiary

6.      Seward Johnson’s Estate: Huge estate, contested by children, settled out of court. Everyone ended up with a ton of money including attorneys.

o   Estate of Clegg (1978) (UNDUE INFLUENCE/CAPACITY)

o   F: At the age of 83, decedent  entered a convalescent hospital after her doctor diagnosed her with suffering from senile dementia and reduced mental capacity. Although decedent had two brothers and a sister living in Minnesota, her will purported to leave all of her property to convalescent home. Administrator of the home was named conservator  just prior to execution of the will, and was named executor by the will itself.

o   Test for competency: Whether decedent had sufficient mental capacity to

o   1) understand the nature of her act;

o   2) understand and recollect the nature and situation of her property; AND

o   3) remember and understand her relations to persons who have claims upon her bounty, and whose interest are affected by the will. (Estate of Fritschi).

o   UI Elements for Presumption of UI (once established by preponderance of the evidence, the burden shifts to the proponent to prove that the will was not induced by UI.

o   1) existence of a confidential or fiduciary relationship between the testator and the person alleged to have exerted the UI

o   2) active participation by such person in preparation or execution of the will (may be established by circumstantial evidence); AND

o   3) an undue benefit to such person or another person under the will thus procured.

o   H/R:   Mrs. Clegg's physician testified that she suffered from a 'general and continuous' mental disorder prior to execution of the will, therefore it may be inferred that she lacked capacity upon execution. It is not necessary to rule on the issue, but there is also substantial evidence of undue influence: there was a confidential relationship between the deceased and the beneficiary, and the representatives of the home had actively participated in the preparation of the will. Also, decedent's relatives were not informed about either the conservatorship or the attempted probate of the purported will which claimed decedent had no brothers or sisters. AFFIRMED!

Section C. Fraud

1.      Fraud occurs where the testator is deceived by a misrepresentation and does that which the testator would not have done had the misrepresentation not been made. Defrauder must have the intent to:

a.     Deceive the testator

b.     Purpose of influencing the testamentary disposition

2.      The portion of the will influenced by fraud is invalid, but only if the testator would not have left the inheritance or made the bequest had the testator know the true facts

a.     Will will be probated and then a court with equity powers can impose a constructive trust to remedy the unjust enrichment caused by fraud

3.      Types of fraud

a.     Fraud in the inducement - occurs when a person misrepresents facts, thereby causing the testator to execute a will to include particular provisions in the wrongdoer’s favor, or to refrain from executing or revoking a will (making testator think you are married, you are not)

                                           i.      Puckett v. Krida – evidence of a confidential relationship, they are nurses and Krida had power of attorney. By limiting information available to the deceased and by concealing their acts from the critical examination of those whom the deceased had previously known and trusted, the d’s isolated the deceased and controlled access to her.

b.     Fraud in the execution – occurs when a person misrepresents the character or contents of the instrument signed by the testator, which does not in fact carry out the testator’s intent. (having testator sign will that is not the one she created)

Section D. Duress

1.   RS 3rd Property: Wills and Other Donative Transfers § 8 When undue influence becomes overtly coercive, it becomes duress. A donative transfer is procured by duress if the wrongdoer threatened to perform or did perform a wrongful act that coerced the donor into making a donative transfer that the donor would not otherwise have made. The law invalidates transfers compelled by duress.

2.   Latham v. Father Divine

Procedural History: The T’s cousins claimed that she told them she was going to change her will and leave them money. They also claimed that a surgery was conducted by someone related to the church. They claimed she was murdered during the surgery. They claimed the B’s did this so that she could not sign the new will.

Rule: Where a devisee or legatee under a will already executed prevents the testator by fraud or duress or undue influence from revoking the will and executing a new will in favor of another or from making a codicil, so that the testator dies leaving the original will in force, the devisee or legatee holds the property thus acquired upon a constructive trust for the intended devisee or legatee.

Reasoning: A constructive trust will be erected whenever necessary to satisfy the demands of justice. Beatty. Heirs should not be able to profit from fraud.

3.   A constructive trust is sometimes said to be a “fraud rectifying” trust. But actually it can be in cases of unjust enrichment. It is an equitable remedy. Once converted into a constructive trustee, the holder of the property must transfer it to the constructive beneficiary.

4.   Pope v. Garret: Even if innocent heirs would have inherited by way of the will tainted by duress, the cannot inherit if they would not have inherited under untainted conditions.

Section E. Tortious Interference With Expectancy

1. Restatement (Second) of Torts § 774B (1979) – intentional interference with an expected inheritance or gift as a valid cause of action

a.       P must prove that the interference involved conduct tortious in itself, such as fraud, duress or undue influence

b.      Seeks to recover tort damages from a third party for tortious interference

                                           i.      Statute of limitations: starts running on the action at the time the p discovered or should have discovered  the fraud or undue influence

                                         ii.      If the p contests the will and loses, ordinarily the p is barred by the principal of res judicata from suing later in tort

                                       iii.      No contest clause does not apply because it is not a contest

                                       iv.      The theory cannot be used when the challenge is based on the testator’s mental incapacity

                                         v.      Tort damages may be recovered unlike seeking to prevent the probate of a will

                                       vi.      The probate exception – a federal court may not probate a will or entertain a suit that could undermine or contradict the judgment of a state probate court. The federal courts may entertain a probate related suit if the parties could have litigated it in a state court of general jurisdiction.

c.       Marshall v. Marshall (Anna Nicole Smith)

                                           i.      Issue: Did Marshall’s son interfere with her inheritance?

                                         ii.      Rule:  Test for tortious interference: (1) the existence of an expectancy; (2) a reasonable certainty that the expectancy would have been realized but for the interference; (3) intentional interference with that expectancy; (4) tortious conduct involved with the interference (5) damages.

                                       iii.      Reasoning: The son destroyed, backdated and altered or prepared and present J Howard with false documents related to the draining of his assets.

       
Chapter 4

Wills: Formalities and Forms

Section A. Execution of Wills

1. Attested Wills

a. The Function of Formalities

        i. The court needs to be convinced that the statements of the transferor were deliberately intended to effectuate a transfer.

                (1) Finality of transfer

                (2) Deliberate transfer

        ii. Increased the reliability of the proof presented to the court   

        iii. Safeguard the testator at the time of the execution of the will against undue influence or other forms of imposition

        iv. Langerbein – Channeling function – it is easier to determine a person’s wishes at death if they are channeled into a will with standardized formalities. Makes judicial costs less and makes testator feel secure.

b. UPC Formalities

i. § 2-502 Execution: Witness Wills; Holographic Wills

        (a) Except as provided in subsection (b) and in Sections 2-503, 2-506 and 2-513, will must be:

                (1) in writing:

                (2) Signed by the testator or in the testator’s name by some other individual in the testator’s conscious presence and by the testator’s direction; and

                (3) signed by at least two individuals, each of whom signed with a reasonable time after he [or she] witnessed either the signing of the will as described in paragraph (2) or the testator’s acknowledgement of that signature or acknowledgement of the will.

        (b) A will that does not comply with subsection (a) is valid as a holographic will, whether or not witnessed, if the signature and material portions of the document are in the testator’s handwriting.

        (c) Intent that the document constitutes the testator’s will can be established by extrinsic evidence, including, for holographic wills, portions of the document that are not in the testator’s handwriting.

ii. In Re Groffman, High Court of Justice, England – Two or more witnesses must be present at the same time.

iii. Stevens v. Casdorph

§  Issue: Is a will valid if a witness has not seen the testator sign the will?

§  Holding: The will is not valid if the witness did not see the testator sign.

§  Rule: W. Va. Code § 41-1-3 : No will shall be valid unless it be in writing and signed by the testator or by some other person in his presence an by his direction in such manner as to make it manifest that the name is intended as a signature; and moreover unless it be wholly in the handwriting of the testator. The signature shall be made or the will acknowledge by him in the presence of at least two competent witness present at the same time and such witness shall subscribe the will in the presence of the testator and of each other but no form of attestation shall be necessary.

§  Reasoning: The mere intent by a testator to execute a written will is insufficient. The actual executing of a written will must also comply with the dictates of the W.Va. Code. It would be OK if they didn’t sign in front of each other if they acknowledged that it was their signature on the will.

§  Dissent: Thinks that the approach of the court was technocratic. There was no claim as to incapacity, fraud or undue influence. Technical issue should not stand in the way of testator intent.

b. Presence – Under the line of sight test the testator does not actually have to see the witnesses sign but must be able to see them were the testator to look.

c. Conscious presence test – the witness is in the presence of the testator if the testor through sight hearing or general consciousness of events comprehends that the witness is in the act of signing.

d. UPC 2-502. –Witnesses do not have to sign in the testator’s presence.

            i. In Re Websters estate – man could not see teller sign with pen. Will not valid.

            ii. Re Colling – must complete entire signing in front of the witness

            iii. Wheat – testator must sign first

            iv. Taylor v. Holt Electronic signature can be used if intentionally included by testator

v. Line can be inserted anywhere on will if in testator’s handwriting if inserted in time before the signature

vi. Videotaped or electronic wills – ok if there is clearn and convincing evidence that it was intended to be a will

vii. Attestation clause – An attestation clause recites that the will was duly executed. Makes a prima facie case that the will was duly executed. Professional malpractice not to use one.

viii. Delayed attestation – witness must attest before the testator’s death. Witness must sign within a reasonable amount of time.

iix. Notarization – Friedmen – notary can serve as second witness

e. Estate of Parsons

§  Issue: Is a subscribing witness to a will who is named in the will as a beneficiary disinherited within the meaning of probate code § 51 by filing a disclaimer of her interest after the testatrix death?

§  Holding: Gifts to the witnesses were void.

§  Rule: §51 specifies that will must be signed by two disinterested witnesses.

§  Reasoning: Should not be verified by people who would have a financial motive to use duress, undue influence or fraud against the testator. This rule upsets the testator’s intent, but governs in this case. Prevents fraud. Legislature has spoken.

e2. Purging statute – purges the witness only of the benefit the witness receives that exceeds the benefit the witness would have received if the will had not been executed.

f. Purging statutes - § 2-505 footnote 7 re Parsons – an interested witness does not forfeit a devise under the will.

g. CA – rebuttable presumption that the devise to an interested witness was procured by duress, menace, fraud, or undue influence

a.   Recommended Method of Executing a Will

                          i.      The law of the decedent’s domicile at death determines the validity of the will insofar as it disposes of personal property.

                        ii.      The law of the state where real property is located determines the validity of a disposition of real property.

                      iii.      Most states have statutes recognizing as valid a will executed with the formalities required by the 1) state where the testator was domiciled at death 2) the state where the will was executed or 3) the state where the testator was domiciled when the will was executed

                      iv.      A lawyer should make the will so that it will work in all states

1.      If the will consists of more than one page, the pages are fastened together securely. The will specifies the exact number of pages of which it consists.

2.      The lawyer should be certain that the testator has read the will and understands its contents.

3.      The lawyer, the testator, three disinherited witnesses, and a notary public are brought together in a room from which everyone else is excluded. (If the lawyer is a notary, an additional notary is unnecessary.) The door to the room is closed. No one enters or leaves the room until the ceremony is finished.

4.      The lawyer asks the testator the following three questions:

a.       Is this your will?

b.      Have you read and do you understand it?

c.       Does it dispose of your property in accordance with your wishes?

                                                                                                                          i.      The testator should answer yes in a voice audible by everyone in the room.

5.      The lawyer should ask the testator if he/she asks _______, _________, and __________ to witness the signing of your will?

a.       The testator should answer yes in a voice audible to everyone in the room.

6.      The witnesses should be standing or sitting so that all can see the testator sign. The testator signs on the margins of each page of the will. This is done for identification and to prevent pages from being inserted. The testator then signs his or her name at the end of the will.

7.      One of the witnesses reads aloud the attestation clause, listing everything that happened during the signing of the will.

8.      Each witness then signs and writes his or her address next to the signature.

9.      A self-proving affidavit, typed at the end of the will, swearing before a notary public that the will has been duly executed, is then signed by the testator and the witnesses before the notary public, who in turn signs and attaches the required seal. This allows will to be probated if witnesses cannot be located.

a.       § UPC § 2-504 (1990) authorizes two kinds of self-proving affidavits. UPC § 2-504 (a) authorizes a combined attestation clause and self-proving affidavit, so that the testator and the witnesses sign their names only once; hence this is called a “one-step” self-proving affidavit

b.      More states permit § 2-504(b) a self-proving affidavit to be affixed to a will already signed and attested, an affidavit that must be signed by the testator and witnesses in front of a notary public after the testator has signed the will and the witnesses have signed the attestation clause

c.       UPC § 3-406 – if a will is self-proved, compliance with the signature requirements for execution is conclusively presumed

10.  Lawyer should make photocopies of the executed will.

11.  Check the will over after everyone has left.

12.  Write a short memo to the file that the will was executed using the usual practices.

13.  Place will in a vault or safe box.

14.  Place the copies in the client’s file.

15.  Send copy to client with a letter stating where the original will be stored, and a copy of earlier letters describing the estate scheme.

                                                ii.      Safeguarding a will

1.      It may be a good idea to keep the will

2.      Some courts discourage it because it seems like solicitation of business

3.      Some probate courts allow it to be kept in a registry.

a.       § UPC §2-515 provides for the deposit of a will in court for safekeeping

b.      In re Pavlinko’s Estate

Facts: Man signed his wife’s will and wife signed husband’s will. Lawyer and secretary signed as witnesses. Secretary did not speak the same language as testators.

Issue: Is the wife’s will signed by the husband valid?

Holding: No.

Rule:  The wills act of 1947 provides § 2 “Every will,… shall be in writing and shall be signed by the testator at the end thereof.

Reasoning: If this will became valid for equity, the door would be wide open to fraud.

Musmanno Dissent: The intent of the testator must be gathered from the four corners of his will. Both wanted the remaining property to go to Elias Martin. It was both of their intent. The fact that some of the clauses must be stricken, it does not mean that the residuary clause must be ignored.

c.       In re Snide

Facts: Husband signed wife’s will. Wife signed husband’s will. Wife survived husband. Two adult children and one minor child represented by a guardian ad litem survived the husband.

Procedural History: The elder children have signed waivers that the will be admitted to probate. TC admitted to probate. Appellate court reversed.

Issue: Can the wills be validated?

Holding: This court reverses. The names can be substituted to create valid wills.

Rule: Will should be signed by the testator.

Reasoning: The wills were the same except for the names. Both wills were executed within the statutory formality.

Dissent Jones: Would adhere to the precedent that in the US relief has not been granted in these rare cases.

d.      Curative Doctrines

                                                  i.      Traditional Rule – formalities required by the Wills Act must be complied with strictly, and almost any mistake in execution will invalidate the will.

                                                ii.      UPC § 2-503 – gives the court the power to dispense with formalities if there is clear and convincing evidence that the decedent intended the document to be his will. HARMLESS ERROR – Although a document or writing added upon a document was not executed in compliance with Section 2-502, the document or writing is treated as if it had been executed in compliance with that section if the proponent of the document or writing establishes by clear and convincing evidence that the decedent intended the document or writing to constitute (1) the decedent’s will, (ii) a partial or complete revocation of the will, (iii) an addition to or an alteration of the will or (iv) a partial or complete revival os his formerly revoked will or of a formerly revoked portion of the will.

e.       In re Will of Ranney

Facts: In this case, the lawyers omitted the attestation clause and used instead an affidavit designed for a two-step self-proving will. Thus, when the witnesses signed the document, they did not attest to the execution of a will but rather signed an affidavit swearing that they had previously signed their names as witnesses during a in the execution that never actually occurred.

Procedural History: Appellate division ruled that the witnesses signatures on the affidavit constituted signatures on the will.

Issue:  Is the will valid if witnesses sign only the attestation clause and not the will?

Holding: The will is valid if clear and convincing evidence establishes that it was the intent of the testator.

Rule: Substantial compliance. Although a document was not executed in compliance with § 2-502 the document is treated as if it had been executed in compliance with that section if the proponent of the document…establishes by clear and convincing evidence that the decedent intended the document to constitute the decedent’s will.

Reasoning: Most of the will was in compliance. It was a deliberate voluntary act of the testator.

f.       In re Estate of Hall

Facts: Husband and wife viewed a draft of a Joint will. Husband asked if the draft could act as the will until the new will was finalized. Attorney said yes and they signed it and he notarized it without any witnesses present. Wife tore up old will.

Procedural History: Daughter objected to the will. Probate court admitted the will to probate.

Issue: Is the signed and notarized will valid without the signatures of witnesses because the husband intended the will?

Holding: The husband intended the will, the court did not err in admitting it because it was valid.

Rule: If two people do not witness the will it can be executed under certain circumstances. If the proponent of the document establishes by clear and convincing evidence that the decedent intended the document to be the decedent’s will.

Reasoning: The husband intended the will to stand until the final will was drafted.

g.      Langbein –

                                                  i.      Substantial compliance – if there is clear and convincing evidence that the purposes of formalities—the evidentiary, cautionary, protective, and channeling functions—were served despite a defective execution, the will is admitted to probate. (meant to deal with minor errors).

                                                ii.      Dispensing power – provides for the probate of a document that was not properly executed if the court “is satisfied that there can be no reasonable doubt that the deceased intended the document to constitute his will.” This act excuses non compliance with the wills act. (L thought that this was better because it makes the court focus on the testator’s intent rather than near miss procedures.

h.      UPC 2-503 – courts are directed to look not at whether the purposes of formalities were served, but at whether the decedent intended the document or writing to constitute the decedent’s will.

i.        RS 3rd Property § 3.3 – A harmless error in executing a will may be excused if the proponent establishes by clear and convincing evidence that the decedent adopted the document as his or her will.

j.        Holographic wills are handwritten wills. They are not recognized in most states.

k.      Off hand statements made in letters are usually not accepted.

Kimmel’s Estate –

Facts: Decedent wrote an informal letter to his two sons stating that he wanted them to have his property. It was signed and dated and addressed to them.

Issue: Can an informal letter be testamentary in nature and is the signature to it a sufficient compliance with the Wills Act?

Holding: His intent to execute is apparent and the will is sufficient.

Rule: While the informal character of a paper is an element in determining whether or not it was intended to be testamentary, this becomes a matter of no moment when it appears thereby that the d’s purpose was to make a posthumous gift.

Reasoning: The words stating that if anything happens precondition the gift and strongly support the idea of testamentary intent.

l.        Eaton – Most cases on conditional wills are in accord. They presume the language of condition does not mean that the will is to be probated only if the stated event happens but is, instead merely a statement of the inducement for execution of the will which can be probated upon death from any cause.

m.    Elements of a valid holographic will:

                                                  i.      Handsigned by the testator

1.      In almost all states permitting holographs, a holograph may be signed at the end, at the beginning, or anywhere on the will, but if not signed at the end there may be doubt about whether the decedent intended his name to be a signature.

                                                ii.      In testator’s handwriting

1.      First generation statutes – entirely, written, signed and dated in the handwriting of the testator. Sometimes courts will completely invalidate, sometimes they only invalidate the printed part. Some require entire date January 8th, 1976.

2.      Second generation statutes (1969) UPC: material provisions – signature and material provisions of the will must be in the testator’s handwriting.

3.      Third generation – material portions and extrinsic evidence allowed – signature and material portions are in the testator’s handwriting

4.      Mulkins – important thing is that the testamentary part of the will be wholly written by the testator and of course signed by him. The printed words were not essential to the meaning of the handwritten words and this the holograph was upheld.

5.      Johnson – Will could not be admitted to probate on the ground that the printed words of the will were essential to establish testamentary intent and hence were material provisions.

6.      Muder – a testator who uses a preprinted form and in his own handwriting fills in blanks by designating his beneficiaries and appointing his estate among them and signs its has created a holographic will. Such handwritten provisions may draw testamentary context from both the printed and the handwritten language on the form.

7.      § 2-502 (c) – holographs may be written on a printed will form if the material portions of the document are handwritten

8.      Statutory form wills – Must be signed and attested in the same manner as any attested will. A large number fail in probate because they are improperly completed or executed.

n.      In re Estate of Kuralt

Facts: Two weeks before he died, Kuralt wrote a letter to Shannon (his long time mistress) assuring her that he would see to it that she would inherit his property in Montana. He provided financial support for her family and gifted her a house in Ireland during life. Holographic will bequeathing property to Shannon May 3, 1989. Formal will with no mention of Shannon on May 4th 1994. April 9th, 1997, Kuralt deeded his interest in the original 20-acre parcel with the cabin to Shannon. Kuralt asked Shannon to send him a buy-sell real estate form. He intended once again to provide the money for the sale. He died before the purchase could go through. He also wrote a letter saying he was ill and that he would have the lawyer come to the hospital to ensure that she inherited the property. He also sent checks worth 17,000.

Procedural History: The TC granted partial summary judgment for the Estate on May 26th, 1998. Shannon appealed from the District Court order which granted partial summary judgment to the Estate. This court reversed the DC and remanded the case for trial in order to resolve the disputed issues of material fact. The DC held that the June, 18th, 1997 letter was a valid holographic codicil to Kuralt’s formal will of May 4th, 1994 and accordingly entered judgment for Shannon.

Issue: Was the letter a holographic codicil?

Holding: The district court was correct when it concluded that the holographic will was a codicil.

Rule: There needed to be clear and convincing evidence that Kuralt inteded the letter to be testamentary.

Reasoning: Kuralt provided significant financial support for Shannon and her children and they had a family like relationship.

o.      Codicil – a testamentary instrument that amends a prior will; it does not replace it.

p.      Possible components to testamentary intent:

                                                  i.      Intent that a document be used as evidence after death,

                                                ii.      Intent that the document convey no present interest

                                              iii.      Intent that it be a will

                                              iv.      Intent that it not be a will substitute

                                                v.      Intent to execute a document

                                              vi.      Intent that I be final unless later revoked

                                            vii.      Intent that certain beneficiaries receive certain property

§  Estate of Lund

§  Court of Appeal of California

§  Facts: A gift of 10K was left to the respondent. A holographic codicil executed after the will left 20 K and a mink coat to the respondent. Subsequent codicils authorized that respondent receive funds for her passage home to Argentina. The entire estate was valued at 260K.

§  Procedural History: Appellant beneficiary challenged the order of the Superior Court of the City and County of San Francisco which sustained exceptions to the recommendations of a probate commissioner in an heirship proceeding. Appellant alleged that a gift of 20K to respondent legatee under decedent’s first codicil was intended as a substitution for the gift of 10K under the will. Probate commissioner said that the 20 K was substitutional. TC determined that respondent was entitled to all of the gifts.

§  Issue: Did the testator intend that the 20 K gift substitute for the 10 K gift, the mink coat and passage to Argentina?

§  Holding: No. Respondent was entitled to 30K, a mink coat and passage to Argentina.

§  Rule: Wills §254(6) A second gift of quantity is presumed to be cumulative unless there is extrinsic evidence of substitutional intent.

§  Reasoning: There was no language in any of the instruments to show substitutional intent.

·         Section B. Revocation of Wills

·         Revocation by Writing or Physical Act

·         A will is an ambulatory document – it is subject to modification or revocation by the testator during his or her lifetime.

·         § UPC § 1-205(56) defines a will to include a codicil and any testamentary instrument that merely appoints an executor or revokes or revises another will.

·         Methods of revocation:

·         Subsequent writing executed with testamentary formalities or

·         By a physical act such as destroying, obliterating or burning the will

·         No oral revocations are valid – would open door to fraud

·         § UPC 1990 2-507. Revocation by writing or act

·         (a) A will or any part thereof is revoked:

·         By executing a subsequent will that revokes the previous will or part expressly or by inconsistency; or

·         By performing a revocatory act on the will, if the testator performed the act wit the intent and for the purpose of revoking the will or part or if another individual performed the act in the testator’s conscious presence and by the testator’s direction. For the purposes of this paragraph, “revocatory act on the will” includes burning, tearing, canceling, obliterating or destroying the will or any part of it. A burning, tearing or canceling is a revocatory act on the will, whether or not the burn, tear or cancellation touched any of the words on the will.

·         Revocation by Inconsistency

·         A subsequent will wholly revokes the previous will by inconsistency if the testator intends the subsequent will to replace rather than supplement the previous will.

·         If the subsequent will does not make a complete disposition of the testator’s estate, it is not presumed to revoke the prior will but is viewed as a codicil.

·         Harrison v. Bird

Facts: Speer executed a will in Nov. 1989 in which she named Katherine Crapps Harrison as the main beneficiary of her estate. Original was given to Ms. Speer and a duplicate was retained by her attorney. March 4th 1991 Speer called attorney and advised him that she wanted to revoke her will. Attoney and secretary tore the will into pieces. And sent her a letter telling her that she had revoked her will. Ms. Speer died September 3rd, 1991.

Procedural History: The circuit court found that the estate of Speer should be administered as an intestate estate. Then upon admission of a will naming Harrison as beneficiary, the circuit court ruled 1) that Ms. Speer’s will was not lawfully revoked when it was destroyed by her attorney at her direction and with he consent, but not in her presence 2) that there could be no ratification of the destruction of Ms. Speer’s will, which was not accomplished 3) that based on the fact that the pieces of the destroyed will were delivered to Ms. Speer’s home, but were not found after her death, there arose a presumption that Ms. Speer thereafter revoked the will herself.

Issue: Was the evidence sufficient to rebut the presumption that Ms. Speer destroyed her will with the intent to revoke it?

Holding: No, the evidence was not sufficient.

Rule: If evidence establishes that Ms. Speer had possession of the will before her death, but the will is not found among her person effects after her death, a presumption arises that she destroyed the will. The burden of rebutting the presumption is on the proponent of the will.

Estate of Johnson

California Court of Appeal

Facts: A woman died leaving on will that had been executed in 1969 and another dated 1972. Although the earlier will intentionally disinherited the children of decedent’s predeceased daughter, they were not mentioned in the subsequent instrument. She named her sons as co-executors of her estate. She left 10 K to children’s hospital and the residue to her sons in ½ shares. She specifically made no provision for her daughter’s children in the first will. If either son predeceased her, his share was to go to his brother.

Procedural History: Appellant omitted heirs sought review of a decision by the Superior Court of Los Angeles County in favor of respondents heirs, in a dispute concerning interpretation of two testamentary instruments admitted to probate, which interpretation resulted in appellants’ not receiving a share in a distribution of an estate pursuant to Cal. Prob. § 1240. Regardless of the fact that the subsequent will was the last will in time, the court found that, pursuant to Cal. Prob. Code §72, the latter will did not revoke the prior will because the provisions of the latter will were not wholly inconsistent with the prior will and inconsistencies were minor.  Because both wills were titled “Will”, the court found that Cal.Prob.Code §101 required the wills be read together according to the decedent’s intentions, unless the two wills were wholly inconsistent, which they were not. Because the wills, read together indicated that the appellants were intentionally omitted from the wills, they were not pretermitted heirs under Cal.Prob.Code §90 and therefore appellants were not entitled to a share of the estate.

Issue: Did the later document titled “Will” revoke the earlier document titled “Will”?

Holding: A subsequent will with the same title as the prior will did not revoke the prior will because the wills were not wholly inconsistent.

Rule: Prob. Code §72: requires that a later will be wholly inconsistent with the terms of an earlier one in order to effect a revocation.

Prob. Code §101 Construe the two instruments together if there was no revocation.

Wills §31 : Governing factor is testator intent.

Wills: § 26 –Revocation, Alteration and Revival—Later Will or Codicil—Revocation by Complete Disposition of Property in Later Will. (1) Although the fact that a later will provides a complete disposition of the testator’s property can be an important consideration for purpose of determining whether it revokes an earlier will, such fact is not necessarily controlling. (2)A subsequent will has the power to revoke a prior will only by statute.

Wills: § 86: When the record on appeal contains no extrinsic evidence the trial court’s interpretation of a contested will stands unless it is not as tenable as the reviewing court’s interpretation.

Wills § 26 Although the dates of testamentary instruments can be a factor in determining which instrument controls the distribution of a T’s estate, the fact that a will is last in time is not in itself a sufficient basis for determining the distribution of an estate. The testator’s intent is the governing facts.

Wills § 30: Unless a subsequent will is wholly inconsistent with and revokes a prior will, two testamentary instruments are construed as on instrument.

Reasoning: Appellants were intentionally omitted from the will, they were not pretermitted heirs and they were not entitled to a share of the estate.

·         King – presumption of revocation rebutted by testimony that the testator had recently referred to the will being in effect

o    Travers - opportunity of disinherited heir to destroy will does not rebut presumption of revocation

o   Lonergan – presumption of revocation of lost will disinherited husband rebutted where husband lived in house with wife and the couple had been fighting before she dies.

o   Probate of Lost Wills – in the absence of statute, a will that is lost or is destroyed without the consent of the testator, or is destroyed with the consent of the testator but not in compliance with the revocation statute can be admitted into probate if its contents are proved. A lost will can be proved by a copy in the lawyer-drafter’s office or by other clear and convincing evidence.

o   In a few states, statutes prohibit the probate of a lost or destroyed will unless the will was “in existence” at the testator’s death (and destroyed thereafter) or was “fraudulently destroyed” during the testator’s life

o   Thompson v. Royall

o   Facts: 9/4/32 K signed a will, typewritten on five sheets of legal cap paper; the signature appeared on the last page duly attested by three subscribing witnesses. The executor was give possession of the will for safekeeping. A codicil typed on the top third of one sheet of paper dated 9/15/32 was signed by K in the presence of two subscribing witnesses. On 9/19/1932 she told her attorney to destroy both in front of Mr. Brittain. Instead of destroying them, she decided to retain them as memoranda. The attorney penned on the will that the will was null and void etc. and that Brittain was to hold it. A similar notation was made on the codicil except it said it was to be held by the attorney. Mrs. K died 10/2/1932 leaving numerous nieces and nephews some of whom were not mentioned in her will and an estate valued at approx $200,000.

o   Procedural History: Jury found that the documents date 9/4 and 9/15 were the last will and testament.

o   Issue: Had the will of Mrs. Kroll been revoked shortly before her death?

o   Holding: The will is valid. The cancellation was not valid under the statute.

o   Rule: No will or codicil, or any part thereof, shall be revoked, unless…by a subsequent will or codicil, or by some writing declaring an intention to revoke the same, and executed in the manner in which a will is required to be executed or by the testator or some person in his presence and by his direction, cutting, tearing, burning, obliterating, canceling, or destroying the same, or the signature thereto with the intent to revoke. * The cancellation message must come into contact with the writing.

o   Reasoning: Cancellation was not in her handwriting and was not attested to by witnesses.

o   UPC § 2-507 – a burning, tearing, or canceling is a revocatory act on the will, ‘whether or not the burn, tear, or cancellation is a revocatory act on the will, ‘whether or not the burn, tear or cancellation must however be written on the will, whether or not they touch the words of the will. They cannot be written on another document.

o   Partial revocation by physical act. In some states a will can only be revoked in part by a subsequent instrument. Avoids fraud.

o   Malloy – partial revocation by physical acts would not be permitted where the intent and effect of the change would result in a substantial enhancement of another bequest.

o   RS 3rd Property § 4.1 – any distinction between revocation of a complete devise and rearranging shares within a single devise or otherwise rewriting the terms of the will by deleting selected words. It is a classic example of a distinction without a difference. It is not supported by the language of the statutes specifically authorizing the revocation by act… The legislature not only granted broad approval of deleting words but of the natural consequence of doing so—giving effect to the will as if the deleted words were not present.

·         Dependent Relative Revocation and Revival

o   If the testator purports to revoke his will upon a mistaken assumption of law or fact, the revocation is ineffective if the testator would not have revoked his will had he known the truth.

o   La Croix v. Senecal

o   Facts: T executed a will leaving ½ to nephew and ½ to friend. She created a codicil replacing her nephew’s nickname with both his nickname and given name. The friend’s husband witnessed the codicil. Under the applicable purging statute, Senecal would be purged.

o   Issue:  Can the doctrine of dependent relative revocation be invoked to sustain a gift by will ,when such gift has been revoked in a codicil which substantially reaffirmed the gift, but was void as to it under § 6952 by reason of the interest of a subscribing witness.

o   Rule: Every devise or bequest given in any will or codicil to a subscribing witness, or to the husband or wife of such subscribing witness’s hall be void unless such will or codicil shall be legally attested without the signatures of such witnesses.

o   Reasoning: Her intent was to ensure that her nephew’s identity was clear, not to change her residuary estate.

o   Estate of Auburn

o   Facts: T had created a will in Milwaukee WI in 1955. She also executed a will in Kankakee, IL in 1959. No one was able to find a third will. Sister-in-law testified that T had wanted the Milwaukee will to stand. Both wills leave the bulk of the estate to two relatives of her deceased husband. The Milwaukie will leaves property to her grand niece. The Kanakkee will elaves property to her brother Robert whom she was not living with at the time of her death.

o   Procedural History: The county held a joint hearing on all three petitions. One for each will and one saying that she died intestate.

o   Issue: Was the finding of the trial court that the deceased revoked the Kankakee will under the mistaken belief that she was thereby reinstating the prior Milwaukee will is against the great weight and clear preponderance of the evidence?

o   Holding: No, the Milwaukee will is valid.

o   Rule: Doctrine of relative revocation: Testator executes one will and thereafter attempts to revoke it by making a later testamentary disposition which for some reason proves ineffective or in some cases the testator revokes a later will under the mistaken belief that by so doing he is reinstating a prior will. In this type of case it renders the revocation ineffective.

o   Reasoning: Since both wills leave property to her husbands relatives, and not next of kin, it is somewhat obvious that she did not want to die intestate. Therefore, it makes sense that she would have wanted to reinstate the Milwaukee will.

o   English common law on revival – wills do not become effective until the testator’s death, so the original will can be considered valid if the will revoking it is destroyed

o   The rest of the states hold that will 2 does revoke will 1, some believe if the testator intends will 1 can be revived, a minority of states say the will cannot be revived unless reexecuted with testamentary formalities

o   Estate of Anderson

o   Court of Appeal of California

o   Facts: The decedent’s 1982 will exercised a testamentary power of appointment over a trust created by her husband. The decedent’s 1993 will revoked the 1982 will, but failed to provide for the power of appointment. The trust created by the husband in his will paid income to Anderson during her life and gave her a testamentary power of appointment over 50% of the trust assets. The will expressly required that the T exercise the power of appointment, her 50% share of trust assets would be divided equally among her husbands living issue by right of representation. In her first will she gave power of appointment to her daughter. After to moving to Pasadena, CA T hired a local law firm to rework her estate plan. T requested that the partner convert her existing estate plan from a will into a living trust with a pour-over will. She also indicated that she wanted to make a 1K gift to each grandchild of her former husband. Her assets were worth 1.3 mill including 500K attributable to her share of the trust. The attorney made handwritten notes. He put OK beside the articles that required changes, but not by the POA. The associate assigned to the estate planning documents did not include the POA. Andreson signed and attested to the will without the POA. Andreson wrote the bank trustee concerning the trust. They said she had a 50% POA. She saw her attorney, he suggested a generation-skipping trust with a life estate for DePaul and the remainder going to her issue.

o   Procedural History: The estate sought to probate the 1993 will and that portion of the 1982 will that exercised the power. Appellant contestant objected. Appellant contestant sought review from the Superior Court of Los Angeles County which admitted to probate the deceden’ts 1993 will and a portion of the decedent’s 1982 will that exercise a testamentary power of appointment.

o   Holding: The court affirmed the trial court’s decision to probate the decedent’s 1993 will and a portion of the decedent’s 1982 will.

o   Rules:

o   §26– A will or any part thereof is revoked by a subsequent will which revokes the prior will or part thereof expressly or inconsistency (Prob.Code. §6120. As a general rule, the presence of a revoking clause in a later writing is accepted as conclusive proof of an intention on the decedent’s part to nullify the provisions of an earlier instrument. In construing a general revocation clause, an exercise of a power of appt. in an earlier will is treated in the same manner as any other provision of that will: A will does not become legally operative until the death of the testator. Hence, the terms of the exercise of a power that are set forth in the will of the donee of the power may be revoked or amended by the donee to the same extent as any other provisions of the will.

o   § 26 (3a)(3b) DRR – Under the doctrine of DRR, an earlier will, revoked only to give effect to a later one on the supposition that the later one will become effective, remains in effect to the extent that the later will proves ineffective. The doctrine is designed to carry out the probable intention of the testator when there is no reason to suppose that he or she intended to revoke an earlier will off the later will became inoperative. The testator has the power to make an expressly conditional revocation. However, if the testator revokes under some mistake, the condition may be implied or presumed. A will may prove ineffective if its results in total or partial intestacy, or if, as a result of a mistake on the testator’s part, it purports to revoke a prior will and fails in a material way to distribute the estate in accordance with the testator’s intent. The doctrine is thus not limited to invalid final wills that would not otherwise result in intestacy. In the construction of wills the paramount rule is that a will is to be construed according to the intention of the testator.

o   Wills § 36 – A will is to be construed according to the intention of the testator, and is not his or her imperfect attempt to express it. Any conclusion as to the testator’s intention must be considered in the light of his or her knowledge at the time of execution of the will. Courts may consider extrinsic evidence of the testator’s intent, including statements of the testator. Extrinsic evidence is admissible to explain any ambiguity arising on the face of a will, or to resolve a latent ambiguity which does not so appear. A latent ambiguity is one which is not apparent on the face of the will but is disclosed by some fact collateral to it. The court must determine the true meaning of the instrument in the light of the evidence available. It can neither exclude extrinsic evidence relevant tot hat determination not invoke such evidence to write a new or different instrument.

o   Reasoning: Although the decedent had to specifically mention the power in order to revoke it in the second will. However, the doctrine of (DRR) applied to prevent the power from being revoked by the second will. DRR could be used to effectuate the decedent’s intent even though the second will did not result in total or partial intestacy. The evidence was sufficient that the decedent conditioned revocation of the first will on exercise of the power. The effect of the second will without the power would have given certain beneficiaries 20 times what the decedent specifically provide for in her second will. Also, she specifically gave 1K gifts to all of Irwin’s grandchildren, if she had intended more, she would have provided for them directly.

·         UPC § 2-509 Revival of Revoked Will

·         If a subsequent will that wholly revoked a previous will is thereafter revoked by a revocatory act under §2=507(a)(2), the previous will remains revoked unless it is revived. The previous will is revived if it is evident from the circumstances of the revocation of the subsequent will or from the testator’s contemporary or subsequent declarations that the testator intended the previous will to take effect as executed.

·         If a subsequent will that partly revoked a previous will is thereafter revoked by a revocatory act under § 2-507(a)(2), a revoked part of the previous will is revived unless it is evident from the circumstances of the revocation of the subsequent will or from the testator’s contemporary or subsequent declarations that the testator did not intend the revoked part to take effect as executed.

·         If a subsequent will that revoked a previous will in whole or in part is thereafter revoked by another, later, will the previous will remains revoked in whole or in aprt unless it or its revoked part is revived. The previous will or its revoked part is revived to the extent it appears from the terms of the later will that the testator intended the previous will to take effect.

·         Revocation by Operation of Law: Change in Family Circumstances – In all but a tiny handful of states, statutes provide that a divorce revokes any provision in the decedent’s will for the divorced spouse. In the remaining states, revocation occurs only of divorce is accompanied by a property settlement. Usually do not apply to nonprobate transfers.

·         UPC § 2-804 Revocation of Probate and Nonprobate Transfers by Divorce. No Revocation by Other Changes of Circumstances

·         Definitions Omitted

·         Revocation Upon Divorce – Except as provided by the express terms of a governing instrument, a court order, or a contract relating to the division of the marital estate made between the divorced individuals before or after the marriage, divorce or annulment, the divorce or annulment of a marriage:

·         Revokes any revocable (i) disposition or appointment of property made by a divorced individual to his or her former spouse in a governing instrument and any disposition or appointment created by law or in a governing instrument to a relative of the divorced individual’s former spouse (ii) provision in a governing instrument conferring a general or nongeneral power of appointment on the divorced individual’s former spouse or on a relative of the divorced individual’s former spouse, and (iii) nomination in a governing instrument, nominating a divorced individual’s former spouse or a relative of the divorced individual’s former spouse to serve in any fiduciary or representative capacity, including a personal representative, executor, trustee, conservator, agent or guardian; and

·         Severs the interests of the former spouses in property held by them at the time of the divorce or annulment as joint tenants with the right of survivorship, transforming the interests of the former spouses into equal tenancies in common…

·         (d) Effect of Revocation – Provisions of a governing instrument are given effect as if the former spouse and relatives of the former spouse disclaimed all provisions revoked by this section or, in the case of a revoked nomination in a fiduciary or representative capacity, as if the former spouse and relatives of the former spouse died immediately before the divorce annulment

·         (f) No revocation for Other Changes in Circumstances. No change of circumstances other than as described in this section and in Section 2-803 dealing with homicide effects a revocation.

·         Marriage – If the testator executes his will and subsequently marries, a large majority of states have statutes giving the spouse her intestate share, unless it appears from the will that the omission was intentional or the spouse is provided for in the will or by a will substitute with the intent that the transfer be in lieu of a testamentary provision

·         Birth of Children – A small minority of states either by statute or judicial decision follow the common law rule that marriage followed by birth of issue revokes a will executed before marriage, but this rule has no been incorporated in the UPC and is disappearing.

·         Almost all states have pretermitted child statutes – giving a child born after execution a share in the parent’s estate

·         In Class Problem Set #2

·         Key facts:

o   Suffered neurological condition

o   Morphine injection two times per month beginning in Nov.

o   He referred to Carrie as his niece on four occasions

o   Will-refers to three children. Only has two children.

o   Mentions blueberry muffins in will.

o   The will was executed on the fourth day of the morphine treatment, the second treatment of the month

o   Wife died 4 years before his death and close to four years before he executed the will

o   Remained close to Cari during W’s life.

o   Occasional calls his restaurant “Hola”, “Mi Loca” “My Crazy”

o   Everyone is inheriting, it is not uneven

C.      Section C. Components of a Will

1.      Integration of Wills – All papers included in the will must be present at the time of execution. Will should be physically connected at the time of execution.

1.      Estate of Beale: Pages added after signing not valid. The rest of the will was upheld.

2.      RS 3rd Property § 3.4 Republication by Codicil – a will is treated as if it were executed when its most recent codicil was executed, whether or not the codicil expressly republishes the prior will, unless the effect of so treating it would be inconsistent with the testator’s intent.

3.      Incorporation by reference – incorporates into a will language or instruments that have never been validly executed.

1.      UPC § 2-510 – Any writing in existence when a will is executed may be incorporated by reference if the language of the will manifests this intent and describes the writing sufficiently to permit its identification.

2.      Clark v. Greenhalge

Facts: T duly executed a will in 1977. In 1972 she created a list of tangible personal property to be distributed. She wrote Farm Scene painting GC and HN. She told GC and HN to give the painting to Virginia Clark. T told VC that the painting would belong to her after HN’s death. HN wrote down the gift in a book. Executor insisted on keeping the painting and stated that the wishes in the book were not binding.

Procedural History: Probate judge found that the painting was to be given to VC. He held that the notebook was incorporated by reference into the terms of the will.

Issue: Were specific written bequests of personal property contained in a notebook maintained by a testatrix were incorporated by reference into the terms of the testatrix’s will.

Rule: A properly executed will may incorporate by reference into its provisions any document or paper not so executed and witnessed, whether the paper referred to be in the form of…a mere list or memorandum…if it was in existence at the time of the execution of the will, and is identified by clear and satisfactory proof.

Reasoning: It was the intent of the testator that the property list in the book be incorporated into the will. The notebook was in existence on the dates HN executed the codicils to her will which republished her will and that it thereby was incorporated into the will pursuant to the language and spirit of Article Fifth.

3.      2-513 Separate Writing Identifying Bequest of Tangible Property – Whether or not the provisions relating to holographic wills applu, a will may refer to a written statement or list to dispose of items of tangible personal property not otherwise specifically disposed of by the will other than money. To be admissible under this section as evidence of the intended disposition, the writing must be signed by the testator and must describe the items and the devisees with reasonable certainty. The writing may be referred to as one to be in existence at the time of the testator’s death; it may be prepared before or after the execution of the will; it may be altered by the testator after its preparation; and it may be a writing that has no significance apart from its effect on the dispositions made by the will.

a.       Johnson v. Johnson

Facts: T’s will gave $10 to his brother in handwriting on his typewritten will. Deceased told his insurance counselor that he was working on a new will in 1947. In 1946 he told his property manager that he wanted him to witness his will. Then later he told him he was changing his will by codicil.

Issue: Is the instrument one complete integrated writing, partly typed and partly handwritten; or is it an unexecuted nonholographic will to which is appended a valid holographic codicil?

Holding: The valid holographic codicil incorporated the prior will by reference and republished and validated the prior will as of the date of the codicil, thus giving effect to the intention of the testator.

Rule: The fact that the codicil was written on the same piece of paper as the typewritten will does not invalidate the codicil. !!!A codicil validly executed operates as a republication of the will no matter what defects may have existed in the execution of the earlier document that the instrumements are incorporated as one and that proper execution of the codicil extends also to the will.

New York Rule: A properly executed codicil validates a will originally invalid for want of testamentary capacity, undue influence, or revocation but does not validate a will defectively executed because of improper attestation.

Corn J. Concurring: If the will follows the intent of the testator it should be probated lest a miscarriage of justice.

Halley Dissenting: Typewritten part is not will and the handwritten part is not a codicil. The handwritten part is a continuation and neither has been attested. Never has there been a will validated by a handwritten codicil when the will was not signed, dated or attested to.

4.      Acts of Independent Significance – permits extrinsic evidence to identify the will beneficiaries or property passing under the will. If the beneficiary or property designations are identified by acts or events that have a lifetime motive and significance apart from their effect on the will, the gift will be upheld.

1.      §2-512 Events of Independent Significance – A will may dispose of property by reference to acts and events that have significance apart from their effect upon the dispositions made by the will. Whether they occur before or after the execution of the will or before or after the testator’s death. The execution or revocation of another individual’s will is such an event.

D.      Contracts Relating to Wills

D. Contracts Relating to Wills

  1. A person may enter into a contract to make a will or a contract not to revoke a will.

a.       Contract law applies.

b.      After a contract becomes binding, a party dies leaving a will not complying with the contract, the will is probated but the contract beneficiary is entitled to remedy for the broken contract.

                                                  i.      An award to the contract beneficiary of the value of the property which was to come to her under the contractual will

                                                ii.      An order compelling the decedent’s successors to transfer the property to the contract beneficiary in accordance with the deceased’s agreement.

c.       Many states require that the contract be written as specified by the statutes of frauds

1.      If the contract does not meet the statute of frauds requirements, the contract beneficiary may nonetheless be entitled to restitution of the value to the decedent of services rendered

2.      Contracts Not to Revoke a Will

1.      Joint will – one instrument executed by two person as the will of both – it is probated two times, once for each person, not a good idea

2.      Mutual wills – are the separate wills of two or more persons that contain similar or reciprocal provisions.

3.      Joint and mutual will – refers to a joint will in which the respective testators make similar or reciprocal provisions

a.       These joint and or mutual wills do not give rise to a presumption of a contract

b.      Astute lawyers include a clause that will was not based on a contract

4.      § 2-514 Contracts Concerning Succession – A contract to make a will or devise, or not to revoke a will or devise, or to die intestate, if executed after the effective date of this Article, may be established only by (i) provisions of a will stating material provisions of the contract, (ii) an express reference in a will to a contract and extrinsic evidence proving the terms of the contract, or (iii) a writing signed by the decedent evidencing the contract. The execution of a joint will or mutual wills does not create a presumption of a contract not to revoke the will or wills.


Chapter 5

Nonprobate Transfers and Planning for Incapacity

Section A. An Introduction to Will Substitutes

  1. The Will Substitutes

                                                              i.      Asset specific

                                                            ii.      Avoids probate

                                                          iii.      Does not have to abide by the Wills Act formailities

    1. Life Insurance

                                                              i.      Beneficiary does not have rights until owner dies

    1. Pension Accounts
    2. Joint Accounts (Bank, Brokerage and Mutual Fund Accounts)

                                                              i.      Pay on death account – accounts over which the depositor retains explicit lifetime dominion while designating beneficiaries to take on his death.

                                                            ii.      Joint bank account – owner creates a present interest in his donee-cotenant

1.      Donee receives an interest equal to the donor’s

2.      Donor loses the power to revoke the transfer

3.      Cotenants must act together to transfer the realty

                                                          iii.      Street accounts – are nominally joint, the beneficial owner of the securities may deal with them as though he has not made the cotenancy designation, but on the owner’s death the cotenant succeeds to the securities or other account proceeds.

    1. Revocable (Inter Vivos) Trusts

                                                              i.      Either by declaration of trust or by transfer to a third-party trustee, the appropriate trust terms can replicate the incidents of a will.

1.      The owner who retains both the equitable life interest and the power to alter and revoke the beneficiary designation has used the trust form to achieve the effect of testation.

    1. Imperfect Will Substitutes

                                                              i.      Common law joint tenancy – cotenant acquires an interest that is no longer revocable and ambulatory.

                                                            ii.      Both cotenants must ordinarily join in any subsequent transfer

                                                          iii.      Allow the survivor to obtain marketable title without probate

                                                          iv.      A death certificate rather than a probate decree suffices to transfer title.

Section B. Revocable Trusts

1.   Introduction

a.   A revocable inter vivos trust is the most flexible if all will substitutes because the donor can draft both the dispositive and the administrative provisions precisely to the donor’s liking

                                         i.    Deed of trust – the creator of the trust known as the settlor, transfers legal title to property to another person as trustee pursuant to a writing in which the settlor retains the power to revoke, alter or amend the trust and the right to trust income during lifetime.

1.   On the settlor’s death, the trust assets are to be distributed to or held in further trust for other beneficiaries.

2.   All jurisdictions now recognize the validity of a trust where property is transferred to another person as trustee and the settlor reserves the power to revoke the trust during life.

3.   The settlor may also reserve an income interest and a testamentary power of appointment.

                                        ii.    Revocable declaration of trust – the settlor declares himself trustee for the benefit of himself during lifetime, with the remainder to pass to others at his death.

1.   Causes assets to pass to others at the settlor’s death without complying with Wills Act formalities.

2.   Farkas v. Williams

Facts: F died intestate at the age of 67, leaving as his only heirs-at-law brothers and sisters, a nephew and a niece. Williams was his employee before he retired. On four occasions F purchased stock of Investors Mutual Inc. At the time of each purchase he executed a written application to Investors Mutual, Inc instructing them to issue the stock in his name as trustee for Richard J. Williams. There was in writing by signature on the fact of the application. Farkas signed separate declarations of trust, all of which were identical except as to dates. During his lifetime all cash dividends were to be paid to F. The title to any stock subject to any subsequent payments or distributions shall be vested absolutely in the beneficiary. F had the right to change the beneficiary during his life.

Procedural History: The decree of the circuit court found that said declarations were testamentary in character, and not having been executed with the formalities of a will, were invalid and directed that the stick be award to the ps as an asset of the estate of Farkas. Upon appeal to the Appellate Court, the decree was affirmed.

Issue: Did the trust instruments create valid inter vivos trusts effective to give the purported beneficiary, Williams, title to the stock in question after the death of the settlor-trustee, Farkas?

Holding: The trust declarations executed by F constituted valid inter vivos trusts and were not attempted testamentary dispositions.

Rule: An absolute owner can dispose of his property, either in his lifetime or by will, in any way he sees fit without notifying or securing approval from anyone and without being held to the duties of a fiduciary in so doing.

Reasoning: Immediately after the execution of the instrument, he could not deal with the stock  therein referred to the same as if he owned the property absolutely, but only in accordance with the terms of the instrument. He manifested an intention to bind himself to having this property pass upon his death to Williams, unless he changed the beneficiary or revoked the trust. The power of F to vote, sell, redeem, exchange or otherwise deal in the stock was reserved to him as trustee and it was only upon sale or redemption that he was entitled to keep the proceeds for his own use. Williams would have had an enforceable claim against Farkas’ estate for whatever damage had been suffered.

2.   A trust is a management relation whereby the trustee manages property for the benefit of one or more beneficiaries.

a.   The trustee holds legal title to the property and, in the usual trust, can sell the trust property and replace it with property thought more desirable

b.   The beneficiaries hold equitable title

                                         i.    Equitable title- an equity court enforces their rights against the trustee and third parties

                                        ii.    The trustee is held to a fiduciary standard of conduct

1.   Loyalty

2.   Prudence

3.   And other duties

                                      iii.    A trustee can be one of the beneficiaries of the trust

1.   There is no trust if the trustee is the sole beneficiary of the trust

c.   § 603(a) Uniform Trust Code – while a trust is revocable and the settlor has capacity to revoke the trust rights of the beneficiaries are subject to the control of and the duties of the trustee are owned exclusively to the settlor.

d.   Estate of Brenner – Brenner took title to the property described in the exhibit as trustee, acquired additional real estate as trustee and that he executed a contract as trustee relating to both properties. The evidence and inferences therefrom being in conflict, the trial court’s determination that Brenner intended to create a trust and thereby provide for his children and niece, as natural objects of his bounty may not be disturbed on review. Court found he did not maintain too much control.

3.   Revocable declaration of trust – sometimes called a living trust – declare yourself trustee of your property by using a revocable declaration of trust, with the trust property to pass to named beneficiaries upon your death.

4.   RE Estate and Trust of Pilafas – common law of presumption – a testator destroyed his will with the intention of revoking it if the will is last seen in the testator’s possession and cannot be found after his death. However, unlike a will, the creation of a trust involves the present transfer of property interests in the trust corpus to the beneficiaries. These interest cannot be taken from the beneficiaries except in accordance with a provision of the trust instrument, or by their own acts or by a decree of a court. Even a revocable trust vests the trust beneficiary with a legal right to enforce the terms of the trust. The terms of the trust also limit the powers of the settlor and the trustee over the trust corpus, even when the settlor declares himself trustee for the benefit of himself and others. Revocation must be delivered during the Settlor’s lifetime by writing delivered to the Trustee. The trust was not revoked and remained valid.

5.   Restatement 3rd of Trusts § 63 – provides that a revocable trust may be revoked, absent contrary provision in the terms of the trust, in any ways that provides clear and convincing evidence of the settlor’s intention to do so, which includes revocation by will.

6.   State Street Bank and Trust Co. v. Reiser – Where a person places property in trust and reserves the right to amend and revoke, or to direct disposition of principal and income, the settlor’s creditor’s may, following the death of the settlor, reach in satisfaction of the settlor’s debts to them, to the extent not satisfied by the settlor’s estate, those assets owned by the trust over which the settlor had such control att he time of his death as would have enabled the settlor to use the trust assets for his own benefit. See IRC §§2308 and 2041.

a.   Life insurance proceeds and retirement benefits are usually exempt from insured’s creditors if payable to a spouse or child.

b.   US Savings bonds with a POD may be exempt.

c.   The creditors of a joint tenant holding a joint tenancy in land cannot reach the land after the joint tenant’s death because the deceased joint tenant’s interest has vanished.

7.   Pour-Over Wills

a.   O sets up a revocable inter vivos trust naming X as trustee.

b.   O transfers to X, as trustee, his stocks and bonds

c.   O then executes a will devising the residue of his estate to X as trustee to hold under the terms of the inter vivos trust.

                                         i.    Incorporation by reference – A will can incorporate by reference a trust instrument in existence at the time the will is executed

                                        ii.    Independent significance – a will may dispose of property by referring to some act that has significance apart from disposing of probate assets, under this doctrine, the trust instrument does not have to be in existence when the will is executed, but the trust must have some assets in it before the time of the testator’s death.

                                      iii.    UPC §2-511 –

a.   A will may validly devise property to the trustee of a trust established or to be established

                                                                                         i.    During the testator’s lifetime by the testator, by the testator and some other person, including a funded or unfunded life insurance trust, although the settlor has reserved any or all rights of ownership of the insurance contracts or

                                                                                        ii.    At the testator’s death by the testator’s devise to the trustee, if the trust is identified in the testator’s will and its terms are set forth in a written instrument, other than a will , executed before, concurrently with or after the execution of the testator’s will or in another individual’s will if that other individual has predeceased the testator, regardless of the existence, size or character of the corpus of the trust. The devise is not invalid because the trust is amendable or revocable, or because the trust was amended after the execution of the will or the testator’s death.

b.   Unless the testator’s will provides otherwise, property devised to a trust described in subsection (a) is not held under a testamentary trust of the testor, but it becomes a part of the trust to which it is devised, and must be administered and disposed of in accordance with the provisions of the governing instrument setting forth the terms of the trust, including any amendments thereto made before or after the testator’s death.

c.   Unless the testator’s will provides otherwise, a revocation or termination of the trust before the testator’s death causes the devise to lapse.

2.   Clymer v. Mayo – woman had revocable trust with ex-husband as beneficiary. Her parents were her intestate heirs. Divorce revokes interests in trusts and wills.

3.   Unfunded life insurance trust – when a settlor names the trustee of her inter vivos  trust as the beneficiary of her life insurance policy but does not add any other funds or assets to the trust

4.   Funded inter vivos trust – when other assets are added to the trust

5.   Trust res – property in the trust

6.   If the testator executes a trust agreement during life then its trustee is a permissible taker under the testator’s will

7.   Revocation by divorce – Some states provide that divorce revokes any provision in a revocable trust for the ex-spouse who is deemed to have predeceased the settlor

8.   UPC § 2-804 – divorce revokes dispositions in favor of the divorced spouse in revocable inter vivos trusts as well as in other will substitutes. Also, revokes any provision for a relative of the divorced spouse.

9.   Egelhoff- federal law preempts the applicability of state revocation on divorce statutes to federally regulated pension benefits.

10.                Testamentary trust – trust beneficiary is named in the will

8.   Use of Revocable Trusts in Estate Planning

a.   Introduction

                                         i.    Declaration of trust – the settlor becomes the trustee of the trust property

1.   Settlor should name a successor trustee to take over upon S’s death

                                        ii.    Deed of trust – naming a third party as trustee

1.   Settlor can be named co-trustee

2.   Can be funded, or can be unfunded

3.   Can make nonprobate assets payable to the trust

4.   CA – probate is optional for transfers to spouses

b.   Consequences During Life of Settlor

                                         i.    Property management by fiduciary – A third party fiduciary may be selected to manage a funded revocable trust. Settlor can change fiduciary based on performance.

                                        ii.    Keeping title clear – may be used to help husband and wives keep community property separate from other property

                                      iii.    Income and gift taxes – gift and estate taxes assets in a revocable trust are treated as still owned by the settlor. There are no federal tax advantages in creating a revocable trust. It is taxable during the life of the settlor.

                                      iv.    Dealing with incompetency – if settlor becomes incompetent, a co-trustee can act on the behalf of the trustee

c.   Consequences at Death of Settlor: Avoidance of Probate

                                         i.    Costs. – If there are trustee fees, they will be smaller than court fees.

1.   Lawyers charge more for trust documents. There me be stock transfer fees.

                                        ii.    Delays -   Income and principal can be distributed much more quickly. Trustees have less rules to deal with when taking care of a business owned by the settlor.

                                      iii.    Creditors – disadvantage, creditors rights last longer

                                      iv.    Publicity – trusts are not recorded in a public place like probate court

                                       v.    Ancillary probate – land in another state can be transferred to a revocable inter vivos trust. Title of the land is changed to the trustee during life.

                                      vi.    Avoiding restrictions protecting family members – elective share to the spouse can be avoided in a trust in some jurisdictions, trend is away from allowing trusts to avoid elective share to the spouse

                                    vii.    Avoiding restrictions on testamentary trusts – do not have to abide by testamentary rules if you have a revocable inter vivos trust

1.   Trustee can be someone from another state

                                   viii.    Choosing the law of another jurisdiction to govern – the settlor of an inter vivos trust of personal property may choose the state law that is to govern the trust

1.   Place where land is located governs trusts of land

2.   Can create a perpetual trust in other states that does not incur generation skipping taxes

3.   UPC § 2-703 – T may select the state law to govern the meaning and legal effect of his will including trusts created by will, unless that law is contrary to the domiciliary state’s law protecting the surviving spouse or any other public policy of the home state

                                      ix.    Lack of certainty in the law – trusts are newer than wills and not all of the problems have been worked out

                                       x.    Avoiding will contests – a revocable trust like a will can be contested for lack of mental capacity and undue influence.

1.   More difficult to contest because trust instrument is not a public document

2.   Heirs cannot view it and will have no idea of what they might inherit and what it says

                                      xi.    Estate taxation – IRC § 2038 – assets of a revocable trust are included in the gross estate of the settlor

                                    xii.    Controlling surviving spouse’s disposition- Surviving spouse may dip into principal if necessary

                                   xiii.    Custodial trusts- Trustee use trust to support the beneficiary and for the b’s dependants

Section C. Life Insurance, Pension Accounts, Bank Accounts, and Other Payable-On-Death-Arrangements

                                                              i.      Provides some protection against fraud because neutral financial institutions

1.      Life insurance

a.       Whole life insurance – involves life insurance and a savings plan

b.      Term life insurance – no savings feature, has no cash surrender value

2.      Wilhoit v. Peoples Life Insurance Co.

Facts: Husband purchased a life insurance policy before he died. Wife inherited the life insurance policy and then asked the company to keep it in trust for Robert G. Owens. Robert died. He devised all of his property to Thomas J. Owens (d). Wife’s will devised the property to the son of her step son, Robert Wilhoit (p).

Procedural History: Company gave the money to Owens. DC Judgment was entered in favor of the p in the sum of $4,700 for Wilhoit.

Issue: If a beneficiary of a contract for savings with an insurance company is listed in the contract, but that person predeceases the policy owner, should the legatee listed in the will inherit rather than the heirs of the beneficiary listed in the policy?

Holding: Robert Wilhoit has the legal right to the money.

Rule: POD designations other than those listed in life insurance policies are invalid of the will lists another beneficiary.

Reasoning: The arrangement between the parties was the result of a separate and independent agreement, unrelated to the terms of the policy…a separate and independent agreement, unrelated to the terms of the policy. She had the right to withdraw the money at the end of any six-month period and only at her death was Owens allowed to collect. Mrs. Wilhoit specifically devised the money to Robert Wilhoit. Mr. Owens never made mention of the money in his will.

3.      Estate of Hillowitz

Facts: Deceased was a member of an investment club. His partnership agreement said that his partnership and share would pass to his wife. The executors say that the proceeds should pass to her as an asset of his estate.

Issue: Can one make a disposition of property via a partnership agreement that does not conform with the statute of wills?

Holding: Yes, partnership agreements may dispose of property without conforming to the statute of wills.

Rule: Partnership undertakings are in effect nothing more or less than third party beneficiary contracts, performable at death.

4.      Family Limited Partnerships – the decedent transfers assets to the partnership in exchange for a limited partnership interest. The decedent’s family likewise transfers assets to the partnership in exchange for limited partnership interests to pass to his family. The value of those interests are discontinued for estate tax purposes because of their lack of control rights and nonmarketability.

5.      §6-101 UPC 1990 – Nonprobate transfers on death

a.       A provision for a Nonprobate transfer on death in an insurance policy, contract of employment, bond, mortgage, promissory note, certified or uncertificated security, account agreement, custodial agreement, deposit agreement, compensation plan, pension plan, individual retirement plan, employee benefit plan, trust conveyance, deed of gift, marital property agreement, or other written instrument of a similar nature is nontestatmentary. This subsection includes a written provision that:

                                                                                                                                      i.      Money or other benefits due to, controlled by or owned by a decedent before death must be paid after the decedent’s death to a person whom the decedent designates either in the instrument or in a separate writing, including a will, executed either before or at the same time as the instrument or later;

                                                                                                                                    ii.      Money due or to become due under the instrument ceases to be payable in the event of death of the promisee or the promisor before payment or demand; or

                                                                                                                                  iii.      Any property controlled by or owned by the decedent before death which is the subject of the instrument passes to a person the decent designates either in the instrument or in a separate writing, including a will executed either before or at the same time as the instrument or later

b.      This section does not limit rights of creditors under other laws of this State.

c.       Note: this section is silent as to what happens when a beneficiary predeceases the benefactor.

d.      Now courts will usually give heirs of deceased beneficiaries the property

6.      Cook v. Equitable Life Assurance Society, 428 N.E.2d 110

a.       Man and woman were married. Got divorced. Man remarried. Still had old wife designated as beneficiary on insurance policy. Court assumed that Douglas’s designation to former wife was not void by divorce. Wanted payments to be quick. Husband “slumbered on his rights”.

  1. Pension Accounts
    1. The enhancement of life expectancy

                                                              i.      Used to be 45 is now 75 and climbing, creates more need for savings

    1. Pension wealth

                                                              i.      Government incentives for tax qualified pension plans

1.      Tax deferred in other words saving with pre-tax dollars

2.      Accrue compound interest on a tax deferred basis

3.      Retirees usually have lower incomes, so their retirement plans are taxed at lower rates.

    1. Annuitization Eliminates Succession

                                                              i.      Governments incentives for consuming retirement savings during life

                                                            ii.      Annuitized accounts use large pools of people, disappear on the death of the annuitant

                                                          iii.      Annuties – in pure form is a payment every year for the rest of the b’s life.

1.      Shifts financial risk of living too long to a pension fund or insurance company

    1. Defined Benefit plans

                                                              i.      Employer promises to pay an annuity upon retirement

    1. Defined contribution plan

                                                              i.      Both the employee and the employer make contributions to a specific pension account for the employee

                                                            ii.      Lump sum payouts on death

                                                          iii.      Tax advantaged savings account that ends with a nonprobate transfer

    1. Employee Retirement Income Security Act of 1974 (ERISA) – federal regulation of pension plans

                                                              i.      Egelhoff v. Egelhoff

Facts: Man’s ex-wife was listed as pension beneficiary. When he died he was remarried with children.

Issue: Does the ERISA pre-empt that statute to the extent it applies to ERIA plans?

Holding: ERISA pre-empts state statutes.

WA State Rule: Designation of a spouse as the beneficiary of a nonprobate asset is revoked automatically upon divorce.

Rule: 29 U.S.C § 1144 states that ERISA shall supercede any and all State laws insofar as they may now or hereafter relate to any employee benefit plan covered by ERISA.

Reasoning: In Shaw v. Delta Airlines, Inc. the S.Ct. previously held that if it has a connection with or reference to such a plan state law relates to ERISA. One of the goals of ERISA is to create uniform plan administration. The state law interferes with that goal.

Dissent: ERISA does not pre-empt all state laws. Ex-wife is inheriting more than her fair share of the property. He says there is a relationship between this statute and the slayer statute.

                                                            ii.      Morgan – ERISA did not pre-empt the state’s simultaneous death statute, administrator was allowed to construe time of death

                                                          iii.      Metropolitan Life v. Johnson – applied Federal common law substantial compliance to a beneficiary change form where the wrong plan was checked

  1. Multiple-Party Bank and Brokerage Accounts
    1. Multiple party bank and brokerage accounts include a joint and survivor account, a POD account, an agency account and a savings account (Totten) trust.

                                                              i.      If an agency account is intended or a POD account is intended in a state that does not allow them, the survivor is not entitled to the proceeds in the account, which instead belong to the depositor’s estate.

                                                            ii.      Franklin v. Anna National Bank of Anna,

1.      Man had joint account with deceased wive’s sister. Card stated that one sig was required for withdrawals and that funds deposited are owned by the signatories as joint tenants with the right of survivorship. Franklin the executor was man’s final caretaker. The one claiming adversely has the burden of establishing by clear and convincing evidence that a gift was not intended.

    1. Robinson v. Delfino – A joint bank account conclusively establishes a right of survivorship; evidence to the contrary is not admissible.
    2. The savings account or Totten trust – tentative revocable [savings account] trust, may imply some fiduciary duty to b.
    3. Uniform Transfer on Death Security Registration Act (UTDSR) permits securities registered in a transfer-on-death form.
    4. UPC §§6-201 to 6-227 authorizes joint tenancy account with the right of survivorship, an agency account, and a POD account. Totten trust is abolished and is treated as a POD account.

                                                              i.      Joint beneficiaries have rights to the sums according to their contributions during their lifetimes

                                                            ii.      POD accounts cannot be accessed by the beneficiary during the lifetime of the owner

                                                          iii.      POD – requires survivorship by the beneficiary

                                                          iv.      UPC §2-706 – antilapse statute substitutes in place of a deceased beneficiary of a POD bank account the beneficiary’s issue if the beneficiary was a close relative of the decedent

Section D. Joint Tenancies in Realty

1.      Joint tenancy in land gives the joint tenants equal interests upon creation.

2.      Require the agreement of all tenants to take most important actions.

3.      A person who during life transfers land into joint tenancy cannot revoke it during life (not a pure will substitute)

                                                              i.      Cannot transfer shares by will

                                                            ii.      Creditor must take the joint tenant’s share during life

Section E. Planning for Incapacity

1.      Durable Power of Attorney

                                                              i.      An ordinary power of attorney creates an agency relationship whereby the agent, called attorney-in-fact is given a written authorization to act on behalf of the principal.

1.      Limited by agency law

2.      Terminates on the incapacity of the principal

b.      §§UPC 5-501 to 5-505 Durable Power of attorney

1.      Continues throughout the incapacity of the principal until the principal dies; otherwise controlled by the law of agency

a.       Principal when competent can terminate the power of attorney

b.      Must be wrfitten instruments in some states witnessed or notarized

c.       Durable power ceased when the principal dies

d.      If the agent dies there can be a second

e.       Useful if you do not want to create a trust

c.       Frazen v. Norwest Bank of Colorado –

1.      Does a trust need to be named in the POA in order for the agent to be able to revoke the trustee. A power of attorney that appears to give the agent sweeping powers to dispose of the principal’s property is to be narrowly construed in light of the circumstances surrounding the execution of the agency instrument. However, the principal may confer authority to amend or revoke trusts on an agent without referring to the trusts by name in the power of attorney.

2.      Under the common law of ademption, when the devised property is not owned by the testator at death, the devisee is adeemed and the devisee takes nothing.

3.      Inter vivos transfers by the decedent over which the decedent retains the power to revoke are included in the decedent’s taxable gross estate.

4.      Townsend v. U.S = gifts made by an agent under a durable power were includible in the principal’s gross estate because the POA did not expressly authorize gifts.

5.      Tip: include in the durable power of attorney the power to make gifts if the donor so desires.

6.      Bad agent may authorize himself to create a trust on principal’s behalf if p becomes incompetent

7.      Possible new law: Power of Attorney Act – All powers of attorney are durable unless the instrument states otherwise (2) the agent will not have the power to amend or revoke trusts or give away the principal’s assets without express authorization to do so.

2.      Directives Regarding Health Care and Disposition of the Body

a.       Advance Directives: Living Wills, Health Care Proxies, and Hybrids Page

                                                              i.      The SCt has held that each person has a constitutional right to make health care decisions, including the right to refuse medical treatment. Cruzan.

                                                            ii.      If state law requirements are met, a person may state her wishes about terminating medical treatment or appointing an agent to make the decision for her.

                                                          iii.      But where a person’s wishes are not clearly expressed, the state may assert an interest in favor of preserving life and preventing the withdrawal of treatment

                                                          iv.      Advance directives resolve a conflict over the wishes of an incompetent person

1.      Living will (aka Medical Directive) – specifies generally or by way of hypothetical examples how one wants to be treated in end-of-life situations or in the event of incompetence;

2.      Proxy directives, such as health care proxy or durable power of attorney for health care, which designate an agent to make health care decisions for the patient

3.      Hybrid or combines directives incorporating both of the first two approaches; directing treatment preferences and designating an agent to make substituted decisions.

                                                            v.      Uniform Health-Care Decisions Act – includes forms that create a durable power of attorney for health care and offer the person a chance to indicate how aggressively he would like to be treated.

1.      The agent must act in accordance with the patient’s wishes.

2.      The health care provider must follow the instructions except where contrary to the provider’s conscience or contrary to generally accepted medical practice

                                                          vi.      Patient Self-Determination Act (PSDA) – The PSDA requires that every patient admitted to a hospital receiving federal funds must be advised of the right to sign an advance directive indicating a desire to withdraw medical treatment in specific situations,

                                                        vii.      Substituted judgment standard – in making health care decisions for an incompetent patient, an agent for health care decisions is supposed to do what the patient would have chosen to do in that situation.

                                                      viii.      Decision maker order of priority in Ill, if more than one person in a class the majority controls:

1.      Guardian of the person

2.      The spouse

3.      Adult son or daughter

4.      Parent

5.      Adult brother or sister

6.      Adult grandchild

7.      Close friend

8.      Guardian of the estate

                                                          ix.      Disposition of the Body – permit a person to give her body to any hospital, physician, medical school or body bank for research or transplantation. It also permits a gift of body parts thereof to any specified individual for therapy or transplantation needed by the individual. A surgeon who relies on the validity of the card or will in good faith is not civilly or criminally liable §11c.


Chapter 6 Construction of Wills

Section A. Mistaken or Ambiguous Language in Wills

1.      The Traditional Approach: No Extrinsic Evidence, No Reformation

1.      Plain meaning or no extrinsic evidence – extrinsic evidence may be admitted to resolve some ambiguities, but the plain meaning of the words of the will cannot be disturbed by evidence that another meaning was intended.

2.      No reformation rule – The reason that the courts will not reform wills is that the court is thereby compelled to interpret the words that the testator actually used, not to interpret the words that the testator is purported to have intended to use.

3.      Mahoney v. Grainger

Facts: Woman told her lawyer that she wanted to leave her estate to her first cousins in equal shares. Lawyer used the language heirs at law which meant by the rules of consanguinity which left her aunt as her only heir at law.

Issue: If the testator states that she would like all of her cousins to inherit in equal shares, but the will says heirs at law can evidence be submitted to contradict the writing of the will.

Holding: No evidence may be submitted, the cousins will not inherit.

Rule: It is only where testamentary language is not clear in its application to facts that evidence may be introduced as to the circumstances under which the testator used that language in order to throw light upon its meaning.

Reasoning: Where no doubt exists as to the property bequeathed or the identity of the beneficiary there is no room for extrinsic evidence, the will must stand as written.

4.      Gustafson v. Svenson – upheld Mahoney v. Grainger, per stirpes inheritance is clear even though testator may not have wanted widow to inherit

5.      Estate of Smith – bequest ended up going to corporation that owned the intended beneficiary, nursing home, because the corporation was named in the will because it used to own the nursing home.

6.      Personal Usage Exception – if the extrinsic evidence shows that the testator always referred to a person in an idiosyncratic manner, the evidence is admissible to show that the testator meant someone other than the person with the legal name of the legatee. Moseley v. Goodman – the court held that the bequest went to Mrs. Lilian Trimble, whom the testator called Mrs. Moseley because her husband worked for Mr. Moseley. She owned the apartment where the testator lived and did kind things for him.

7.      Patent ambiguities – appear on the face of the will, typos etc. , courts have in the past not admitted evidence to clarify

8.      Latent ambiguities – manifests itself when the terms of the will are applied to the testator’s property or designated beneficiaries

                                                              i.      Equivocation -  two or more persons fit the description, evidence will be admitted, direct expressions of the testator’s intent. Bacot – picked the closest same sex lover

                                                            ii.      Misdescription -  When no person fits, but two or more people partially fit. Ihl – divorced wife took over new wife because the testator intended that she inherit because he and she had an interest in antiques.

                                                          iii.      Against the no extrinsic evidence rule

                                                                iv.      Professor Wigmore – says there is no plain meaning, there is only the meaning of the people who wrote the document. The plain meaning is simply the meaning of the people who did not write the document.

                                                                  v.      Holmes – the meaning in the mouth of the normal English speaker under the circumstances

2.      Slouching Toward Reformation: Correcting Mistakes Without the Power to Reform Wills

1.      Why is relief not granted for mistakes when relief is granted for fraud, undue influence, and lack of capacity and relative revocation (thinks son is dead, forgets to update will after birth of a child etc.)

2.      Arnheiter v. Arnheiter

Facts: Testator owned one half interest in a home on Harrison Ave number 317. The will listed the house number as 304.

Issue: Can the court change the number of the house since it is obvious it was an innocent mistake?

Holding: The court rejected the false number because she did not own it and the found that the rest of the description has them come to the conclusion that the property meant was 317 Harrison.

Rule: Falsa demonstratio non nocet. Where a description of a thing or person consists of several particulars and all of them do not fit any one person or thing, less essential particulars may be rejected to provided the remainder of the description clearly fits.

Reasoning: They cannot reform the will, but they can remove a mere erroneous description.

3.      Estate of Gibbs – Under the rules as to construction of a will, unless there is ambiguity in the text of the will read in the light of surrounding circumstances, extrinsic evidence is inadmissible for the purpose of determining intent. However, details of identification, particularly such matters as middle initials, street addresses and the like which are highly susceptible to mistake particularly in metropolitan areas should not be accorded such sanctity as to frustrate an otherwise clearly demonstrable intent.

3.      Openly Reforming Wills for Mistake

1.      Erickson v. Erickson

Facts: The decedent executed a will. At that time he had three daughters and was unmarried. Two days later, he married the d. He died about eight years later.

Procedural History: The probate court had admitted the will of the decedent to probate. The trial court ruled that the decedent’s will which had been executed shortly before his marriage to the defendant provided for the contingency of marriage.

Issue: Should the trial court have admitted extrinsic evidence regarding the decedent’s intent that his will would not be revoked automatically by his subsequent marriage?

Holding: The nature of the provisions for the contingency of marriage do constitute clear and convincing evidence of provision for the contingency of marriage. However, that under the circumstances of this case, the trial court improperly excluded evidence of a mistake by the scrivener that if believed would permit a finding that the will provided for the contingency of marriage.

Rule: On the basis of case law, the question of whether a will provides for the contingency of a subsequent marriage must be determined: (1) from the language of the will itself and (2) without resort to extrinsic evidence of the testator’s intent. However, if there is extrinsic evidence of that error that scrivener made and testators were misled by the scrivener such evidence may be submitted.

Reasoning: It would be preposterous to assume that the decedent was instead executing a will to make provisions that were to be revoked two days later. A third person interfered. It is a serious concern that testamentary intent be implemented.

Restatement (Third) of Property: Wills and Other Donative Transfers (2003)

§ 12.1 Reforming Donative Documents to Correct Mistakes

            A donative document, though unambiguous, may be reformed to conform the text to the donor’s intention if the following are established by clear and convincing evidence:

1.      That a mistake of fact or law, whether in expression or inducement, affected specific terms of the document; and

2.      What the donor’s intention was.

Direct evidence of intention contradicting the plain meaning of the text as well as other evidence of intention may be considered in determining whether elements (1) and (2) have been established by clear and convincing evidence.

1.      John H. Langbein, Curing Execution Errors and Mistaken Terms in Wills

                                      i.      Trend away from formalism

1.      Both by judicial decisions and by legislation, the courts have been empowered to excuse harmless execution errors and to reform mistaken terms

a.       Mistaken terms - § 12 RS 3rd Property: Wills and Other Donative Transfers – authorizes courts to reform mistaken terms in a will.

                                                                                                              i.      Clear and convincing evidence standard

1.      Mistake of fact or law whether in expression or inducement affected specific terms of the document

2.      What the donor’s intention was

2.      Why?

a.       The rise of the nonprobate system

b.      Experience in other jurisdictions

c.       Growing embarrassment that failure to cure well-proved mistakes inflicts unjust enrichment; and

d.      Concern to spare lawyers from needless malpractice liability

3.      Unifying the Law of Probate and Nonprobate Transfers

a.       Clear and convincing evidence has long been enough to rectify mistakes in trusts and other non probate transfers, to unify the two systems we are heading in this direction

4.      Preventing Unjust Enrichment – when someone takes under a mistake in a will they are being unjustly enriched, they would be entitled to restitution under the law of ‘quasi-contracts’ due the fusion of law and equity

5.      Malpractice Liability

a.       Although an attorney who drafts a will improperly can be sued, it does not help the case at hand

b.      Reformation will lessen the need for malpractice suits and will help people who would otherwise have been trying to get money from a lawyer who was judgment proof.

6.      Estate of Lord – court changed mistaken language from trust and trustee to estate and personal representative

7.      Estate of Getman – change oldst to youngest when it was found that the term did not make sense

8.      Herceg (last lines left off by computer error, three prior wills gave to niece and nephew. Courts have sometimes inserted missing words.

9.      Growing number reforming wills for tax advantage.

10.  Holloway – attorney is not liable for drafting an ambiguous document

11.  Gifts by implication –

a.       BIELEY – implying a gift to fill in the omission

b.      Siegel – court places itself in the testator’s shoes and makes the decision

c.       Kime – made executor the beneficiary because non was specified and woman filling in form thought that the term meant beneficiary.

12.  Fleming v. Morrison – B created a fake will to induce F to sleep with him. He told his lawyer that it was a fake will, but did not tell the two witnesses. Court allowed the extrinsic evidence to show that it was not a will.

Section B. Death of Beneficiary Before Death of Testator

1.      Introduction

a.    Lapse – if a devisee does to survive the testator

b.    All gifts made by will are subject to a requirement that the devisee survive the testator, unless the testator specifies otherwise

                                                              i.      Anti-lapse statutes in nearly all states have been enacted to substitute another beneficiary for the predeceased devisee.

                                                            ii.      Common Law Rules

1.      Specific or general devise – If a specific or general devise lapses, the devise falls into the residue

2.      Residuary devise – if the devise of the entire residue lapses because the sole residuary devisee or all the residuary devisees predecease the testator, the heirs of the testator take by intestacy

a.       No-residue of a residue rule roundly criticized by courts and commentators because it does not carry out testator intent - If a share of residue lapses, such as when one of two residuary devisees predeceases the testator, at common law the lapsed residuary passes by intestacy to the testator’s heirs rather than to the remaining residuary devisees.

3.      Class gift – If the devisee is to a class of persons, and one member of the class predeceases the testator, the surviving members of the class divide the gift.

4.       if the devisee is dead at the time the will is executed, or the devisee is a cat or dog or some other ineligible taker, the devise is void.

c.    Estate of Russell

Facts: Russell deceased devised all of the residue of her estate be distributed to Chester H. Quinn. Russell died September 8th, 1965 leaving a validly executed will written on a small card. She left her estate to H. Quinn who was a friend who lived in her home, Roxy Russell a dog and Thelma Russell her niece.

Procedural History: Hembree appeals heirship determination. The TC found that Georgia Nan Russell Hembree was not have any other real property than the gold coin and diamonds. The court concluded that the T intended to and did make an absolute and outright gift to Mr. Quinn of all the residue of her estate. They concluded that there was no lapse.

Issue: If a dog is named as heir with another person and there is an heir who is to inherit the residue in the will, who inherits the share left to the dog?

Holding: The T intended to leave the residue of the estate in equal shares to Quinn and the dog as tenants in common. The part subject to the dog is void and is subject to the lapsed gift rules. The residue of the estate is to pass in equal shares to Quinn and Georgia Nam Russell Hembree, niece.

Rule:

A disposition in equal shares to two beneficiaries cannot be equated with a disposition in equal shares to two beneficiaries cannot be equated with a disposition of the whole to one of them who may use whatever portion thereof as might be necessary on behalf of the other.

A dog cannot be a beneficiary of a will.

Note: It is more likely under the no residue of the residue rule that one half would have gone to Quinn and one half would have passed by intestacy.

2.      Anti-lapse statutes – A typical antilapse statute provides that if a devisee is of a specified relationship to the testator and is survived by issue who survive the testator, the issue are substituted for the predeceased devisee.

a.      Some statutes apply only to descendants

b.      UPC applies only to devises to a grandparent or lineal descendant of a grandparent or step-child.

                                                              i.      §2-605. Antilapse; Deceased Devisee; Class Gifts - If a devisee who is a grandparent or a lineal descendant of a grandparent of the testator is dead at the time of execution of the will, fails to survive the testator, or is treated as if he predeceased the testator, the issue of the deceased devisee who survive the testator by 120 hours take in place of the deceased devisee and if they are all of the same degree of kinship to the devisee they take equally, but if of unequal degree then those of more remote degree take by representation. One who would have been a devisee under a class gift if he had survived the testator is treated as a devisee for purposes of this section whether his death occurred before or after the execution of the will.

c.       If the testator manifests an intent that the antilapse statute no apply, and he does not include an alternative gift when a devisee predeceases the testator, the common law default rule applies

d.      Allen v. Talley

Facts: Devised property to living brothers and sisters and then named them. At the time she died only two of the siblings were alive, Claude and Lera.

Procedural History: Nephew of deceased brother asked that will be admitted to probate. The trial court found that the will contained words of survivorship that precluded application of the anti-lapse statute.

Issue: Did the decedent’s will contain words of survivorship which preclude application of the anti-lapse statute?

Holding: the will contained words of survivorship that precluded application of the anti-lapse statute.

Rule: Share and share a’like and living brothers and sisters are terms of survivorship and create a class gift that precludes the anti-lapse statute.

Reasoning: Living brothers and sisters has clear meaning.

e.       § 2-603(b)(3) – Words of survivorship are not in the absence of additional evidence, a sufficient indication of an intent contrary to the application of this section.

                                                              i.      There is dispute as to this provision, that it inhibits testator intent. Ascher.

f.       Nonprobate transfers – Under the law of wills, a beneficiary is required to survive the testator in order to take. If a beneficiary does not survive, an antilapse statute may be applicable if its terms are met.

g.      Payable-on-Death designations. –

                                                              i.      Under the law of contracts, third party beneficiaries of contracts are not required to survive the benefactor or the time of performance and may pass their contract rights to their heirs or devisees.

1.      §6-212 – beneficiaries of both payable-on-death bank accounts and transfer on death brokerage accounts must survive the depositor

2.      § 2-706 if she survives me does not prevent the application of the anti-lapse statute

h.      Revocable trusts – inter vivos trusts ordinarily create vested or contingent remainders in the beneficiaries.

                                                              i.      No requirement of survivorship is implied when a remainder is created.

                                                            ii.      Statutes in a few states require the beneficiary of a revocable trust to survive the transferor and apply an antilapse statute if the beneficiary predeceases the transferor.

                                                          iii.      Joint tenancies – A joint tenant who predeceases the other joint tenant loses her interest in property. No antilapse statute applies.

i.        Jackson v. Schultz – Devised property to her and her heirs assigns and forever

                                                              i.      To A and her heirs and assigns – words of purchase

                                                            ii.      And her heirs and assigns – words of limitation

                                                          iii.      Since heirs and assigns meant that it was in fee simple and not that her heirs were also directly to inherit

j.        Holfing v. Willis

                                                              i.      It is hardly reasonable to suppose that the grantor would create a substitutionary gift and at the same time designate the assigns of the named takers to take by way of substitution.

3.      Class Gifts – Under the common law of lapse, a class gift is treated differently from a gift to individuals. If a class member predeceases the testator, the surviving members of the class divide the total gift, including a deceased member’s share.

a.       Using group names such as nieces and nephews creates class

b.      Naming a people who comprise a group may be a class based on extrinsic evidence.

c.       Restatement 3rd of Property: Wills and Other Donative Transfers §13.1 Class Gift Defined – How Created

b.      A class gift is a disposition to beneficiaries who are described by a group label and are intended to take as a group. Taking as a group means that:

1.      The membership of the class is typically not static, but is subject to fluctuation by increase or decrease until the time when a class member is entitled to distribution; and

2.      Upon distribution, the property is divided among the then-entitled class members on a fractional basis.

c.       If the terms of the disposition identify the beneficiaries only by a group label, the disposition creates a class gift, unless the language or circumstances indicate that the transferor intended the beneficiaries to take as individuals.

d.      § 13.2 Class Gift Distinguished from Disposition to Beneficiaries Taking as Individuals—How Created … (b) If the terms of the disposition identify the beneficiaries only by name, without any reference to a group label, the disposition does not create a class gift, but is to the beneficiaries taking as individuals. (c) If the terms of the disposition identify the beneficiaries (i) by a group label and (ii) either by name or by the number of beneficiaries who then fit the group label, the disposition is presumed not to create a class gift, but is to the beneficiaries taking as individuals. The presumption is rebutted if the language or circumstances  indicate that the transferor intended the beneficiaries to take as a group.

e.       Dawson v. Yucus

Facts: T died on May 29th, 1965, devised her interest in her late husband’s family farm to two nephews on her husband’s side of the family.

Procedural History: TC found that a class gift was not intended.

Issue: Was a class gift intended?

Holding: T’s will did not create a class gift and that the gift in that clause to Gene Burtle had lapsed and passed to the residue of her estate.

Rule: A gift to a class is defined as a gift of an aggregate sum to a body of persons uncertain in number at the time of the gift, to be ascertained at a future time, and who are all to take in equal or some other definite proportions, the share of each being dependant upon the ultimate number of persons. A class is a body of persons with common characteristics. If the share is certain ahead of time, it is not a class gift. If there is an intent for survivorship, then the survivor should take.

Reasoning: Testatrix had told people that she wanted only the two nephews to inherit because she had a close relationship with them. The terms “and believing as I do that these farm lands should go back to my husband’s side of the house,” together with the extrinsic evidence, clearly requires a class gift. However, there is language in the ninth section of the will that indicates that she knows how to create class gift. Also, the other beneficiaries are of her husbands side of the house.

f.       Sullivan v. Sullivan – woman devised property to 3 out of fives nieces and nephews. One nephew predeceased T, court gave his share to her best friends as a class.

g.      RE: Moss – the living out of the named beneficiaries take

h.      American Law of Property § 22.13 – The prevailing view seems to be that a gift to A and the children of B is a gift to an individual and a class in the absence of additional facts. Thus, if A dies before the testator, his share lapses and does not pass to the children of B. Likewise, if all children of B die, their share lapses. Also, the revocation of the share of A or of the share given the children of B should result in a lapse as to the revoked share.

i.        Application of antilapse statutes to gifts –Almost all states apply their anti-lapse statutes to class gifts.

Section C. Changes in Property After Execution of Will

1.       Ademption by Extinction – Property sold or disposed of before the testator dies is adeemed.

a.       Ademption does not apply to general demonstrative or residuary devises.

                                                              i.      Demonstrative devise – a general devise yet payable from a specific source

                                                            ii.      Residuary devise – conveys that portion of the testator’s estate not otherwise effectively devised by other parts of the will

                                                          iii.      Identity theory of ademption – if a specifically devised item is not in the testator’s estate, the gift is extinguished

                                                          iv.      Intent theory of ademption – the beneficiary may nonetheless be entitled to the cash value of the item, depending on whether the beneficiary can show that this is what the testator would have wanted.

b.      Wasserman v. Cohen

Facts: The d is the surviving trustee of a trust established by Freida M. Drapkin. Trustor sold apartment building promised in the trust documents before her death and never conveyed interest in conveyed her interest in the property to the trust. Trustor had a pour over will.

Procedural History: P brought an action for declaratory judgment in the Middlesex division of the probate and family court against the defendant, trustee.

Issue: Does the doctrine of ademption by extinction apply to a specific gift of real estate contained in a revocable inter vivos trust.

Holding: The doctrine of ademption as traditionally applied in wills should also apply to the trust in the instant case.

Rule: When a testator disposes during his lifetime of the subject of a specific legacy or devise in his will, that legacy or devise is held to be adeemed. “Whatever may have been the intent or motive of the testator doing so.”

Reasoning: The doctrine seeks to give effect to a testator’s probably intent by presuming he intended to extinguish a specific gift of property when he disposed of that property prior to his death. The practice of determining whether a devise is general or specific is the proper first step in deciding questions of ademption.

c.       Kelly – T contracts to sell real property to B, but dies before the closing. In her will, T leaves the property to A. After T’s death, the executor closes on the deal, as required. The devise to A adeems based on intent.

d.      Mayberry – bank account exchanged for certificates of deposit – adeemed

e.       Reddit – property exchanged for stock – change in form - not adeemed

f.       Hume – foreclosure alters form of house regardless of whether proceeds remain, ademption by extinction

g.      Jurisdictions following the identity theory –

                                                              i.      Classify the devise as general or demonstrative rather than specific

1.      Value of stock if still owned – general

2.      Kenaday - 10,000 more or less in bank book – demonstrative

                                                            ii.      Classify the inter vivos distribution as a change in form, not substance

1.      UPC § 2-605(a)(2) Most courts hold that corporate merger or reorganization is only a change in form, not substance and A takes the stock

                                                          iii.      Construe the meaning of the will as of the time of death rather than as of the time of execution

1.      Most viable when the language of the will is broad enough to cover the new item and the new item was purchased not to change the estate plan, but rather as a matter of ordinary living (also falls under acts of independent significance) – I give my car to Harold.

                                                          iv.      Create exceptions – If the conservator of an incompetent or insane person transfers the item, most cases have held the legacy not adeemed on the theory that ademption requires a voluntary act of the testator.

                                                            v.      1960 UPC exceptions UPC § 2-608 (a) & (b)

a.       Any remaining balance on the purchase price of the specific property sold

b.      Any unpaid amount of condemnation award for the property

c.       Any unpaid fire or casualty insurance proceeds after the property has been destroyed

d.      Any property owned by the testator as a result of foreclosing a mortgage devised to the specific devisee, and

e.       The sale price of specifically devised property sold by a conservator

                                                          vi.      1990 UPC § 2-606(a)(1)-(4) and (b) – abandons the identity theory and adopts the intent theory, but as amended in 1997 creates a presumption in favor of ademption. The party claiming the property has the burden of proving that the ademption is inconsistent with the testator’s intent

                                                        vii.      § 2-606 Non ademption of specific devises; unpaid proceeds of sale, condemnation, or insurance; sale by conservator or agent

a.        A specific devisee has a right to the specifically devised property in the testator’s estate at death and:

1.      Any balance of the purchase price, together with any security agreement owing from a purchaser to the testator at death by reason of the sale of the property;

2.      Any amount of condemnation award for the taking of property unpaid at death;

3.      Any proceeds unpaid at death on fire or casualty insurance or on other recovery for injury to the property

4.      Property owned by the testator at death and acquired as a result of foreclosure, or obtained in lieu of foreclosure, of the security interest for a specifically devised obligation;

5.      Real or tangible personal property owned by the testator at death which the testator acquired as a replacement for specifically devised real or tangible personal property; and

6.      If not covered by paragraphs (1) through (5) a pecuniary devise equal to the value as of its date of disposition of other specifically devised property disposed of during the testator’s lifetime but only to the extent it is established that ademption would be inconsistent with the testator’s manifested plan of distribution or that at the time the will was made, the date of disposition or otherwise, the testator did not intend that the devise adeem. (Criticized for increasing litigation).

b.      If specifically devised property is sold or mortaged by a conservator or by an agent acting within the authority of a durable power of attorney for an incapacitated principal, or if a condemnation award, insurance proceeds, or recovery for injury to the property are paid to a conservator or to an agent acting within the authority of a durable power of attorney for an incapacitated principal, the specific devisee has the right to a general pecuniary devise equal to the net sale price, the amount of the unpaid loan, the condemnation award, the insurance proceeds, or the recovery

c.       The right of a specific devisee under subsection (b) is reduced by any right the devisee had under subsection (a)

2.      Stock Splits and the Problem of Increase

a.       Many modern courts find that absent a contrary showing of intent a devisee of stock is entitled to additional shares received by the testator as a result of a stock split

b.      §UPC 2-605 – stock dividends are treated the same as stock splits: the beneficiary gets them along with the other shares

3.      Satisfaction of General Pecuniary Bequests

a.       If the testator is a parent of the beneficiary and after execution of the will transfers the beneficiary property of a similar nature to that give by the will, there is a rebuttable presumption that the gift is in satisfaction of the gift made by the will

                                                              i.      Some states require that an attempt to adeem by satisfaction evidence the testator’s intent in writing

4.      Exoneration of Liens (some states)

a.       Other assets will be used to pay off liens so that the devisee takes the property free and clear

b.      § 2-607 – A specific devise passes subject to any mortgage interest existing at the date of the death without right of exoneration, regardless of a general directive in the will to pay debts.

5.      Abatement

a.       When the testate has insufficient assets to pay debts as well as all the devises; some devises must be abated or reduced

                                                              i.      Residuary devises are reduced first

1.      Sometimes the residuary devisee is the most important in the case of large estates and this may leave them without any property. Is this fair?

2.      It is advisable to make substantial devises as shares of the residue

                                                            ii.      General devises are reduced second

                                                          iii.      Specific and demonstrative devises are the last to abate and are reduced pro rata


Chapter 7

Restrictions on the Power of the Disposition: Protection of the Spouse and Children

Section A. Rights of the Surviving Spouse

1.      Introduction to Marital Property Systems

a.       Separate Property – husband and wife own separately all property each acquires, except those items one spouse has agreed to put into joint ownership with the other

b.      Community Property – husband and wife own all acquisitions from earnings after marriage in equal undivided shares

                                                              i.      The community dissolves at death

                                                            ii.      The deceased spouse owns and has testamentary power over only his or her one-half community share

·         Their half by statute goes to you

2.      Rights of Surviving Spouse to Support

a.       Social Security – retirement benefits are paid to a worker and his or her surviving spouse

                                                              i.      Eligibility

1.      Before 1938 – age 65

2.      1943 and 1954 – age 66

3.      After 1960 – age 67

b.      Private Pension Plans

                                                              i.      Governed by ERISA

1.      Defined contribution plan – funded by contributions from both the employer and the employee

a.       At retirement, the employee is entitled to the assets in the fund held in her name

2.      Defined benefit plan – Funded by the employer, at retirement the employee is entitled to a defined benefit. Say 40% of their last years income.

a.       ERISA requires that the spouse of an employee must have survivorship rights if the employee predeceases the spouse (ERISA preempts any contradictory state law)

b.      Waiver – a spouse may waive her rights to benefits under the employee’s pension plan, but ERISA discourages waivers by strict rules regarding the validity. Premarital agreements cannot waive such rights.

c.       Homestead – Nearly all states have homestead laws designed to secure the family home to the surviving spouse and minor children, free of the claims of creditors. (aka probate homestead).

                                                              i.      Spouse has the right to occupy the family home for his or her lifetime.

                                                            ii.      Some states only a small amount of money is set aside like $15,000.

d.      § UPC 2-403 Personal Property Set-Aside – The right of the surviving spouse and sometimes minor children to have set aside a certain tangible personal property of the decedent up to a certain value. UPC sets the limit at $10,000. May include furniture, clothing, car and farm animals.

e.       Family Allowance – Every state has a statute authorizing the probate court to award a family allowance for maintenance and support of the surviving spouse (and often of dependent children).

                                                              i.      Some states have a statutory amout

                                                            ii.      § UPC §2-404 allows a reasonable allowance

                                                          iii.      Australia, New Zealand and Canada allow a larger amount to be distributed if the court deems it necessary after the closing of the will, the US does not

f.       Dower and Curttesy

                                                              i.      Dower - A life-estate in 1/3 of her husband’s qualifying land (MI (must elect to take dower rather than statutory share, Ohio, Ark, Kentucky)

1.      Today both spouses have to sign to sell land

                                                            ii.      Curtesy – a husband had a support interest in his wife’s lands

1.      Not acquired unless children were born of the marriage

2.      The husband was given a life estate in the entire parcel, not just 1/3

3.      Rights of Surviving Spouse to a Share of Decedent’s Property (Molto statutory variation!!!)

a.       The Elective Share and Its Rationale

                                                              i.      Partnership – property passes to taking spouses heirs because spouses were partners in accumulating the marital assets

                                                            ii.      Support – not needed after death of taking spouse

                                                                iii.      All but one of the separate property states give the surviving spouse, in addition to any support rights mentioned above, an elective share of the decedent’s property.

1.      The spouse can take under the decedent’s will or

2.      The spouse can renounce the will and take a fractional share of the decedent’s estate.

                                                          iv.      UPC Article II Part 2 Elective Share of Surviving Spouse (page 426 in book) General Comment, The Partnership Theory of Marriage

1.      Presumes intent of husbands and wives to pool their fortunes on an equal basis.

2.      Compensation for non monetary contributions to the marriage

3.      Elective share law and equitable distribution upon divorce in separate property systems also give similar compensation

4.      UPC gives a sliding scale accrual method

                                                            v.      The Estate Tax Marital Deduction and The Dependency of Women

1.      Now interspousal transfers are not taxed

2.      The following transfers qualify for the marital deduction

a.       H transfers property outright or in fee simple to W

b.      H creates a trust giving W  income for life and power to appoint the trust principal at death to whomever she pleases

c.       H creates a trust giving W income for life (a QTIP trust).

b.       In re Estate of Cross

Facts: T died leaving estate to son who was not a child of his surviving spouse. Spouse was in a nursing home paid for by Medicaid. She could not choose to elect. Court chose to elect.

Procedural History: Probate court found that she should receive a 25 K spousal allowance and half of the net estate $9 K. Court of Appeals found that it was not necessary for her to take because she was already taken care of by Medicaid. Administrator of her will appealed and county board of administrators also appealed.

Issue: If T left entire estate to his son, should wife take if she is wholly dependent on Medicaid benefits?

Holding: Wife should take from the will.

Rule: Where a surviving spouse is under legal disability, the probate court is given the authority under RC 2106.08 to appoint a suitable person to ascertain the surviving spouses adequate support needs and to compare the value of the surving spouses rights under the will with value of her rights under the statute of descent and distribution. The court may elect for the surving spouse to take against the will and only if it finds after taking into consideration the other available resources and the age, probably life expectancy, physical and metnal condition and present reasonably anticipated future needs of the surviving spouse, that the election to take is necessary to provide adequate support for the surviving spouse during his life expectancy.

Reasoning: Nonutilization of income available upon request constitutes ineligibility.

c.       Faller – Trust provisions specifying that beneficiary cannot take any amount that would make them inelligible for benefits is in violation of CO statutes

d.      Majority rule – the right to an elective share is personal to the surviving spouse and that creditors of the surviving spouse cannot force her to elect her share

e.       Majority view – allows the guardian to take into account the preservation of the decedent’s estate plan and whether the surviving spouse would have wanted to abide by her dead spouse’s will

f.       1969 UPC § 2-203 – probate court acting for an incompetent could order election against the spouses will only after finding that exercise is necessary to provide adequate support for the protected person during his probably life expectancy.

g.      1990 UPC § 2-212 – if a conservator or guardian elects the electives share, the portion of the elective share that exceeds what the decedent spouse provided for the survivor must be placed in a custodial trust for the benefit of the surviving spouse.

                                                              i.      The trustee of such a trust, appointed by the court, has the power to expend income and principal for the surviving spouse’s support, and upon the spouse’s death the trustee must transfer the trust property to the residuary devisees under the will of the predeceased spouse against whom the elective share was taken or to the predeceased spouse’s heir

                                                            ii.      Bilse - If spouses own estate is enough to cover expenses, cannot collect forced share

                                                          iii.      In some states the elective share is denied to a spouse who has abandons or refused to support the other spouse, but not in most states

h.      In most community property states if the couple separates, the earnings of both spouses continue to be property until divorce. In CA, earnings acquired after separation are not community property.

i.        The failure of a lawyer to warn the client about the elective share, which would dismantle the client’s estate plan, can be grounds for a malpractice action

j.        In re Estate of Cooper

                                                              i.      Procedural History: Appeal from a decree denying the right of a spouse’s election to the surviving member of a same-sex relationship.

                                                            ii.      Facts: After Cooper’s death, Chin his current same sex lover petitioned the court to elect to take a statutory share of Cooper’s estate as surviving spouse rather than take his share under the will.

                                                          iii.      Rule: The survivor of a homosexual relationship alleged to be a spousal relationship is not entitled to exercise a right of election against the decedent’s will.

                                                          iv.      Reasoning: Court said that homosexuals have to constitutional right to marriage

                                                            v.      Note: HI said state has to an interest in not allowing homosexual

k.      Defense of Marriage Act 1996 – Domestic partners get no federal benefits

l.        States with elective share rights for same-sex domestic partners:

                                                              i.      HI reciprocal benefits

                                                            ii.      Vermont :Civil Unions

                                                          iii.      California: domestic partners

m.    Property Subject to Elective Share

                                                              i.      Surviving spouse usually gets a 1/3 fractional share of probate assets

                                                            ii.      Non probate assets

1.      Sullivan v. Burkin

a.       Procedural History: Appeal from a dismissal of a complaint for determination of estate assets

b.      Sullivan contended that the value of real estate in trust by her late husband should be considered part of his estate for purposes of providing her a portion of the estate

c.       Rule: The surviving spouse has no claim against the assets of a valid inter vivos trust created by the deceased spouse even when the deceased spouse allowed retained substantial rights and powers under the trust instrument

d.      Note: Court stated that this rule only applied to this case and past cases. From that day forward, revocable inter vivos trusts would be treated as part of the estate.

e.       Notes:

                                                                                                                                      i.      Newman: illusory transfer test, wife had share of revocable inter vivos trust, trust is not invalid, but counts as part of the estate assets

                                                                                                                                    ii.      Intent to defraud – whether decedent intended to defraud surviving spouse

                                                                                                                                  iii.      Present donative intent – did transferor intend to make a gift of the property

                                                                                                                                  iv.      OH and CN- spouse not entitled to share of revocable trust

                                                                                                                                    v.      UPC §2-202 – the law of the decedent’s domicile shall govern the right to take an elective share of property located in another state

2.      Statutory Schemes

a.       NY

                                                                                                                                      i.      Suriving spouse 50K or 1/3 of the decedent’s estate, plus a personal property set aside

1.      Including

a.       Gifts causa mortis

b.      Gifts made within one year before death, except those not exceeding 11K

c.       Savings account

d.      Joint bank accounts (deceden’t contribution)

e.       Joint tenancies

f.       Property payable on death

g.      Lifetime transfers

h.      Pension plans

i.        Any property over which the decedent had a general power of appointment

2.      Subtract the value of any interest other than a life estatem which passes form the decedent to the surviving spouse by intestacy, by will  or by will substitute

b.      DE

                                                                                                                                      i.      Elective share as all property includible in the decedent’s gross estate under the federal estate tax, whether a tax return is filed

c.       Uniform Probate Code 1969

                                                                                                                                      i.      Augmented estate – probate estate augmented by certain nonprobate transfers

                                                                                                                                    ii.      UPC §2-202: The surviving spouse is entitled to an elective share of 1/3 of the augmented estate.

1.      Any transfer under which the decedent retains the right to possession or  income from the property

2.      Any transfer which the d can revoke or invade or dispose if the principal for his own benefit

3.      Any transfer in joint tenancy with someone other than the spouse

4.      Any transfer made within two years before death exceeding 11K

5.      Property given to the surviving spouse during life, including a life estate in a trust, and property received by the spouse at death derived from the decedent, such as life insurance and pensions

                                                                                                                                  iii.      Probate Code 1990

1.      Elective share percentage based on length of marriage 50% for 15 years of marriage (includes property brought to the marriage, includes life insurance)

a.       Unlike community property (only earned income)

2.      Determine value of augmented estate

3.      Determine elective share amount

d.      Life Estate

                                                                                                                                      i.      Surviving spouse is not charged for the life estate against her elective share,

e.       Waiver – Spouses can agree to waive elective shares

                                                                                                                                      i.      Some courts say against public policy

                                                                                                                                    ii.      Unfair bargaining position for poorer spouse

                                                                                                                                  iii.      Should have separate attorneys.

f.       UPC § 2-213 1990 amended 1993

                                                                                                                                      i.      Waiver may be created

1.      Not unconscionable

2.      Reasonable disclosure

3.      In re Estate of Garbade

a.       Appeal from decision to set aside notice of surviving spouse’s election

b.      Facts: Decedent’s wife claimed that the prenuptial agreement she signed should be invalidated because of the surrounding circumstances

c.       Rule: A duly executed prenuptial agreement is given the same presumption of validity as any other contract in the absence of fraud

4.      In re Grieff: the contestant of a prenumptial agreement must establish a fact-based particularized inequality before a proponent of a prenuptial agreement suffers a shift in the burden to disprove fraud or overreaching

4.      Rights of Surviving Spouse in Community Property

a.       Basic Information

                                                              i.      LA, TX, NM, AR, CA, NE, WA and ID, WI community property states

                                                            ii.      AL allows for an election of community property

b.      Community property - H and W own the earnings and acquisitions from earnings of both spouses during marriage in undivided equal shares

                                                              i.      Stepped up basis for determining tax at death

                                                            ii.      In CA, husband and wife have power to manage community property separately

                                                          iii.      People not living in a community property state can place property in an Alaska Community Property Trust

1.      The surviving spouse will this have to pay no tax on capital gain incurred before the decedent’s death if she shells the property. Not for sure yet. IRS has made no ruling.

2.      If H makes a gift to someone other than spouse in CA, spouse is entitled to ½

3.      Spouses own equal shares in each item of community property at death

4.      They do not own equal undivided shares in the aggregate of community property

5.      Upon divorce items will be divided based on value, not half shares in each item

c.       Separate property is property acquired before marriage and property acquired during marriage by gift or inheritance

                                                              i.      Only decedent’s interest in property get stepped up basis

d.      Putting the Survivor to an Elections

                                                              i.      “Widow’s election” – electing all community property in trust to pay the income to his wife for life, with the remainder to others on the wife’s death, and requiring the wife to elect between surrendering her half and taking under the will

                                                            ii.      A better alterative to a forced widow’s election is a plan by husband and wife to transfer all the community property into a revocable trust, paying income to husband and wife for their joint lives and for the life of the survivor

5.      Migrating Couples and Multistate Property Holdings

a.       The law of the situs controls problems related to land

b.      The law of marital domicile at the time that personal property is acquired controls the characterization of the property (that is, as separate or community).

c.       The law of the marital domicile at the death of one spouse controls the survivor’s marital rights

                                                              i.      §2-202(d) provides that the rights of a spouse to an elective share in land located in the state shall be governed by the law of the decedent’s domicile at death.

d.      Moving from a separate property state to a community property state

                                                              i.      The ownership of movable property is determined by the laws of the state where the couple is domiciled when the property is acquired.

                                                            ii.      As a result of the move, the wife loses protection of the elective share system provided by the state where the moveable property was acquired and is not protected by the system of community property whichs he would have if the couple had been domiciled in the community property state when the husband was working.

                                                          iii.      Quasi-community property – property owned by the husband or the wife acquired while domiciled elsewhere which would have been community is considered CP

                                                          iv.      §Cal. Prob. Code §102. Spouse has a right to ½ of any nonprobate transfer of quasi-community property where the decedent retained possession.

e.       Moving from a Community Property State to a Separate Property State

                                                              i.      Disposition of Community Property Rights at Death - Community property continues to be community property unless spouses agree to convert it into testamentary disposition

1.      Should have written documentation that new property purchased with community property assets is to remain community property

2.      Some states have community property with rights of survivorship to avoid probate

3.      Cal Prob Code §13500 – when property passes at death to the decedent’s spouse no admin is necessary unless the surviving spouse elects administration, CA wanted to ensure that it would qualify for the 1014 stepped up basis (not to be confused with joint tenancy)

6.      Spouse Omitted from Premarital Will

a.       Estate of Shannon

Facts: When Russell Shannon died, after marrying Lila and without changing the will he had executed 12 years b4 the marriage, L argued that she should be entitled to estate distribution as an omitted surviving spouse

Rule: Cal Prob 6516: If a testator fails to provide by will for a surviving spouse who married the testator after the execution of the will, the omitted spouse shall receive a statutorily prescribed share of the estate, unless he intentionally disinherited, provided for spouse outside of will or prenup.

Reasoning: The face of the will did not manifest any intention to disinherit L. There also was no provision for L outside of the will. There is strong public policy in favor of providing for spouse.

b.      UPC § 2-301 Entitlement of Spouse; Premarital Will

                                                              i.      (a) If a testator’s surviving spouse married the testator after the testator executed his will, the surviving psouse is entitled to receive, as an intestate share, no less than the value of the share of the estate he or she would have received if the testator had died intestate as to that protion of the testator’s estate, if any, that is neither devised to a child of the testator who was born before the testator married the surviving spouse and who is not a child of the surviving spouse no devised to a descendant of such a child or passes under sections 2-603 or 2-604 to such a child or to a descendant of such a child, unless:

1.      It appears from the will or other evidence that the will was made in contemplation of the testator’s marriage to the spouse

2.      The will expresses that it remains in effect after subsequent marriage

3.      The T provides for the spouse by transfer outside the will and the intent that the transfer be in lieu of a testamentary provision is shown by either the T’s state mets or is reasonably inferred from the amoubt of the transfer or other evidence

                                                            ii.      (b) In satisfying the share provided by this section, devises made by the will to the testator’s surviving spouse, if any are applied first and other devises other than a devise to a child of the testator who was born before the testator married the surviving spouse and who is not a child of the surviving spouse or a devise or substitute gift under §2-603 or 2-604 to a descendant of such a child, abate.

Section B. Rights of Issue Omitted From the Will

1.      Protection from Intentional Omission

a.       The Domestic Approach

                                                              i.      In all states except LA, a child or other descendant has no statutory protection against intentional disinheritance by a parent. There is no requirement that a testator leave any property to a child.

                                                            ii.      Disinheritance of children is almost always risky, unless there is a surviving spouse

1.      American rule of allowing children to be disinherited increased will contests

2.      LA –forced share for children under 23, mentally infirm and disabled.

a.       Unless child has (before the will was executed)

                                                                                                                                      i.      Hit parent

                                                                                                                                    ii.      Guilty of crime

                                                                                                                                  iii.      Married without consent as a minor

                                                                                                                                  iv.      Failed to communicate with parent for more than two years

b.      A Look Abroad: Family Maintenance Statutes

                                                              i.      Right to maintenance in an amount determined by the court if decedent supported them while he was alive

c.       Lambeff – Australia applied family maintenance system gave inheritance to son with less income and children.

2.      Protection from Unintentional Omission

a.       Azcunce v. Azcunce

Facts: When Azcince died shortly after executing a second codicil to his will, republishing the original will, P who had been born prior to execution of the second codicil, petitioned the court for a share of her father’s estate as a pretermitted child.

Rule: When a testator fails to provide in his will for any child born after making the will, the child shall receive a share of the testate equal in value to that he would have received if the testator had died intestate, unless it appears the omission was intentional.

Holding: She did not inherit because the codicil republished and if her father had wanted her to inherit, he would have included her when he republished.

Note: She could not bring a malpractice suit against the attorney because she did not have privity of contract under Fla ruling. However, most states say that attorney has a duty to inform the beneficiary.

Note: Guardians ad litem would not share minor children’s share of the estate because they did not want to be sued for malpractice because they have a duty to those children.

b.      UPC §2-302 Omitted Children

                                                              i.      Except as provided in subsection(b) if a testator fails to provide in his will for any of his or her children born or adopted after the execution of the will, the omitted after-born or after-adopted child receives a share in the estate as follows:

1.      If the testator had no child living when he or she executed the will, an omitted after-born or after-adopted child receives a share in the estate equal in value to that which the child would have received had the testator died intestate, unless the will devised all or substantially all of the testate to the other parent of the imitted chuld that other parent survives the testator and is entitled to take under the will.

c.       In re Estate of Laura

                                                              i.      Facts: Laura executed a will that intentionally omitted and disinherited his son and grandchildren. The grandchildren’s issue claimed they were entitled to a share of the estate

                                                            ii.      Rule: Testators who specifically name an heir in an effort to disinherit has referred to the issue of that heir for that purpose as well.

                                                          iii.      Reasoning: This effects testator intent.

d.      Estate of Treloar

                                                              i.      Because neither mother or father were specifically mentioned in the will (son-in-law named executor), grandchildren were found to be pretermitted heirs and were allowed to take.

In Estate of Katangian

Cal.App. 4 Dist.

Cunningham Judge

Facts: Decedent was married to appellant Vickie. They had two children. Eric born in 1948 and Debra born in 1952. Eric is severely retarded. He has been in a state institution since 1954. Vicki is conservator of Eric. In 1957, Rachel and Vicki divorced. In 1956, decedent and wife, Vicki, entered into a marital settlement agreement, pursuant to which Raphael paid $20 a month to the state of California for Eric’s support until Eric turned 18. Decedent died in 2001. His will executed in 1996 stated that Debra was his only child; it named her as his executor, and it left his entire estate to her and her two daughters. Vicki testified that decedent went to see Eric two or three times year at the end of his life.

Issues: Is Eric entitled to a share of the estate as an omitted heir. Is Eric entitled to a family allowance?

Procedural History: The trial court denied both claims.

Holding: Eric is not entitled to a share of the estate as an omitted heir and he is not entitled to a family allowance.

Rules:

Probate Code § 21620: Except as provided in §21621, if a decedent fails to provide in a testamentary instrument for a child of decedent born or adopted after the execution of all of the decedent’s testamentary instruments, the omitted child shall receive a share in the decedents estate equal in value to that which the child would have received if the decedent had died without having executed any testamentary instrument.

Probate Code §21621: A child shall not receive a share of the estate under §21620 if any of the following is established: The decedent’s failure to provide for the child in the decedent’s testamentary instruments was intentional and that intention appears from the testamentary instruments.

Relevant to Problem, Probate Code § 21622: If, at the time of the execution of all of the decedent’s testamentary instruments effective at the time of the decedent’s death, the decedent failed to provide for a living child solely because the decedent believed the child to be dead or was unaware of the birth of the child, the child shall receive a share in the estate equal in value to that which the child would have received if the decedent had died without having executed any testamentary instruments.

Burden of Proof: With respect to a child born after the making of the will, the burden of proving that the decedent intended to omit the child still is on the party opposing the child’s claim, and still cannot be met with extrinsic evidence. Relevant to Problem, But with respect to a child born before the making of the will, the burden of proving that the decedent did not intend to omit the child because the decedent though the child was dead, or was unaware of the child’s birth is on the child.

Reasoning: There was insufficient evidence that the decedent thought that his son Eric was dead. There is evidence that he knew he was alive. Decedent provided for him in the divorce settlement and visited him infrequently.

Allowance Reasoning: Family allowances are for support during probate proceedings. Adult children are entitled to a family allowance if but only if they were actually dependent in whole or in part upon the decedent for support.

Boetteger v. Roberts

Facts: Child was born to P and W, who were married and cohabitating when child was conceived and born. W was not impotent or sterile. P had sexual relations with both W and M in June 1996. Heidi’s blood test showed that W was not her father. In March 1995, P sent child’s orthodontia bill to M; he did not pay it. P did not file for paternity tests within the first two years of child’s life. Child’s birth certificate names W as her father. W did not attend child’s school events except middle school graduation. Alleged biological father married C on July 27th, 1996. October 4th, 1997, alleged biological father executed a will. He gave property to C and stated, “ I have specifically and intentionally omitted any other heirs to my estate whether known or unknown from this my Last Will and Testament.” Alleged biological father died June 5th, 2000.

Issue: Is child’s alleged biological father, not her father because her mother’s husband is conclusively presumed to be her father? Is she due a share of her alleged biological father’s estate?

Procedural History: In paternity action, wife filed motion for summary judgment on issue of whether conclusive presumption of paternity applied such that deceased husband was not child’s father. In probate action, wife filed motion to dismiss child’s motion to vacate spousal property order and petition for determination of entitlement to distribution of husband’s estate. Superior court granted wife’s motions. Court of Appeal held that conclusive presumption of paternity applied, and child failed to establish entitlement to distribution of husband’s estate as pretermitted heir.

Holding: Her alleged biological father is not her father because her mother’s husband is conclusively presumed to be her father and therefore, she is not due a share of her alleged biological father’s estate.

Rule: Conclusive presumption of paternity applied such that mother’s husband was child’s father, where child was born while mother and father were married and cohabitating, father was not impotent or sterile, and blood tests of putative father were neither court-ordered nor noticed within two years of child’s birth.

Rule #2: Child could not establish parent-child relationship with decedent, and this, child was not entitled to distribution of decedent’s estate as pretermitted heir, where mother’s husband was child’s conclusively presumed father and no order establishing decedent’s paternity was entered during decedent’s lifetime.

Probate Code 6453: A child can have only one natural father for the purposes of inheritance as a pretermitted heir. If the child has a presumed father, that father is the natural father.

Reasoning: Since, child’s mother was living with and married to W when child was born and no blood tests were sought from M during the two years after child’s birth, W is presumed to be her father for the purposes of family court. Therefore, M cannot be her father for the purposes of inheritance as a pretermitted heir. However, even if W were not her natural father, she would no be able to inherit from M. M did not hold her out as his own. Child did not allege that M had been unable to acknowledge her before his death. Child does not meet any of the requirements listed in the statutes. Child’s interests do not outweigh the states interests because the father is dead and there is no possibility of the child and alleged father maintaining a relationship. Her only interest would be solely financial.

Sala v. Flanagans Boys Home

Court of Appeal, Third District, CA

1999

Facts: Testator kept the existence of his children and grandchildren a secret from his fiduciary. Testator and his wife had received a letter from petitioner’s wife with pictures of his grandchildren in 1965. In 1971, petitioner flew to MA and spent week with petitioner and family. They had no visits after that, but continued to exchange gifts and cards at Christmas. Petitioner stated that there would be no reason why testator would think him dead.

Procedural History:  Testator’s son from a first marriage who the testator had pretermitted filed a petition asserting entitlement to the proceeds of the estate under the omitted child statute. The superior court entered judgment against the son, and he appealed. The court of appeal held that: (1) the son had the burden of proving that the sole reason for his pretermission was the testator’s mistaken belief that he was dead, and (2) evidence that the testator had misled his executrix into believing that he had no living relatives did not establish that he thought that his son was dead (what???)

Rule: Pretermitted heir asserting entitlement to the proceeds of the testator’s estate under the omitted child statute had the burden of proving that the sole reason for his pretermission was the testator’s mistaken belief that the heir was dead.

§ 21622: If at the time of the execution of all of decedent’s testamentary instruments effective at the time of decedent’s death, the decedent failed to provide for a living child solely because the decedent believed the child to be dead or was unaware of the birth of the child, the child shall receive a share in the estate equal in value to that which the child would have received if the decedent had died without having executed any testamentary instruments.

Reasoning: The right to dispose of property in contemplation of death is as old as the right to acquire and possess property and the laws of all civilized countries recognize and protect this right.  In the omitted children statutes, the Legislature has attempted to balance the possibility of inadvertent disinheritance against the freedom of testamentary disposition of property with respect to the paramount concern of carrying out the testator’s intent. Under 21622, petitioner needed to prove that testator thought he was dead. Due to evidence of their contact from 1965 to the 1980’s the evidence shows that t did not think p was dead.

Estate of Breeden

Facts: When Breeden died, his will designated the bulk of his estate to be placed in trust and used to promote the principles of socialism and related causes. He willed the residue of his estate to the Breeden-Schmidt Foundation with Milton Lessner and Willard Sinclair as co-trustees. Only Lesner survived the decedent. It shall include but not be limited to, subsidizing publications, establishing and conducting reading rooms, supporting radio, television and the newspaper media and candidates for public office. After Lesner started the foundation, two attorneys Katz and Gostin were selected to serve as co-trustees. Breeden had established a family allowance trust for his step=daughter, Lasca Tooles, with a $150,000 corpus to be administered by Sinclair. Lasca was to receive $1200 per month. Upon her death remaining trust funds were to be added to the main fund. He also left $7,000 to both his niece and nephew. He also left $1000 to the San Diego Bible College and the General Conference of Seventh-Day Adventists. It also stated that it is not intended to be a charitable trust, although the trustees may in the future if they unanimously determine apply for such designation and tax status. They can use the funds for non charitable purposes unless the apply for charitable status.

Procedural History: Nephew and niece of decedent filed a petition to have trust provision of uncle’s will declared invalid and estate residue given to them as intestate heirs. SC denied petition. Nephew and niece appealed.

Issue: Was the trust a charitable trust?

Holding: Court of Appeal held that testamentary trust whose purpose was to advance principles of socialism and related causes was a valid charitable trust, notwithstanding fact that it might not qualify as “tax-exempt”.

Rule: A bequest is charitable where it is made for a charitable purpose, the aims and accomplishments of which are religious, educational, political or in mankind’s general social interests, and the ultimate recipient is either the whole or an unascertainable part of the community Including relief of poverty; the advancement of education, the advancement of religion; the promotion of health; governmental or municipal purposes; other purposes the accomplishment of which is beneficial to the community. In case of doubt the trust will be construed as charitable. (unless extrinsic evidence is in conflict).

Reasoning: Charitable intent which characterizes the motivating purpose of a trust so as to identify a trust as charitable and the specific use of trust funds in a manner which may not qualify the trust for tax-exempt status. Breeden did not want to restrict the trustee from making donations outside those allowed for tax-exempt organizations.

 


 

Chapter 8 – Trusts: Creation and Characteristics

Section A. Introduction

1.      Background

a.       Trust – generally a device whereby a trustee manages property as a fiduciary for one or more beneficiaries. Better than life estates because they have equitable rather than legal interests.

                                                                                      i.      Trustee holds legal title to the property

                                                                                    ii.      Trustee can sell the property and replace it with more desirable property

                                                                                  iii.      The beneficiaries hold equitable title

                                                                                  iv.      Entitled to payments from the trust income and sometimes from the trust corpus

b.      Private express trust – created gratuitously for the benefit of individual beneficiaries.

                                                                                      i.      Revocable trust – O declares herself trustee of property to pay the income to O for life, then on O’s death to pay the principal to O’s children. O retains power to revoke the trust. A revocable trust avoids the delays, costs and publicity of probate.

                                                                                    ii.      Testamentary marital trust – the federal estate tax law permits a marital deduction for property give to the surviving spouse. The deduction is allowed for a life estate give to the spouse. To get the deduction, H devises property to X in trust to pay the income to W for her life and  on her death to pay the principal to H’s children

                                                                                  iii.      Trust for incompetent person – O’s son A is mentally or physically impaired and is unable to manage his property. O transfers property to X in trust to pay the income to A for life, remainder to A’s issue and if A dies without issue to his sister B.

                                                                                  iv.      Trust for minor – The federal gift tax allows a tax-free gift of $1000 per year to a donee. A gift to a minor creates special problems inasmuch as the minor is legally unable to manage her property. To permit annual tax-free gifts of $11,000 to his minor daughter A, O creates a trust to use the income and principal for the benefit of A before she reaches 21, and to pay A the principal when she reaches 21. Every year O can make a gift of $11,000 into the trust.

                                                                                    v.      Discretionary trust –

1.       T devises property to X in trust. The trust provides that the trustee is its sole and absolute discretion may pay the income or principal to A or for A’s benefit, as the trustee may see fit.

2.      Or the trustee may be given discretion to pay income to any one or more of a class of persons such as A and her issue.

3.      Discretionary trusts are useful in lessening the tax burden on family wealth by distributing income to the members of the family in the lowest tax bracket.

4.      Discretionary trusts are also useful in preventing creditors of the beneficiary—including ex-spouses with alimony or child support judgments, IRS, Medicaid from reaching the principal of the trust.

5.      And more

6.      NOTE: Foreign Trust Law – many cultures have something akin to a trust

2.      The Parties to a Trust –  A trust ordinarily involves three parties, but one person can wear all three hats

a.       The settlor (trustor) – the person who creates a trust, must owe equitable duties to someone else, if the settlor is also trustee and beneficiary (i.e. A must inherit trust on O’s death)

                                                                                      i.      Inter vivos trust – a trust created during the person’s life

1.      Can be created by declaration (often used as a will substitute) – settlor declares that she holds certain property in trust and is trustee, or by

a.       Does not require delivery

b.      Must manifest intention to hold property in trust

                                                                                                                                                              i.      May be oral, but

                                                                                                                                                            ii.      If it is to be funded by real property, the trust must be in writing

                                                                                    ii.      Deed of trust (when not inter vivos) – settlor transfers property to another person as trustee

1.      Deed of trust and property must be delivered to the trustee

                                                                                  iii.      Testamentary trust – a trust created by will, cannot be created by the settlor because the settlor will be dead

b.      The Trustee

                                                                                      i.      May be one or more persons, may be a corporation, banks often carry out the duties of trustee

1.      Amateur

a.       Closer family ties

b.      Knows the settlors wishes betteer

2.      Bank, professional, institutions

a.       Familiar with portfolio management

b.      More likely to be able to afford liability

c.       Bureaucracy provides safeguards for beneficiaries

d.      Demands a sizeable commission

e.       Unresponsive inflexible

                                                                                    ii.      May be settlor, third party or beneficiary (H creates trust for W for life, then, H’s children may inherit the property outside of trust)

                                                                                  iii.      RS 3rd of Trusts §31, 34 court will appoint a trustee if settlor does not name one or the trustee dies (unless the trust was only for the life of trustee), usually the executor

                                                                                  iv.      Must accept the position because they will be held liable by beneficiaries or court

1.      Uniform Trust Code § 705 allows trustee to resign with 30 days notice

                                                                                    v.      Held to a fiduciary standard of conduct

1.      Loyalty

a.       Soley in the interest of the beneficiaries

b.      Self-dealing is prohibited, wherein the trustee acts in the same transaction both in its fiduciary capacity and in an individual capacity

2.      Prudence

a.       Held to an objective standard of care

3.      Subsidiary rules that enforce the above

a.       The duty of impartiality between classes of beneficiaries such as the income of beneficiaries (who are interested in high yields) and the remaindermen (who are concerned about preservation of principal and appreciation in values)

b.      Keep trust property separate from your own property

c.       Duty to inform and account to beneficiaries

d.      Dry trust – beneficiaries inherit the property because the trustee has no duties to perform RS 3rd Trusts §6

                                                                                  vi.      Payment of trustee

1.      Amount in some states set by statute, specified by percentage of the corpus

2.      CA – reasonable compensation standard

3.      Corporations usually have their own fee schedules

                                                                                vii.      Duties

1.      Investment

a.       Initial selection of securities

b.      Monitoring investments

c.       Investing new funds

d.      Voting the shares

2.      Administration

a.       Accounting

b.      Reporting

c.       Filing taxes

d.      Maintaining real estate

e.       Insuring etc.

3.      Distribution

a.       Decide how to comply with trust

b.      View beneficiary needs

c.       Beneficiaries – hold equitable interests

                                                                                      i.      Remedies

1.      The beneficiaries have a personal claim against the trustee for breach of trust, not higher than that of a creditor

a.       Personal creditors of the trustee cannot reach the trust

b.      If trustee sold trust property and purchased new property the beneficiaries can impose the trust on the new property

c.       Often successive beneficiaries have interests, O also has a reversion interest

3.      A Trust Compared with a Legal Life Estate

a.       A legal life tenant has possession and control of the property

                                                                                      i.      To A for life, remainder to A’s children

                                                                                    ii.      Sale

1.      The legal life tenant has no power to sell a fee simple unless such a power is granted in the instrument.

2.      Otherwise life tenant and all remaindermen must agree to sell.

                                                                                  iii.      Reinvestment of proceeds of sale

1.      If power to sell is given, there are serious tax disadvantages

a.       Gives them power of appointment which makes the property includable in the person’s taxable gross estate at death, on which a federal estate tax may be payable

                                                                                  iv.      Borrowing Money

1.      Real estate cannot be mortgaged by the life tenant

2.      All remaindermen would have to agree

3.      Creates a general power of appointment (see above)

                                                                                    v.      Leasing

1.      Would not be rentable beyond the person’s life

2.      Also, creates a general power of appointment

                                                                                  vi.      Waste

1.      Remaindermen are entitled to an injunction or damages against waste (such as removing natural resources from the land, oil, timber, minerals etc.)

                                                                                vii.      Expenses

1.      Someone must pay taxes and maintain the property, but only to the extent the income from the property covers the expenses

2.      Must pay interest on, but not the principal of the mortgage

                                                                              viii.      Creditors

1.      Creditors can seize the life estate and sell it, if the life debtor lives long enough, the creditor reaps a windfall

2.      If the debtor is a remainderman, the creditor may seize the property and sell it

                                                                                  ix.      Personally verus realty

1.      Hard to protect remaindermen from waste when the property is personal property such as stocks; easily sold etc.

b.      A trustee has legal title to the trust property

                                                                                      i.      To X in trust for A for life, remainder to A’s children

                                                                                    ii.      Power of sale

                                                                                  iii.      Power to lease

                                                                                  iv.      Power to give oil

                                                                                    v.      Pay taxes, insurance and other charges

                                                                                  vi.      Put out of reach of creditors

4.      Commercial Uses of the Trust

a.       29 states have codified the business trust, creating the statutory business trust

b.      The trust today is a preferred form of organization for mutual funds and for structured finance transactions such as asset securization.

                                                                                      i.      Mutual fund

1.      Managed by a trustee (professional fund manager) for multiple people who each have shares relative to their investment

2.      Allows them to pool assets and achieve more diversification

c.       Federal law imposes a mandatory trust form on employee pension funds

                                                                                      i.      Until the EE’s retirement, then pension trust is managed by a professional trustee who is subject to a fiduciary obligation

                                                                                    ii.      Subject to the federal EE Retirement Security Act

d.      Asset securization

                                                                                      i.      O, a bank makes loans to individuals who secure them with their homes

                                                                                    ii.      O makes business loans that cannot be secured, may not be able to recover

1.      O sells alls rights to payments under its entire portfolio of individual loans to T

2.      To pay for those rights, T sells passive equitable ownership shares in the trust to sophisticated investors

a.       The rights of repayment under the secured individual loans are segregated in trust

b.      The sophisticated investors need not consider the risk attending to O’s business loans when maing their investment in the asset securitization trust

c.       O is able to realize the full value of its portfolio of individual loans notwithstanding risits risk on the business loans

3.      Once the transaction is complete, T manages the portfolio of individual loans subject to a fiduciary duty to the sophisticated investors. Often hiring the bank that made the loans to undertake the day-to-day management of those loans.

Section B. Creation of a Trust

1.       Intent to Create a Trust

a.       The words trust or trustee need not be used

b.      RS 3rd Trusts § 13, Uniform Trust Code § 402(a)(2)(2000) – Sole question is did the grantor manifest an intention to create a trust

                                                              i.      Where the grantor conveys the property to a grantee to hold “for the use and benefit” of another, this is sufficient

c.       Lux v. Lux – Did grandmother devise property to her grandchildren in trust or outright

                                                              i.      “Maintained for the benefit of” deemed sufficient words to create a trust, named the executor, the trustee

d.      Jimenez v. Lee

Facts: P’s grandmother purchased a savings bond to use for p’s education in her name and her mother’s name. A client of her father’s deposited $500 in account for her in her name and her fathers and also $500 and also placed the same amounts in accounts for her siblings. D invested all of this money in commercial bank of Salem stock in his name as custodian for Betsey Lee.

Procedural History: P appeals from a decree that dismissed her complaint that her father was holding assets in trust for her.

Issue: Did the d breach his duty as trustee when he invested the trust funds as a custodian?

Holding: Yes. Defendant’s attempt to broaden his powers over the trust estate by investing the trust funds as custodian violated his duty to the beneficiary “to administer the trust soley in the interest of the beneficiary.”

Reasoning: It was clear that d knew and that the settlors had meant that the funds were to be held in trust for p’s education.

e.       Precatory language – expresses a “wish, hope, or recommendation that the property devised should be disposed of by the devisee in some particular manner, but this language does not clearly indicate whether the testator intends to create a trust (with a legal duty so to dispose of the property) or merely a moral obligation unenforceable in court.

                                                              i.      Precatory trust – an unenforceable disposition

                                                            ii.      Colton v. Colton, “ I recommend to her the care and protection of my mother and sister, and request her to make such gift and provision for them as in her judgment will be best

1.      May be merely words of suggestion, counsel or advice

2.      May be an imperative command

3.      Court concluded that it was an enforceable trust

4.      Lesson: DO not put recitals in testamentary instruments, be clear. I wish, but do not legally require that …

f.       Equitable charge

                                                              i.      If a testator devises property to a person, subject to the payment of a certain sum of money to a third person, the testator creates an equitable charge, not a trust.

g.      Elements of a gift (trusts only require intent, so failed gifts can become trusts)

                                                              i.      Intent

                                                            ii.      Acceptance – courts infer acceptance from the absence of refusal or disclaimer

                                                          iii.      Delivery – need not be physical – a constructive or symbolic delivery will do

1.      Constructive delivery – gives the donee the means of obtaining the property such as a key

2.      Symbolic delivery – gives the donee something symbolic of the object – written instrument (deed)

h.      The Hebrew University Association v. Nye

Facts: Professor Yahuda before his death forwarded certain of the books in his library to a warehouse in New Haven with instructions that they be packed for overseas shipment. The books remained in his name, there was no cosignee and they were never shipped. Ethel purchased the books from her husbands estate. They expressed to friends that they wanted to create a scholarship research center in Israel which would serve as a memorial to them. At a luncheon, Ethel announced that the library would be given to the Institute of Oriental Studies at the university in Jerusalem. The president of Israel was at the luncheon. She signed a release saying that she had give the library to the plaintiff. She told a friend that she could not have the item because it belonged to the university. She never finished sending the library.

Procedural History: P obtained a judgment declaring that it is the rightful owner of the library of Abraham S. Yahuda a distinguished Hebew Scholar who died in 1951. The court said that she had shown her intention to create a trust for p by her declaration to the new paper.

Issue: Did the decedent create a trust when she stated that she was going to give the library to the university in a news paper article?

Holding: She did not create a trust.

Rule: A gift which is imperfect for lack of a delivery will not be turned into a declaration of trust for no better reason than that it is imperfect for lack of a delivery

Reasoning: The evidence shows perhaps that she meant to give an inter vivos gift of which she never made delivery. It is not sufficient to declare yourself a donor. It was not evident that she considered herself a trustee.

Dicta: Perhaps under the law of gift, she created an inter vivos gift.

 

Second Case

The Hebrew University Association v. Nye

Superior Court of Connecticut, 1966

26 Conn. Supp. 342, 223 A.2d 397

Parskey J.

 

Facts: See above. The decedent gave to the p a memorandum containing a list of most of the contents of the library and all of the important books, documents and incunabula…

Issue: Did the decedent give the library as an inter vivos gift?

Holding: Yes, she gave an inter vivos gift to the library.

Rule: For constructive delivery, the donor must do that which, under the circumstances, will in reason be equivalent to an actual delivery. It must be as nearly perfect and complete as the nature of the property and the circumstances will permit. It is sufficient if manual delivery is impractical or inconvenient.

Reasoning: The delivery of the memorandum coupled with the decedent’s acts and declarations, which clearly show an intention to give and to divest herself of any ownership of the library, was sufficient to complete the gift. “An excessive regard for formalism should not defeat the ends of justice”. Holmes.

Dicta: Does not mean to abrogate the element of delivery.

i.        RS 3rd Trusts § 16(2) – If a property owner intends to make an outright gift inter vivos, but fails to make the transfer that is required in order to do so, the gift intention will not be given effect by treating it as a declaration of trust.

                                                              i.      Comment (d) – unless owner intended declaration of a trust, not a gift in the future, by clear and convincing evidence

2.      Necessity of Trust Property

a.       Elements of a Trust

                                                              i.      Trustee

                                                            ii.      Beneficiary

                                                          iii.      Trust property (res)

1.      Any interest in property that can be transferred

b.      Unthank v. Rippstein

Facts: Three days before his death C.P. Craft penned a lengthy personal letter to Mrs. Iva Rippstein.

Procedural History: Mrs. Ripstein, unsuccessfully sought to probate the writing as a holographic codicil to the will. The Court of Civil Appeals held that the writing was not a testamentary instrument which was subject to probate. The present suit was filed by Mrs. Rippstein against the executors of the estate of Craft for declaratory judgment adjudicating the liability of the executors to pay future installments as they mature. TC granted executors motion for summary judgment. Court of Civil Appeals reversed and rendered judgment for Mrs. Rippstein holding that the writing established a voluntary trust under which Craft bound his property to the extent of the promised payments and upon his death his legal heirs held the legal title for the benefit of Mrs. Rippstein to that portion of the estate required to make the promised monthly payments.

Issue: Does the letter constitute a declaration of trust whereby [Craft agrees to his estate making the payments]?

Holding: The notation in the letter did not constitute a declaration of trust.

Reasoning: The marginal notation in the letter is not enough to create a trust. The language in the notation cannot be expanded to show intention. No property was specifically specified for the trust res.

Note: No res is required if executor uses a pour over will. The letter was deemed not a holographic will. So there is no property to create a trust res.

c.       Trusts distinguished from debts

                                                              i.      The requirement of an identifiable trust res distinguishes trust res from a debt

                                                            ii.      Whether the recipient of the funds is entitled to use them as his own and commingle them with his own monies.

d.      Resulting trusts – trustee must reconvey the property to the beneficial owner upon demand. An equitable reversionary interest that arises by operation of law in two situations:

                                                              i.      Where an express trust fails or makes an incomplete disposition

1.      A has no descendants. X holds money in trust for O’s descendants or heirs.

                                                            ii.      Where one person pays the purchase price for property and causes title to the property to be taken in the name of another person who is not a natural object of the bounty of the purchaser

1.      Purchase money resulting trust – B purchases Blackacre with money supplied by A. Unless B can show that A intended to make a gift to B, B holds title to Blackacre on resulting trust for A

3.      Necessity of Trust Beneficiaries

a.       A trust must have one or more ascertainable beneficiaries. RS 3rd § 44. UTC §402(a)(3).

                                                              i.      There must be someone to whom the trustee owes a fiduciary duty, someone who can call the trustee to account

                                                            ii.      A private trust must be for the benefit of its beneficiaries

                                                          iii.      If beneficiaries are too indefinite to be ascertained, the trust fails; however a trust can be created for unborn children

b.      Unlike a private trust, a charitable trust need not have an ascertainable beneficiary to be valid.

c.       Clark v. Campbell

Facts: Man in his will gave his property to his trustees to distribute to his friends as they saw fit. Any residue was to be sold and become part his estate. He also said in his will that he would distribute much property to his friends before death.

Issue: Does the bequest for the benefit of the testator’s “friends” must fail for the want of certainty of the beneficiaries?

Holding: The trustees therefore hold title to the property enumerated in the paragraph under consideration, to be disposed of as part of the residue, and the trustees are so advised.

Rule: Where a gift is impressed with a trust ineffectively declared and incapable of taking effect because of the indefiniteness of the cestui que trust, the donee will hold the property in trust for the next taker under the will, or for the next of kind by way of a resulting trust.

Reasoning: A power unlike a trust, is not imperative and leaves the act to be done at the will of the donee of the power. There must be someone to claim the property and compel performance from the trustee. He intended to make an absolute gift to the trustees of the property. The language “I give to my trustees my property in trust to make disposal of to such of my friends as they shall select” is clear language. A class must be capable of delineation. The clause is an attempt to create a private trust. The word friends has no statutory or controlling limitation. No sufficient criterion is furnished to govern the selection of the individuals from the class.

d.      Valid power of appointment-. If the class of beneficiaries is so described that some person might reasonably be said to answer the description, the power is valid. An appointment is invalid, however, if it cannot be determined whether the appointee answers the description.

                                                              i.      The power of appointment is discretionary; it is a non fiduciary power.

                                                            ii.      Power of appointment cannot be coupled with fiduciary duty, do not use the word trustee when naming the person with power of appointment

Section C. Rights of the Beneficiaries to Distributions from the Trust

1.      Trusts can be divided intro mandatory and discretionary trusts

a.       Mandatory – the trustee must distribute all the income (no discretion to choose who gets the income or the amount)

b.      Discretionary trust – the trustee has discretion over payment of either the income or the principal or both, provides greater flexibility

                                                              i.      Spray trust – the trustee must distribute all of the income currently, but has discretion to determine who gets it and in what amount. If desired, the trustee could be given discretionary power to accumulate income and add it to principal.

                                                            ii.      May be limited by an ascertainable support standard

1.      Support trust - such as “amounts as are necessary to support in manner in which they have been accustomed.”

a.       Discretionary support trust – combines an explicit statement of discretion with a stated support standard – “such amounts as the trustee shall, in his uncontrolled discretion, deem necessary to support my children in the style living to which they have become accustomed.

c.       Marsman v. Nasca (“cappy”)

Facts: T was survived by second husband and daughter from first marriage. T gave 1/3 of the residue of her estate in trust to support her husband comfortably. The rest was given to her daughter. The will contained the following exculpatory clause: “No trustee hereunder shall ever be liable except for his own willful neglect or default.”. Her husband took out a mortgage to pay his bills. He married a new wife. He drew up a new will with his new wife to inherit the property. Husband asked trustor for additional funds because his business was at a standstill. Trustor said that he thought it would be possible to give principal. 1974 Sally agreed to all payments related to the house so that she could have it upon Cappy’s death. Sarah received a letter from Farr, the attorney asking if Margaret would be allowed to live in the house if Cappy predeceased her and no copy of the letter was sent to Cappy. Sara died. Her husband inherited the property. He sent a letter to Margaret to vacate.

Procedural History: TC held that Farr was in breach of his duty to Cappy when he neglected to inquire as to the latter’s finances. Cappy would not have conveyed the property to Sara had Farr met his duty. The judge ordered Farr to pay Sara’s husband out of the trust for the repairs made to the home etc. and to convey the house to Margaret.

Issue: Does a trustee, holding a discretionary power to pay principal for the comfortable support and maintenance of a beneficiary, have a duty to inquire into the financial resources of that beneficiary so as to recognize his needs? If so, what is the remedy for such failure?

Holding: The court agrees that Farr had a duty to inquire into the beneficiaries financial resources, but they think that the remedy not be that Marlette convey the house to Margarette. A constructive trust in favor of cappy’s estate should be placed on the amount from the trust that would have allowed Cappy to keep the home.

Rule: The trustee is held to a duty of inquiry into the needs of the beneficiary.

Reasoning: Sara’s will clearly placed a duty on Farr to look into Cappy’s finances. She specified that he should be maintained comfortably. Not necessary to save for illness in old age. He should have been able to live in the home in which he lived with the settlor unencumbered. Cappy did not know that he had the right to receive from the principal in order to maintain the home. Farr was not reckless, he is not liable. Since the clause was not an abuse of Farr’s fiduciary relationship with Sara at the time of drawing her will, the clause is effective.

d.      Duty to inquire into beneficiaries financial circumstances is a standard rule Kolodney and Trusts §187.3

e.       The beneficiaries other resources (fuzzy issue) – Professor Scott. – the presumption is that the settlor intended the beneficiary to receive his support from the trust estate regardless of the beneficiary’s other financial resources. Or vice versa. Mostly left up to the trust instrument to determine.

f.       Extended Discretion – If the trustee has simple discretion unqualified by adjectives such as sole, absolute, uncontrolled, or the like, the courts will not substitute their judgment for that of the trustee so long as the trustee “acts not only in good faith and from proper motives, but also within the bounds of a reasonable judgment.”

                                                              i.      RS 3rd §50  of trusts – Words such as absolute, unlimited or sole fiduciary discretion are not interpreted literally. Even under the broadest grant of fiduciary discretion, a trustee must act honestly and in a state of mind contemplated by the settlor. Thus, the court will not permit the trustee to act in bad faith or for some purpose or motive other than to accomplish the purposes of the discretionary power.

g.      Exculpatory Clauses

                                                              i.      Should burden of proof that the exculpatory clause is not a breach of fiduciary duty be shifted to the attorney?

1.      UTC § 1008(b) – An exculpatory term drafted or caused to be drafted by the trustee is invalid as an abuse of fiduciary or confidential relationship unless the trustee proves that the exculpatory term is fair under the circumstances and that its existence and contents were adequately communicated to the settlor (helps to ensure that the clause was not unwittingly embraced). Includes both banks and attorneys. Another attorney representing the client to read over the trust satisfies this provision.

                                                            ii.      Mandatory Arbitration Clauses

1.      Trust instrument provides that all disputes between the trustee and the beneficiary must be resolved by arbitration.

a.       There is growing support for arbitration

b.      Schoneberger v. Oelze, - says that beneficiaries cannot be denied their day in court

                                                          iii.      UTC §814 – The trustee shall exercise discretionary power in good faith and in accordance with the terms and purposes of the trust and the interest of the beneficiaries

Section D. Rights of the Beneficiary’s Creditors

1.      Discretionary Trusts

a.       Traditional Rule: The creditor cannot, by judicial order, compel the trustee of a discretionary trust to pay him

                                                              i.      Because the beneficiary has no right to a payment, neither does the creditor

                                                            ii.      In some states, the creditor may be entitled to an order directing the trustee to pay the creditor before the beneficiary

b.      Rule when trust contains an ascertainable support standard (say an amount necessary for support and education, from the principal if needed)

                                                              i.      Beneficiary cannot alienate their interest

                                                            ii.      Creditors cannot reach their interest; except suppliers of necessaries

                                                          iii.      Beneficiaries children and spouse may enforce claims for child support

                                                          iv.      Beneficiary may ask court to review the trust for bad faith on the part of the trustor

c.       UTC § 504 Discretionary Trust Code (2000 as amended 2004)

a.       In this section child includes any person for whom an order or judgment for child support has been entered in this or another State

b.      Except as otherwise provided in subsection (c), whether or not a trust contains a spendthrift provisions, a creditor of a beneficiary may not compel a distribution that is subject to the trustee’s discretion, even if:

                                                                                                                                      i.      The discretion is expressed in the form of a standard of distribution; or

                                                                                                                                    ii.      The trustee has abused the discretion

c.       To the extent a trustee has not complied with a standard of distribution or has abused a discretion

                                                                                                                                      i.      A distribution may be ordered by the court to satisfy a judgment or court order against the beneficiary for support or maintenance of the beneficiaries child, spouse, or former spouse; and

                                                                                                                                    ii.      The court shall direct the trustee to pay to the child, spouse or former spouse such amount as is equitable under the circumstances but not more than the amount the trustee would have been required to distribute to or for the benefit of the beneficiary had the trustee complied with the standard or not abused the discretion.

d.      This section does not limit the right of a beneficiary to maintain a judicial proceeding against a trustee for an abuse of discretion or failure to comply with a standard for distribution.

e.       The provisions of this section apply even if the beneficiary is the co-trustee of the trust

d.      Protective Trust – The trustee is directed to pay income to A, but if A’s creditors attach A’s interest, A’s mandatory income ceases, whereupon a discretionary trust arises.

                                                              i.      Discretionary trusts are protective

                                                            ii.      Mandatory trusts are predictable

                                                          iii.      Protective trusts are both

e.       General trust property policy – the beneficiaries interest in a trust is ordinarily freely transferable, both voluntarily and by sale and  involuntary to satisfy a judgment against the beneficiary

2.      Spendthrift Trusts – in contrast to general policy –a beneficiary of a spendthrift trust cannot voluntarily alienate her interest, nor can her creditors reach her interest in the trust. This is true even if it provides mandatory payments to the beneficiary

                          i.      In most states, provision must be inserted

                        ii.      Criticized for:

1.      Dead hand reach

2.      Creating a separate class

                      iii.      Scheffel v. Krueger

Facts: Grandmother left (later convicted child molester) grandson a trust that was to pay the net income of the trust for his support and education. Principal was not to be attached by creditors or by the beneficiary until age 50. He was convicted of molesting a young girl. Court awarded her over $500 K in damages.

Issue: Can a tort creditor attach a claim to a spendthrift trust?

Holding: No. The statute does not allow attachment unless there has been fraud or the beneficiary is also the settlor.

Rule: In the event the governing instrument so provides, a beneficiary of a trust shall not be able to transfer his or her right to future payments of income and principal, and a creditor of a beneficiary shall not be able to subject the beneficiary’s interest to the payment of its claim.

Reasoning: Court refused to go beyond the text of the statute which clearly did not allow any type of creditor to attach, even tort creditors. Additionally, the purpose of the trust was still in effect because he could use the funds for support and education while in prison and after.

Note: Isn’t a tort creditor different from a creditor that can investigate financial solvency.

                      iv.      Shelly v. Shelly Oregon – man was not paying child support and alimony, trust contained a spendthrift clause, court concluded that it had the power to make exceptions to the general rule that a spendthrift clause bars the claims of the beneficiaries creditors, the court held that the interest of the beneficiary of a trust should be subject to the claims for support of his children. Not sound public policy, society would have to support his children. Alimony should be decided on a case by case basis, but in this case, former wives were given support, but that wives and children could not reach the corpus of the trust. The court also named the children as trust beneficiaries. Children could only access corpus in an emergency deemed so by the trustee.

                        v.      UTC § 502 Spendthrift Provision

a.       A spendthrift provision is valid only if it restraints both voluntary and involuntary transfer of a beneficiaries interest

b.      A term of a trust providing that the interest of a b is held subject to a spendthrift trust or words of similar import, is sufficient to restrain both voluntary and involuntary transfer of the beneficiary’s interest

c.       A beneficiary may not transfer an interest in a trust in violation of a valid spendthrift provision and, except as otherwise provided in this [article], a creditor or assignee of the b may not reach the interest or a distribution by the trustee before its receipt by the b.

                      vi.      UTC § 503 Exceptions to Spendthrift Provision

a.        In this section, “child” includes any person for whom an order or judgment for child support has been entered or in this or another State

b.      Even if a trust contains a spendthrift provision, a b’s child, spouse or former spouse who has a judgment or court order against b for the support or maintenance, or judgment creditor who has provided services for the protection of a b’s interest in the trust, may obtain from a court an order attaching present or future distributions to or for the benefit of the beneficiary.

c.       A spendthrift provision is unenforceable against a claim of this state or the US to the extent a statute of this state or federal law so provides.

                    vii.      Lack of a spendthrift trust might increase the number of informal agreements where someone is charged with the care of another with potential costs to the beneficiaries

                  viii.      Child Support and Alimony – different in various states, some statutes, some protect the trust

                      ix.      Tort creditors – undecided, RS maybe moving that way, Sligh case overruled gave attachment for tort creditor. UTC §503 does not recognize claims.

                        x.      Furnishing necessary support – someone who has given necessary support can reach the beneficiaries interest

                      xi.      Federal tax lien – US can reach, federal tax laws trump state spendthrift statutes

                    xii.      Excess over amount needed for support – In NY creditors can reach the amount over what is needed for support, but only more than their station-in-life requires.

                  xiii.      Percentage levy, spendthrift caps – only a certain percentage can be reached

                  xiv.      Pension trusts – pension plan benefits may not be assigned except for child support, alimony and marital property rights

                    xv.      Bankruptcy – cannot be reached by creditors in bankruptcy for spendthrift and pensions

3.      Self-settled Asset Protection Trusts

a.       You cannot shield your assets from creditors by placing them in a trust for your own benefit

b.      Creditors can reach the max amt. that the trustee could pay the settlor or apply for the settlor’s benefit.

c.       Some states offer selft-settled protective trust laws, but the trustee must also be located in the state to be sure

                                                              i.      Cook Islands

                                                            ii.      Alaska as long as no fraud

                                                          iii.      Delaware – although allows child, spouse and tort

d. Federal Trade Commission v. Affordable Media LLC

                                                          iv.      Self-settled trust in Cook Islands with a duress clause activated by judgments

1.      Court found them in contempt because they would not comply, but could not do anything else because the trust was located in another jurisdiction

2.      Trusts for the state supported

a.       Self settled by individual or spouse or guardian

                                                                                                                                      i.      Corpus and all assets are available to the individual

b.      Discretionary

                                                                                                                                      i.      Assets in amounts available for the individual are considered available for them for purposes of Medicaid

                                                                                                                                    ii.      Usually not reachable by the state, but since trustee has duty to ensure beneficiary is supported at his discretion, state may be able to attach via court order on behalf of the beneficiary

c.       Mandatory support trust created by third party and b has legal right to the income, such income is treated as available to the b for the purposes of Medicaid

                                                                                                                                      i.      Spendthrift clause is unenforceable against the state

d.      Exceptions

                                                                                                                                      i.      A trust created for the support of the beneficiary by will by a spouse

                                                                                                                                    ii.      A trust established for a disabled individual from the individual’s property by a parent, grandparent, or guardian of the individual or by a court and the court provides that the state will receive upon the individual’s death all amounts remaining in the trust equal to all of the medical assistance paid by the state

Section E. Modification and Termination of Trusts

1.       Introduction

a.       If the settlor and all of the beneficiaries consent, an irrevocable trust may be modified or terminated

                                                              i.      No one else has any beneficial interest in the trust.

                                                            ii.      The trustee has no beneficial interest and cannot object.

                                                          iii.      Such a right exists even if the trust contains a spendthrift clause.

b.      Clafin doctrine - In the US, the great weight of authority holds that a trust cannot be terminated or modified prior to the time fixed for termination, even if all of the beneficiaries consent, if termination or modification would be contrary to a material purpose of the settlor. Clafin v. Clafin.

                                                              i.      A testator has a right to dispose of his own property with such restrictions and limitations, no repugnant to law, as he sees fit, and … his intentions ought to be carried out unless they contravene some positive rule of law, or are against public policy.

2.      Modification

a.       In re Trust of Stuchell

Facts: Petitioner has a retarded son who will require care his entire life. He received Social Security and Medicaid. Other beneficiary and the remaindermen approved the modification of the trust. If the other beneficiaries die first, he will not receive his social benefits due to his trust income.

Procedural History: Petitioner appeals from the TC’s dismissal of her petition for an approval of an agreement to modify a trust.

Issue: If all beneficiaries agree may the trust be modified?

Holding: Petitioner may not modify the trust.

Rule: RS 2nd Trusts §167(1) – The court will direct or permit the trustee to deviate from a term of the trust of owing to circumstances not known to the settlor and not anticipated by him compliance would defeat or substantially impair the accomplishment of the purposes of the trust; and in such case, if necessary to carry out the purposes of the trust, the court may direct or permit the trustee to do acts which are not authorized or are forbidden by the terms of the trust. Comment b The court will not permit the trustee to deviate from the terms of the trust merely because such deviation would be more advantageous to the beneficiaries than a compliance with such direction.

b.      Some states provide statutes that will change trust income amounts for widow(er)s who are having a hard time surviving

c.       Drafting advice – you should consider giving a beneficiary or a trust protector, the ability to modify or terminate the trust

d.      Administrative deviation and changed circumstances

                                                              i.      UTC § 105(b)(4) – should the settlor be permitted to make the trust’s terms immutable, that is, to opt out of the law of modification and termination. No.

1.      Posner. This would make wills more immutable than the constitution which can be amended.

e.       Equitable Distribution of dispositive terms and changed circumstances.

                                                              i.      Cal. Prob. Code § 15409 – authorizes the court to modify the administrative or dispositive provisions of the trust or terminate the trust if, owing to circumstances not known to the settlor and not anticipate by the settlor, the continuation of the trust under its terms would defeat or substantially impair the accomplishment of the purposes of the trust

                                                            ii.      UTC (Uniform Trust Code)

1.      § 412 – Modification or Termination Because of Unanticipated Circumstances or Inability to Administer Trust Effectively

a.       The court may modify the administrative or dispositive terms of a trust or terminate the trust if, because of circumstances not anticipated by the settlor, modification or termination will further the purposes of the trust. To the extent practicable, the modification must be made in accordance with the settlor’s probable intention.

b.      The court may modify the administrative terms of a trust if continuation of the trust on its existing terms would be impracticable or wasteful or impair the trust’s administration.

c.       Upon termination of a trust under this section, the trustee shall distribute the trust property in a manner consistent with the purposes of the trust.

                                                          iii.      Reformation and Modification for Tax Advantages

1.      In recent years, courts in several states have reformed or modified a trust so as to obtain income or estate tax advantages

2.      Reformation - Sometimes courts have corrected a lawyers error in drafting the instrument, conforms the instrument to what the settlor intended. UTC § 415; RS 3rd Property: Wills and Other Donative Transfers § 12.1.

3.      Equitable deviation – Modification to achieve the settlor’s probably intent in light of changed circumstances.

a.       UTC § 416 and RS 3rd § 12.2 – To achieve tax objectives, the court may modify the terms of a trust in a manner that is not contrary to the settlor’s probably intention.

                                                          iv.      Trust Protectors (state statutes in several states and UTC §808(b)-(d) – T may authorize P to (may or may not have a fiduciary duty, except Alaska no duty)

1.      Replace X with another corporate fiduciary or trustee

2.      Approve modifications to the trust’s administrative and dispositive provisions

3.      Terminate the trust

4.      Select a successor trust protector

3.      Termination

a.       Generally a trust cannot be terminated if it is a spendthrift trust, if the beneficiary is not to receive the principal until attaining a specified age, if it is a discretionary trust, or if it is for support of the beneficiary

b.      In re Estate of Brown

Facts: Settlor died in 1977. Settled his entire estate in trust. The trust was to be used to provide a college education for the children of his nephew. After all children had their educations, nephew and wife could use income from trust for their support. Remainder to be paid to their then living children in equal shares.

Procedural History: TC found that the trust could not be terminated. The trustee of a testamentary trust appeals an order of the WA SC granting the petition of the lifetime and residual beneficiaries of the trust to terminate it and to distribute the proceeds to the life tenants.

Issue: Does any material purpose of the trust remain to be accomplished, thus barring its termination?

Holding: There is a material purpose of the trust remaining to be accomplished.

General Rule: An active trust may not be terminated, even with the consent of all the beneficiaries, if a material purpose of the settlor remains to be accomplished.

Reasoning: Although the educational purpose of the trust had been achieved, assurance of a long income for the beneficiaries had not been achieved.

c.       Spendthrift trust – a valid restraint on the voluntary and involuntary transfer of the interest of the beneficiary is imposed

d.      It may be possible to terminate a testamentary trust by compromise agreement ending a will contest. Some courts however refuse to approve a will compromise where the compromise destroys a trust that is essential to a material purpose of the settlor.

e.       Spendthrift trusts and material purpose –

                                                              i.      Usually presumed to be material purpose

f.       Change in circumstances – UTC § 412(a) – allows for termination in light of circumstances not anticipated by the settlor if termination will further the purposes of the trust.

g.      Uneconomic trusts, combination and division

                                                              i.      UTC § 414 provides a mechanism for modifying or terminating small trusts in which the “value of the trust property is insufficient to justify the cost of administration.

                                                            ii.      §UTC 417 authorizes the combination or division of trusts if doing so does not adversely affect any beneficiary or the purposes of the trust.

h.      Unanimity of the beneficiaries – UTC § 411(e) – provides a mechanism for obtaining a modification or termination even without the consent of all the beneficiaries provided that the interest of a beneficiary who does not consent will be adequately protected

i.        Continuing viability of the material purpose standard – RS § 65 – authorizes modification without a showing of unanticipated circumstances and without the settlor’s consent if all the beneficiaries consent and the beneficiaries can show that the rationale for the modification outweighs the settlor’s material purpose

j.        Revocable versus irrevocable trusts

                                                              i.      Most states say written trusts are irrevocable

                                                            ii.      A trust is revocable unless declared to be irrevocable in California and some others

                                                          iii.      UTC § 602(a) – Unless the terms of a trust expressly provide that the trust is irrevocable, the settlor may revoke or amend the trust.

1.      A revocable trust cannot be revoked by will , unless the terms of the trust provides so.

2.      § UTC 602(c)(2)(A) – allows for revocation by a later will or codicil that expressly refers to the trust or specifically devises property that would otherwise have passed according to the terms of the trust.

4.      Trustee Removal – removal of the trustee is a remedy for breach of trust, not a modification of trust terms

a.       Courts are authorized to remove a trustee who is dishonest or who has engaged in a serious breach of trust, but may not remove a trustee for breach that is not serious or for a simple disagreement with a beneficiary

                                                              i.      If the settlor is aware of an asserted ground for removal at the time of naming the trustee, that ground will not serve as a basis for the later removal of the trustee unless the trustee is entirely unfit to serve.

                                                            ii.      Even if the beneficiary is dissatisfied with the performance of the trustee or is dissatisfied with the fees charfed, the court will not remove the trustee and appoint a new one unless the trustee has been guilty of breach of trust or has shown unfitness.

b.      UTC § 706 Removal of Trustee

a.       The settlor, a cotrustee, or a beneficiary may request the court to remove a trustee, or a trustee may be removed by the court on its own initiative.

b.      The court may remove a trustee if:

                                                                                                                                      i.      The trustee has committed a serious breach of trust

                                                                                                                                    ii.      Lack of cooperation among cotrustees substantially impairs the administration of the trust;

                                                                                                                                  iii.      Because of unfitness, unwillingness, or persistent failure of the trustee to administer the trust effectively, the court determines that removal of the trustee best serves the interests of the beneficiaries; or

                                                                                                                                  iv.      There has been a substantial change of circumstances or removal is requested by all of the qualified beneficiaries, the court finds that removal of the trustee best serves the interest of all the beneficiaries and is not inconsistent with a material purpose of the trust, and a suitable cotrustee or successor trustee is available

c.       Pending a final decision on a request to remove a trustee, or in lieu of or in addition to removing a trustee, the court may order such appropriate relief under Section 1001(b) as may be necessary to protect the trust property or the interests of the beneficiaries.


Chapter 12

Charitable Trusts

Section A. Nature of Charitable Purposes

1.      Unlike a private trust, a charitable trust need not have an ascertainable beneficiary to be valid.

2.      However to qualify as a charitable trust, the trust must have a valid charitable purpose

a.       Shenandoah Valley National Bank v. Taylor

Facts: T died testate on the 23rd day of April. His will was date April 21st, 1949, was duly admitted to probate and the Shenandoah Valley National Bank, the designated executor and trustee, qualified thereunder. He left the money in trust for the children in first and second grade to be paid out in equal shares for each child the day before Easter and the day before Christmas for to be used for their education. He left no other immediate heirs.

Issue: Does the will create a charitable trust?

Holding: It is not a charitable trust

Rule: RS 3rd §28 Charitable purposes include: (a) the relief of poverty; (b) the advancement of education; (c) the advancement of religion; (d) the promotion of health; (e) governmental or municipal purposes; and (f) other purposes the accomplishment of which is beneficial to the community.

Rule: Where a trust is set up and a class is designated as beneficiary which generally contains needy persons, the testator will be presumed to have intended as recipients those members of the class who are in necessitous circumstances.

Reasoning: Trusts which are merely benevolent cannot be upheld as charities. Benevolent objects include acts dictated by mere kindness, goodwill, or a disposition to do good. Other purposes that are beneficial to the community is an open ended category. No actual educational purpose is achieved by the trust. It just places income outside of the trust. Must consider the realities of life. Usually, it will be found that where a gift results in mere financial enrichment, a trust was sustained only when the court found and concluded from the entire context of the will that the ultimate intended recipients were poor or in necessitous circumstances. A trust from which income is to be paid at stated intervals to each member of a designated segment of the public without regard to whether or not the recipients are poor or in needs, is not for the relief of poverty, it is benevolence and it is not a charitable trust. Gifts which are made out of mere sentiment, and will have no practical result except the satisfying of a whim of the donor, are obviously lacking in the widespread social effect necessary to a charity.

3.      Charitable trusts may endure forever, they are privileged with an exemption from the Rule Against Perpetuities.

4.      Marsh v. The Frost Natl. Bank – create a trust that over 346 years generated income for every American of majority age.

5.      However, majority of states apply the wait and see rule.

6.      A trust may be a valid charitable trust even though the persons who directly benefit from it are limited in number. i.e. two scholarships a year.

a.       A trust to educate a particular named person is not charitable.

b.      Trust to educate descendants of the settlor is not charitable. RS 3rd § 28.

c.       On the other hand, a trust for education of young people has been held charitable even if in selecting beneficiaries the trustee must give preference to the descendants of the settlor’s grandparents. 4A Scott, supra, § 375.3.

d.      A trust may also be a valid charitable trust although the settlor delegates the selection of charitable purpose to the trustee. RS 3rd of trusts § 28.

e.       Charitable trusts should be given the benefit of the doubt because they try many experiments to which it would be inappropriate to delegate public funds. 4A Scott supra § 374.7.

7.      Noncharitable purpose trusts – in a significant number of states, the rule that trusts for noncharitable purposes fail for want of an ascertainable beneficiary has been relaxed somewhat.

a.       Many states have recognized honorary trusts, where the trustee has the power, but not a duty to perform.

b.      Second, quite a few states have enacted statutes that allow a trust for a specific purpose for 21 years or longer even if that purpose is not charitable and the trust lacks an ascertainable beneficiary, provided that the purpose is not capricious.

                                                              i.      Cemetary plot

                                                            ii.      Care of an animal

                                                          iii.      Saying of the masses

8.      Trusts to benefit a political party

a.       It is against public policy to endow perpetually a political party; hence, a trust to promote the success of a particular political party is not charitable.

b.      However, a trust for the improvement of the structure and methods of government in a manner advocated by a particular political part, is charitable. Breeden.

c.       Kidd. Locate existence of human soul. Considered charitable and gave it to institute for psychic research.

d.      Drafting advice –

                                                              i.      Locate exact legal name of charity

                                                            ii.      Make sure that charitable purpose is deductible for both state and federal tax purposes

                                                          iii.      Avoid benevolent or philanthropic trusts

9.      Mortmain statutes – Most states one had statutes permitting spouses and children to set aside death bed wills making gifts to charity. These have been eliminated as unconstitutional.

10.  Shaw’s alphabet Trusts

a.       The court held the alphabet trust was not for the advancement of education nor beneficial to the community, and therefore it was not a charitable trust. The court further held that the devise could not be treated as a private trust because it was not in favor of an ascertainable beneficiary. A compromise was reached.


Chapter 13

Trust Administration: The Fiduciary Obligation

Section A. Introduction

1.      A trust makes it possible to separate the benefits of ownership from the burdens of ownership.

2.      Trustee lacks a direct financial incentive to act with loyalty and care in management of the trust fund.

3.      Fiduciary Obligation – acts as a punishment rather than an incentive, minimizes agency costs in trust governance, applied with vigor

a.       John H Langbein, The Contractarian Basis of the Law of Trusts

                                                              i.      Uniform Trustee’s Power Act

1.      Empowers trustees to engage in every conceivable transaction that might wrest market advantage or enhance the value of the assets

b.      Standards

                                                              i.      Loyalty – forbids the trustee from self-dealing with trust assets and from engaging in conflict-of-interest transaction adverse to the trust

                                                            ii.      Care (prudence) – a reasonableness norm

                                                          iii.      Subsidiary rules

1.      Duties to keep and render accounts

2.      Furnish information

3.      Invest or preserve trust assets and make them productive

4.      Enforce and defend claims

5.      Diversity investments

6.      Minimize costs

c.       Agency Costs and the Fiduciary Obligation

                                                              i.      Principal -agent or agency problem – safeguarding the beneficiary against mismanagement or misappropriation by the trustee presents a problem

1.      Agent will make not make efforts that cost him more than he will benefit, even though the beneficiary will benefit

2.      Only the beneficiary has standing to bring suit against the principal

a.       In a private trust created gratuitously for the benefit of one or more beneficiaries is a vehicle for effecting the settlor’s donative intent. The settlor is also viewed as principal

b.      The net loss is called agency cost

d.      What of morality – Meinhard v. Salmon – higher level of duty than for those in arm’s length transactions

e.       Powers of the Trustee –Derived exclusively from the instrument creating the trust, task carry out the settlor’s intent

                                                              i.      Legislation

1.      An act that permits the settlor to incorporate by express reference in the trust instrument all or some enumerated statutory powers. This permits a trust drafter to omit a long and detailed list of trustee powers, incorporating the statutory powers instead.

2.      A broad trustee’s powers act that grants to trustees basic powers set forth in the statute, as exemplified by the UTPA §3(c). Express incorporation of statutory powers in the trust instrument is unnecessary under this type of statute.

f.       UTC § 815 – In addition to powers conferred by the terms of the trust, § 815 authorizes the trustee to exercise, “all powers over the trust property which an unmarried competent owner has over individually owned property” and “any other powers appropriate to achieve the proper investment, management, and distribution of the trust property.” “is intended to grant trustees the broadest possible powers.

g.      § 816 – enumerates more than two dozen specific transaction powers

                                                              i.      acquire and sell property

                                                            ii.      deposit trust money in an account in a regulated financial service institution

                                                          iii.      to pay or contest any claim

                                                          iv.      sign and deliver contracts

h.      UTC § 1012 – person who deals in good faith with trustee is not required to look into trustees validity

Section B. Loyalty – The trustee must administer the trust soley in the interest of the beneficiaries

1.      Hartman v. Hartle

Facts: Expressly directed her executors to sell her real estate and to divide the proceeds equally among her children. Executors sold part of the real estate at public auction to one of T’s sons who bought it for his sister who is the wife of one of the executors. She then sold the property to another.

Issue: Can an executor or his wife purchase property from the trust at his own sale?

Holding: No, executors must pay sister 1/5 share from the resale.

Rule: A trustee cannot purchase from himself at his own sale, and that his wife is subject to the same disability, unless leave so to do has been previously obtained under an order of the court.

2.      Gleeson’s Will – A trustee cannot deal in his individual capacity with the trust property

3.      Self-dealing- if the trustee engages in self-dealing, good faith and fairness to the beneficiaries are not enough to save the trustee from liability. In such a case further inquiry is made, unless settlor authorized the self-dealing beneficiaries can take any profits made on the dealing

Section C. The Duty of Prudence – objective standard of care

1.      RS 2nd Trusts § 174- The trustee is under a duty to the beneficiary in administering the trust to exercise such care and skills as a man of ordinary prudence would exercise in dealing with his own property.

2.      UTC § 804 – A trustee shall administer the trust as a prudent person would, by considering the purposes, terms, distributional requirements, and other circumstances of the trust using reasonable care, skill and caution.

3.      Modern portfolio theory – keeps in mind that some risk is necessary to profit. Safe investments expose trustees to risk of inflation.

4.      Modern-Trust-Investment Law

a.       Prudent investor standard – espouses modern portfolio theory

                                                                                      i.      An increased sensitivity to the tradeoff between risk and return

                                                                                    ii.      A diversification imperative

                                                                                  iii.      A reversal of the non delegation rule

b.      Sensitivity to Risk and Return – Uniform Prudent Investor Act

                                                                                      i.      §2 Standard of Care; portfolio Strategy; Risk and Return Objectives

a.       A trustee shall invest and manage trust assets as a prudent investor would, by considering the purposes, terms, distribution requirements, and other circumstances of the trust. In satisfying this standard, the trustee shall exercise reasonable care, skill and caution

b.      A trustee’s investment and management decisions respecting individual assets must be evaluated not in isolation but in the context of the trust portfolio as a whole and as a part of an overall investment strategy having risk and return objectives reasonably suited to the trust

c.       Among circumstances that the trustee shall considering investing and managing trust assets are such of the following as are relevant to the trust or its beneficiaries:

                                                                                                                                                              i.      General economic conditions;

                                                                                                                                                            ii.      The possible effect of inflation or deflation

                                                                                                                                                          iii.      The expected tax consequences

                                                                                                                                                          iv.      The role that each investment or course of action plays within the overall trust portfolio,

                                                                                                                                                            v.      The expected return from income and the appreciation of capital

                                                                                                                                                          vi.      Other resources of beneficiaries

                                                                                                                                                        vii.      Needs for liquidity, regularity of income, and presevation or appreciation of capital

                                                                                                                                                      viii.      An assets special relationship or special value, if any, top the purposes of the turst or to one or more of the beneficiaries

d.      A trustee shall take reasonable steps to verify facts relevant to the investment and management of trust assets.

e.       An estate may invest in any kind of property or type of investment consistent with the standards of this act

f.       A trustee who has special skills or expertise, or is named trustee in reliance upon the trustee’s representation that the trust has special skills or expertise, has a duty to use those special skills or expertise.

5.      Uniform Prudent Investor Act (UPIA) §3 Diversification – A trustee shall diversify the investments of the trust unless the trustee reasonably determines that, because of special circumstances, the purposes of the trust are better served without diversifying.

                                                                                      i.      The higher expected return on the investment compensates me for bearing the greater risk of the investment being disappointing.

1.      Market risk – reflects general economic and political conditions, interest rates, and so forth

2.      Industry risk – specific to firms in the particular industry or grouping

3.      Firm risk – factors that touch only the individual firm

In re Estate of James

Facts: When Levine died, 71% of his stock portfolio was in Kodak stock. He created three trusts. One a support trust for his wife, one a charitable trust and lastly a trust from which Mrs. Levine’s remaining assets would pour over at her death. She met with the trustees and they recommended selling some Kodak stock to pay expenses, but did not suggest any other investment strategies. In the space of two years the stock plummeted in value.

Procedural History: Petitioner sought judicial settlement of her account. The Surrogate found that the attorney was liable for damages in the decrease of the accounts. The Appellate Court modified soley as to damages. Held that the proper measure of damages was the value of capital that was lost – the value of the stock at the time at which it should have been sold and the value when it was sold.

Issue: At what point did the fiduciary breach his duty?

Holding: He breached his duty when he held the failing stock past the time when he should have sold it.

Reasoning: Prudent person rule dictates that the court examine the facts specific to every case. They did not take into account the scant growth rate of Kodak stock before any problems emerged. They never undertook a formal estate analysis to establish primary objectives and they didn’t follow their own rules of only allowing 20% portfolio concentration. They failed to conduct more than routine reviews of the Kodak holdings in this estate over a seven year period of steady decline.

Rule: A court may examine a fiduciary’s conduct throughout the entire period during which the investment was at issue. The court may then determine a reasonable time.

Damages Rule: The measure of the damages is the lost capital. The court should determine the value of the stock on the date at which it should have been sold and subtract from that figure the proceeds from the sale of the stock, or if the stock is still retained by the testate, the value of the stock at the time of accounting.

Estate of Collins

Facts:

·         Collins died in 1963

·         June 1965 court ordered the estate to be distributed

·         $80,000 trust principal

·         $50,000 available for investment after the college money was put aside

·         The objectors are beneficiaries established in the will of Ralph Collins, deceased.

o   Collin’s wife and children

§  Daughter specifically to get $4000 a year for five years for her college and graduate education

o   Collin’s mother and father

·         Carl Lamb and C.E. Milikin were respectively Collins business partner and lawyer. They were named as trustees in the will.

o   Made a loan to a client construction company

§  Secured by a trust deed and was payable in quarterly installments of interest only

§  Payable in full in three years

§  The trustees did not check to see if their were any liens against the property

§  They obtained and unaudited financial statement from the company

§  Defendants pledge 20% of the stock in their company

ú  Attorney’s never possessed the stock

§  1966, the construction company was placed in bankruptcy

§  1st trust deed owners foreclosed

§  $60,000 of the trust fund was lost

Procedural History: TC found that the trustees applied reasonable discretion with the funds? SC approved account, terminated the trust and discharged trustees, and beneficiaries appealed.

Issue: Did the trustees improperly invest trust funds?

Holding: 1) Defendant trustees failed to follow prudent investor standard with respect to administration of testamentary trust of which p’s were beneficiaries where they invested two thirds of trust principal in a single investment, invested in real property secured only by a second deed of trust, and made that investment without adequate investigation of either borrowers or collateral; (2) rule that determination whether an investment is proper must be made in light of circumstances existing at time of investment was inapplicable under circumstances of the case and (3) defendant trustees were subject to being surcharged, not because they lacked prescience of what would happen, but because they both lacked and ignored information about what was happening at the time.

California Prudent Investor Rule: The trustee is under a duty to the beneficiary to distribute risk of loss by reasonable diversification of investments, unless under the circumstances it is prudent not to do so.

Reasoning:

·         Defendants failed to follow the prudent investor standard first by investing two-thirds of the trust principal in a single investment, second by investing in real property secured only by a second deed of trust, and third by making that investment without adequate investigation of either the borrowers or collateral.

·         Second or other junior mortgages are not proper trust investments, unless taking a second mortgage is a reasonable method of settling a claim or making possible the sale of property.

·         In buying a mortgage for trust investment, the trustee should give careful attention to the valuation of the property, in order to make certain that his margin of security is adequate

Section D. Impartiality and the Principal and Income Problem

1.      The trustee must strike a fair balance between the beneficiaries, giving due regard to their respective interests.

a.       Dennis v. Rhode Island Hospital Trust Co.

Facts: Great grandmother left trust to cease 21 years after the death of her last surviving child. The principal to go to her surviving issue in 1991. Since, the death of their mother the two ps are her sole surviving issue. Trustee was authorized to purchase real estate. He purchased buildings worth 300,000 when he sold them they were worth 185,000.

Procedural History: TC found that p failed to act impartially. He had favored the trust’s income beneficiaries over the remaindermen. The court ordered a surcharge of $365,000 because he should have sold the buildings earlier when the character of downtown began to change.

Rule: The trustee’s duty to take corrective action would arise from the fact hat he knows that his present course of action will injure the remaindermen; settled law requires him to act impartially, with due regard for the respective interests of both parties.

RS of Trusts: Trustee is under a duty to the beneficiary who is ultimately entitled to the principal not to retain property which is certain or likely to depreciate in value, although the property yields a large income, unless he makes adequate provision for amortizing the depreciation.

Reasoning: Other than selling the buildings in 1950, he could have also modernized and refurbished them to ensure that they maintained income.

b.      In re Mulligan – widow and professionals were co-trustees, she refused to diversify even though professionals advised it. Remaindermen brought suit. Court said professionals should have pushed harder and then brought suit if she still refused.

Codicil to the Outline;) Omitted Heirs

Cy Pres

Estate of Myers

Facts: Testator set up a trust with the Bank of America as trustee. It was to be used to provide two college scholarships.

Procedural History: Petition for determination of entitlement to estate distribution was filed by Lena Lee Myers cousin, sole heir at law of Leta Puckett.  Executor and trustee under will filed objections. SC found there was no intestate property and that petitioner had no interest in trust property or income from such property and petitioner appealed. The Court of Appeal held that there was no intestate property; rather, doctrine of cy pres would apply to dispose of excess income generated by trust, the income of which was intended to provide scholarships in the sciences and religion to two students a year on a continuing basis.

Issue: Did the testatrix had a general charitable purpose?

Holding: Trusts for the advancement of education have a general charitable purpose.

Cy Pres: Doctrine if cy pres is an equitable power which makes it possible for a court to carry out a testamentary trust established for a particular charitable purpose if the testator has expressed a general charitable intent, and for some reason his purpose cannot be accomplished in the manner specified in the will; the court, to meet unexpected contingencies, directs the disposition of the property to some related charitable purpose in order to carry out the testator’s intention as nearly as possible.

Rule: If the income of the charitable trust is or becomes more than sufficient to achieve all the charitable objectives named by the donor in the manner prescribed by him, cy pres is generally applied to the surplus income, in the discretion of the court, since there is an impossibility of using the income to advance any of the charitable purposes of the settlor. There is authority for the proposition that where the testator has provided certain sums shall be paid to a certain number of person, and the income is or becomes more than sufficient for the purpose, the court may direct that larger sums be paid to the same number of persons. On the other hand in an English case where the income was such that there was more than sufficient funds to pay the designated sums to the designated number of recipients, the court allowed an increase in the number of beneficiaries

Reasoning: Article II of her will which was repealed before execution gave any remainder to the Regents of the University of California. Similar case gave scholarship to another person.

 

818-819